From owner-obm-rj@saci.mat.puc-rio.br Mon Oct 19 12:30:16 1998 Received: by saci.mat.puc-rio.br (AIX 3.2/UCB 5.64/4.03) id AA24438; Mon, 19 Oct 1998 12:30:15 -0200 Received: from mail.iis.com.br by saci.mat.puc-rio.br (AIX 3.2/UCB 5.64/4.03) id AA18034; Mon, 19 Oct 1998 12:30:01 -0200 Received: from default (rio-as1-tty03.iis.com.br [200.202.97.19]) by mail.iis.com.br (8.8.7/8.8.14) with SMTP id MAA06618; Mon, 19 Oct 1998 12:29:58 -0200 Reply-To: "Marcio" From: "Marcio" To: Cc: Subject: Novo membro da lista Date: Mon, 19 Oct 1998 07:58:42 -0300 Message-Id: <01bdfb4f$6f124b60$7861cac8@default> Mime-Version: 1.0 Content-Type: multipart/alternative; boundary="----=_NextPart_000_007C_01BDFB36.49C51360" X-Priority: 3 X-Msmail-Priority: Normal X-Mailer: Microsoft Outlook Express 4.71.1712.3 X-Mimeole: Produced By Microsoft MimeOLE V4.71.1712.3 Sender: owner-obm-rj@saci.mat.puc-rio.br Precedence: bulk Status: RO X-Status: X-Keywords: X-UID: 9 This is a multi-part message in MIME format. ------=_NextPart_000_007C_01BDFB36.49C51360 Content-Type: text/plain; charset="iso-8859-1" Content-Transfer-Encoding: quoted-printable Acabei de entrar nessa lista e j=E1 gostaria de propor um problema = cuja solucao eu ainda nao conheco. Nao deve ser um problema no nivel que = voces estao acostumados a fazer haja vista que ele foi proposto numa = prova de um curso IME-ITA: =20 Tem-se tres caixas com bolas numeradas. Na primeira caixa, tem-se 4 = bolas iguais com o n=FAmero 1. Na segunda caixa, tem-se 4 bolas iguais = com o n=FAmero 2. E na ultima, 7 bolas numeradas de 3 a 9.=20 Pede-se: a) Quantos numeros podem ser formados se tomarmos, nao necessariamente = nessa ordem, duas bolas da primeira caixa, duas da segunda e quatro da = terceira? b) Qual a soma de todos esse numeros?=20 A letra a. parece ser apenas simbolica. Quanto a letra "b", eu tentei = estabelecer o numero de vezes que cada numero aparece em cada posicao. = Mas isso nao me ajudou muito. =20 Se alguem ja tiver visto esse problema (letra b) favor responder = para a lista. []'s Marcio PS: Boa sorte para todos os membros dessa lista que irao fazer a 3a fase = da olimpiada brasileira no sabado. =20 ------=_NextPart_000_007C_01BDFB36.49C51360 Content-Type: text/html; charset="iso-8859-1" Content-Transfer-Encoding: quoted-printable
    Acabei de=20 entrar nessa lista e já gostaria de propor um problema cuja = solucao eu=20 ainda nao conheco. Nao deve ser um problema no nivel que voces estao = acostumados=20 a fazer haja vista que ele foi proposto numa prova de um curso=20 IME-ITA:
   =20
    Tem-se=20 tres caixas com bolas numeradas. Na primeira caixa, tem-se 4 bolas = iguais com o=20 número 1. Na segunda caixa, tem-se 4 bolas iguais com o = número 2.=20 E na ultima, 7 bolas numeradas de 3 a 9.
   =20 Pede-se:
a) Quantos = numeros podem ser=20 formados se tomarmos, nao necessariamente nessa ordem, duas bolas da = primeira=20 caixa, duas da segunda e quatro da terceira?
b) Qual a soma = de todos esse=20 numeros?
 
 
A letra a. = parece ser apenas=20 simbolica. Quanto a letra "b", eu tentei estabelecer o numero = de vezes=20 que cada numero aparece em cada posicao. Mas isso = nao me=20 ajudou muito.
     
    Se alguem ja tiver visto esse = problema (letra=20 b) favor responder para a lista.
 
[]'s=20 Marcio
 
PS: Boa=20 sorte para todos os membros dessa lista que irao fazer a 3a fase da = olimpiada=20 brasileira no sabado.
     
= ------=_NextPart_000_007C_01BDFB36.49C51360-- From owner-obm-rj@saci.mat.puc-rio.br Mon Oct 19 13:01:51 1998 Received: by saci.mat.puc-rio.br (AIX 3.2/UCB 5.64/4.03) id AA17508; Mon, 19 Oct 1998 13:01:50 -0200 Received: from mula.mat.puc-rio.br by saci.mat.puc-rio.br (AIX 3.2/UCB 5.64/4.03) id AA25177; Mon, 19 Oct 1998 13:01:34 -0200 Received: (from nicolau@localhost) by mula.mat.puc-rio.br (8.8.7/8.8.7) id NAA00689; Mon, 19 Oct 1998 13:01:30 -0200 Date: Mon, 19 Oct 1998 13:01:30 -0200 From: Nicolau Corcao Saldanha Message-Id: <199810191501.NAA00689@mula.mat.puc-rio.br> To: mcohen@iis.com.br Subject: Re: Novo membro da lista Cc: obm-rj@saci.mat.puc-rio.br Sender: owner-obm-rj@saci.mat.puc-rio.br Precedence: bulk Status: RO X-Status: X-Keywords: X-UID: 10 > Acabei de entrar nessa lista e j=E1 gostaria de propor um problema = > cuja solucao eu ainda nao conheco. Nao deve ser um problema no nivel que = > voces estao acostumados a fazer haja vista que ele foi proposto numa = > prova de um curso IME-ITA: > Tem-se tres caixas com bolas numeradas. Na primeira caixa, tem-se 4 = > bolas iguais com o n=FAmero 1. Na segunda caixa, tem-se 4 bolas iguais = > com o n=FAmero 2. E na ultima, 7 bolas numeradas de 3 a 9.=20 > Pede-se: > a) Quantos numeros podem ser formados se tomarmos, nao necessariamente = > nessa ordem, duas bolas da primeira caixa, duas da segunda e quatro da = > terceira? > b) Qual a soma de todos esse numeros?=20 Oi Marcio, pelo que entendi as bolas formam os algarismos de um inteiro de oito algarismos escrito na base 10, certo? Nao entendi para que quatro bolas nas duas primeiras caixas se soh serao usadas duas das bolas... Para o item (a), podemos selecionar as 4 bolas de 35 = (7*6*5*4)/(4*3*2*1) maneiras diferentes, e depois disso podemos ordenar as bolas de 8! maneiras diferentes. Como trocando os dois 1's ou os dois 2's de lugar o numero nao muda temos 35*(8!)/4 = 352800 numeros distintos. (Serah que devemos contar os numeros com repeticoes?) Para o item (b), observemos que em cada posicao o algarismo 1 aparece 1/4 do tempo, o algarismo 2 tambem 1/4 do tempo e os demais algarismos durante a mesma proporcao de tempo, ou seja, 1/14. Assim o valor medio de cada algarismo eh: 1/4 + 2/4 + 3/14 + 4/14 + ... + 9/14 = 15/4. A media dos numeros eh portanto 15/4 * 11111111 e a soma eh 15/4 * 11111111 * 352800 = 14699999853000. (De novo, sem contar repeticoes...) []s, N. From owner-obm-rj@saci.mat.puc-rio.br Mon Oct 19 21:38:15 1998 Received: by saci.mat.puc-rio.br (AIX 3.2/UCB 5.64/4.03) id AA26777; Mon, 19 Oct 1998 21:38:13 -0200 Received: from mail.iis.com.br by saci.mat.puc-rio.br (AIX 3.2/UCB 5.64/4.03) id AA27029; Mon, 19 Oct 1998 21:37:56 -0200 Received: from default (rio-as6-tty02.iis.com.br [200.202.97.98]) by mail.iis.com.br (8.8.7/8.8.14) with SMTP id VAA28532; Mon, 19 Oct 1998 21:37:53 -0200 Reply-To: "Marcio" From: "Marcio" To: "Nicolau Corcao Saldanha" Cc: Subject: Re: Novo membro da lista Date: Mon, 19 Oct 1998 16:46:19 -0300 Message-Id: <01bdfb99$24741f80$3261cac8@default> Mime-Version: 1.0 Content-Type: text/plain; charset="iso-8859-1" Content-Transfer-Encoding: 7bit X-Priority: 3 X-Msmail-Priority: Normal X-Mailer: Microsoft Outlook Express 4.71.1712.3 X-Mimeole: Produced By Microsoft MimeOLE V4.71.1712.3 Sender: owner-obm-rj@saci.mat.puc-rio.br Precedence: bulk Status: RO X-Status: X-Keywords: X-UID: 11 -----Original Message----- From: Nicolau Corcao Saldanha To: mcohen@iis.com.br Cc: obm-rj@mat.puc-rio.br Date: Segunda-feira, 19 de Outubro de 1998 12:01 Subject: Re: Novo membro da lista >> b) Qual a soma de todos esse numeros? > >Oi Marcio, pelo que entendi as bolas formam os algarismos >de um inteiro de oito algarismos escrito na base 10, certo? >Nao entendi para que quatro bolas nas duas primeiras caixas >se soh serao usadas duas das bolas... Realmente, essas quatro bolas ao inves de duas nao servem para nada. Eu simplesmente reproduzi porque eu lembrei que tinha esse dado na questao. Adorei a resolucao. Essa parte de fazer as medias dos numeros a partir da media de cada algarismo nem tinha passado perto da minha cabeca Obrigado, Marcio >Para o item (a), podemos selecionar as 4 bolas de 35 = (7*6*5*4)/(4*3*2*1) >maneiras diferentes, e depois disso podemos ordenar as bolas de 8! >maneiras diferentes. Como trocando os dois 1's ou os dois 2's >de lugar o numero nao muda temos 35*(8!)/4 = 352800 numeros distintos. >(Serah que devemos contar os numeros com repeticoes?) > >Para o item (b), observemos que em cada posicao o algarismo 1 >aparece 1/4 do tempo, o algarismo 2 tambem 1/4 do tempo >e os demais algarismos durante a mesma proporcao de tempo, >ou seja, 1/14. Assim o valor medio de cada algarismo eh: >1/4 + 2/4 + 3/14 + 4/14 + ... + 9/14 = 15/4. >A media dos numeros eh portanto 15/4 * 11111111 e a soma eh >15/4 * 11111111 * 352800 = 14699999853000. > >(De novo, sem contar repeticoes...) > >[]s, N. > From owner-obm-rj@saci.mat.puc-rio.br Fri Oct 23 11:53:59 1998 Received: by saci.mat.puc-rio.br (AIX 3.2/UCB 5.64/4.03) id AA23196; Fri, 23 Oct 1998 11:53:58 -0200 Received: from mail.iis.com.br by saci.mat.puc-rio.br (AIX 3.2/UCB 5.64/4.03) id AA27032; Fri, 23 Oct 1998 11:53:44 -0200 Received: from default (rio-as4-tty12.iis.com.br [200.202.97.76]) by mail.iis.com.br (8.8.7/8.8.14) with SMTP id LAA08959 for ; Fri, 23 Oct 1998 11:53:31 -0200 Reply-To: "Marcio" From: "Marcio" To: Subject: The USRR Olympiad Problem Book Date: Fri, 23 Oct 1998 07:22:11 -0300 Message-Id: <01bdfe6e$fed56ee0$4c61cac8@default> Mime-Version: 1.0 Content-Type: multipart/alternative; boundary="----=_NextPart_000_0019_01BDFE55.D98836E0" X-Priority: 3 X-Msmail-Priority: Normal X-Mailer: Microsoft Outlook Express 4.71.1712.3 X-Mimeole: Produced By Microsoft MimeOLE V4.71.1712.3 Sender: owner-obm-rj@saci.mat.puc-rio.br Precedence: bulk Status: RO X-Status: X-Keywords: X-UID: 13 This is a multi-part message in MIME format. ------=_NextPart_000_0019_01BDFE55.D98836E0 Content-Type: text/plain; charset="iso-8859-1" Content-Transfer-Encoding: quoted-printable Esse livro do subject eh um livro muito interessante com problemas = legais e todos eles resolvidos. De acordo com o prefacio do livro = (volume 1?), ele tem origem russa, e existem tres volumes. O primeiro = sobre algebra e aritmetica, o segundo sobre geometria plana e o terceiro = sobre geometria espacial. Gostaria de saber se alguem tem alguma informacao sobre os outros = dois volumes. O primeiro eu ja vi ate na secretaria da obm, no IMPA = (traduzido para o Ingles). Alguem aqui sabe se foi publicada alguma = versao em Ingles dos outros dois volumes? Em caso afirmativo, onde ele = pode ser encontrado? ------=_NextPart_000_0019_01BDFE55.D98836E0 Content-Type: text/html; charset="iso-8859-1" Content-Transfer-Encoding: quoted-printable
    Esse=20 livro do subject eh um livro muito interessante com problemas legais e = todos=20 eles resolvidos. De acordo com o prefacio do livro (volume 1?), ele tem = origem=20 russa, e existem tres volumes. O primeiro sobre algebra e aritmetica, o = segundo=20 sobre geometria plana e o terceiro sobre geometria = espacial.
 
    Gostaria=20 de saber se alguem tem alguma informacao sobre os outros dois volumes. O = primeiro eu ja vi ate na secretaria da obm, no IMPA (traduzido para o = Ingles).=20 Alguem aqui sabe se foi publicada alguma versao em Ingles dos outros = dois=20 volumes? Em caso afirmativo, onde ele pode ser=20 encontrado?
------=_NextPart_000_0019_01BDFE55.D98836E0-- From owner-obm-rj@saci.mat.puc-rio.br Fri Oct 23 15:47:55 1998 Received: by saci.mat.puc-rio.br (AIX 3.2/UCB 5.64/4.03) id AA25696; Fri, 23 Oct 1998 15:47:53 -0200 Received: from mula.mat.puc-rio.br by saci.mat.puc-rio.br (AIX 3.2/UCB 5.64/4.03) id AA16986; Fri, 23 Oct 1998 15:47:38 -0200 Received: (from nicolau@localhost) by mula.mat.puc-rio.br (8.8.7/8.8.7) id PAA03956; Fri, 23 Oct 1998 15:47:26 -0200 Date: Fri, 23 Oct 1998 15:47:26 -0200 From: Nicolau Corcao Saldanha Message-Id: <199810231747.PAA03956@mula.mat.puc-rio.br> To: mcohen@iis.com.br Subject: Re: The USRR Olympiad Problem Book Cc: obm-rj@saci.mat.puc-rio.br Sender: owner-obm-rj@saci.mat.puc-rio.br Precedence: bulk Status: RO X-Status: X-Keywords: X-UID: 14 Caro Marcio, Nao conheco os volumes 2 e 3, mas eles provavelmente podem ser encomendados pelo IMPA. Gugu, voce sabe de alguma coisa? []s, N. From owner-obm-rj@saci.mat.puc-rio.br Mon Nov 2 13:31:07 1998 Received: by saci.mat.puc-rio.br (AIX 3.2/UCB 5.64/4.03) id AA24126; Mon, 2 Nov 1998 13:31:06 -0200 Received: from mail.iis.com.br by saci.mat.puc-rio.br (AIX 3.2/UCB 5.64/4.03) id AA30266; Mon, 2 Nov 1998 13:30:50 -0200 Received: from Rffsa.rffsa.gov.br (rio-as5-tty13.iis.com.br [200.202.97.93]) by mail.iis.com.br (8.8.7/8.8.14) with SMTP id NAA20004 for ; Mon, 2 Nov 1998 13:30:38 -0200 Reply-To: "Marcio" From: "Marcio" To: Subject: cos (arccos 5) = 5? Date: Mon, 2 Nov 1998 08:57:57 -0300 Message-Id: <01be0658$22a54de0$5d61cac8@Rffsa.rffsa.gov.br> Mime-Version: 1.0 Content-Type: multipart/alternative; boundary="----=_NextPart_000_0006_01BE063E.FD5815E0" X-Priority: 3 X-Msmail-Priority: Normal X-Mailer: Microsoft Outlook Express 4.71.1712.3 X-Mimeole: Produced By Microsoft MimeOLE V4.71.1712.3 Sender: owner-obm-rj@saci.mat.puc-rio.br Precedence: bulk Status: RO X-Status: X-Keywords: X-UID: 15 This is a multi-part message in MIME format. ------=_NextPart_000_0006_01BE063E.FD5815E0 Content-Type: text/plain; charset="iso-8859-1" Content-Transfer-Encoding: quoted-printable Gostaria de fazer uma pergunta que me surgiu ha pouco tempo atras: Existe algum conjunto que esteja para as funcoes trigonometricas assim = como os numeros complexos estao para a "radiciacao" dos numeros reais? = Nao sei se consegui expressar bem minha pergunta, mas o que eu quero = saber e o seguinte:=20 podemos dizer que ((-1)^.5)^2 =3D i^2 =3D 1 pertence aos reais, embora = (-1)^.5 nao pertenca. Podemos, procedendo dessa forma, dizer por exemplo que cos(arccos 5) =3D = 5?=20 A duvida surgiu quando eu estava lendo um texto na internet sobre a = resolucao da equacao completa do terceiro grau. Dada uma equacao do tipo = x^3 -3px + q =3D 0 (apos devidas simplificacoes de uma eq. completa = ax^3+bx^2+cx+d=3D0), o autor do texto sugeria a seguinte formula: x =3D 2 p^(1/2) cos ((1/3) arccos (q)p^(-3/2)) , deduzida a partir = do usual sistema {t + u=3Dq; tu=3Dp^3/3} do qual x=3Dt^1/3 + u^1/3 e = solucao de x^3 + px + q} para encontrar uma das raizes dessa equacao (e = consequentemente as outras duas). O que eu estranhei foi que o texto = dizia que essa era uma maneira de contornar o problema que a maioria das = calculadoras tem em lidar com operacoes do tipo exponenciacao com = numeros complexos (ja que a formula comum envolve muitos radicais).=20 O autor disse ter se baseado na identidade cos 3x =3D 4(cos x)^3 - = 3(cos x) para chegar a isso. Abracos, Marcio ------=_NextPart_000_0006_01BE063E.FD5815E0 Content-Type: text/html; charset="iso-8859-1" Content-Transfer-Encoding: quoted-printable
    Gostaria de fazer = uma=20 pergunta que me surgiu ha pouco tempo atras:
 
Existe algum conjunto que esteja = para as funcoes=20 trigonometricas assim como os numeros complexos estao para a=20 "radiciacao" dos numeros reais? Nao sei se consegui expressar = bem=20 minha pergunta, mas o que eu quero saber e o seguinte:
podemos dizer que ((-1)^.5)^2 =3D = i^2 =3D 1 pertence=20 aos reais, embora (-1)^.5 nao pertenca.
Podemos, procedendo dessa forma, = dizer por=20 exemplo que cos(arccos 5) =3D 5?
 
A duvida surgiu quando eu estava = lendo um texto=20 na internet sobre a resolucao da equacao completa do terceiro grau. Dada = uma=20 equacao do tipo x^3 -3px + q =3D 0 (apos devidas simplificacoes de uma = eq.=20 completa ax^3+bx^2+cx+d=3D0), o autor do texto sugeria a seguinte=20 formula:
    x =3D 2 p^(1/2) = cos ((1/3)=20 arccos (q)p^(-3/2))  , deduzida a partir do usual sistema {t + = u=3Dq;=20 tu=3Dp^3/3} do qual x=3Dt^1/3 + u^1/3 e solucao de x^3 + px + q} para = encontrar uma=20 das raizes dessa equacao (e consequentemente as outras duas). O que eu = estranhei=20 foi que o texto dizia que essa era uma = maneira de=20 contornar o problema que a maioria das calculadoras tem em lidar com = operacoes=20 do tipo exponenciacao com numeros complexos (ja que a formula comum = envolve=20 muitos radicais).
 
    O autor disse ter = se baseado=20 na identidade cos 3x =3D 4(cos x)^3 - 3(cos x) para chegar a = isso.
 
    = Abracos,
   =20 Marcio
------=_NextPart_000_0006_01BE063E.FD5815E0-- From owner-obm-rj@saci.mat.puc-rio.br Tue Nov 3 14:17:43 1998 Received: by saci.mat.puc-rio.br (AIX 3.2/UCB 5.64/4.03) id AA24995; Tue, 3 Nov 1998 14:17:41 -0200 Received: from mula.mat.puc-rio.br by saci.mat.puc-rio.br (AIX 3.2/UCB 5.64/4.03) id AA18845; Tue, 3 Nov 1998 14:17:20 -0200 Received: (from nicolau@localhost) by mula.mat.puc-rio.br (8.8.7/8.8.7) id OAA13041; Tue, 3 Nov 1998 14:17:07 -0200 Date: Tue, 3 Nov 1998 14:17:07 -0200 From: Nicolau Corcao Saldanha Message-Id: <199811031617.OAA13041@mula.mat.puc-rio.br> To: mcohen@iis.com.br Subject: Re: cos (arccos 5) = 5? Cc: obm-rj@saci.mat.puc-rio.br Sender: owner-obm-rj@saci.mat.puc-rio.br Precedence: bulk Status: RO X-Status: X-Keywords: X-UID: 16 Oi Marcio, Nao tenho certeza se entendi bem suas perguntas, mas é possível resolver a equação $cos z = 5$ nos numeros complexos. As soluções são os números da forma $2 k \pi + b i$ onde $k \in Z$, $\pi$ é o número que você conhece e $b = arc cosh 5$. A funcao $cosh t$, conhecida como cosseno hiperbolico, eh definida por $cosh t = (e^t + e^(-t))/2$. Analogamente, $senh t = (e^t - e^(-t))/2$. Para um número complexo $z = x + i y$ temos $cos z = cos x cosh y - i sen x senh y$ e $sen z = sen x cosh y + i cos x senh y$. Assim, a equação que você propos, $5 = cos(arc cos 5)$, é correta já nos números complexos. BTW, os números complexos são em vários sentidos o ponto "certo" para parar: aumentar mais ainda o conjunto dos números ou estraga a álgebra (como com os quatérnios, que não comutam) ou cria infinitos/infinitesimais. []s, N. From gugu@impa.br Tue Nov 3 14:56:58 1998 Received: from brahms.fluid.impa.br by saci.mat.puc-rio.br (AIX 3.2/UCB 5.64/4.03) id AA26714; Tue, 3 Nov 1998 14:56:57 -0200 Received: from Euler.impa.br by Brahms (5.x/SMI-SVR4) id AA11875; Tue, 3 Nov 1998 14:52:02 -0200 Received: from Gauss.impa.br (gauss.impa.br [147.65.4.1]) by Euler.impa.br (8.8.6/8.8.6) with ESMTP id OAA18579; Tue, 3 Nov 1998 14:56:54 -0200 (EDT) From: Carlos Gustavo Tamm de Araujo Moreira Received: by Gauss.impa.br (8.8.8) id OAA01460; Tue, 3 Nov 1998 14:56:53 -0200 (EDT) Message-Id: <199811031656.OAA01460@Gauss.impa.br> Subject: Re: cos (arccos 5) = 5? To: mcohen@iis.com.br Date: Tue, 3 Nov 1998 14:56:53 -0200 (EDT) Cc: nicolau@Euler.impa.br (Nicolau Corcao Saldanha) In-Reply-To: <01be0658$22a54de0$5d61cac8@Rffsa.rffsa.gov.br> from "Marcio" at Nov 2, 98 08:57:57 am X-Mailer: ELM [version 2.4 PL25] Mime-Version: 1.0 Content-Type: text/plain; charset=US-ASCII Content-Transfer-Encoding: 7bit Status: RO X-Status: X-Keywords: X-UID: 17 Alem do que o Nicolau falou,sobre equacoes do terceiro grau,ou aparecem na formula raizes cubicas de numeros reais(caso em que nao ha' necessidade de usar funcoes trigonometricas) ou de numeros complexos nao reais,caso em que se pode usar funcoes trigonometricas para calcular as raizes,e os arcocossenos que aparecem sao todos de numeros entre -1 e 1. A formula a que me refiro da' as raizes de x^3+px+q=0 por x=raizcub(-q/2+raizquad(q^2/4+p^3/27))+raizcub(-q/2-raizquad(q^2/4+p^3/27)) Abracos, Carlos Gustavo Moreira(Gugu) > >This is a multi-part message in MIME format. > >------=_NextPart_000_0006_01BE063E.FD5815E0 >Content-Type: text/plain; > charset="iso-8859-1" >Content-Transfer-Encoding: quoted-printable > > Gostaria de fazer uma pergunta que me surgiu ha pouco tempo atras: > >Existe algum conjunto que esteja para as funcoes trigonometricas assim = >como os numeros complexos estao para a "radiciacao" dos numeros reais? = >Nao sei se consegui expressar bem minha pergunta, mas o que eu quero = >saber e o seguinte:=20 >podemos dizer que ((-1)^.5)^2 =3D i^2 =3D 1 pertence aos reais, embora = >(-1)^.5 nao pertenca. >Podemos, procedendo dessa forma, dizer por exemplo que cos(arccos 5) =3D = >5?=20 > >A duvida surgiu quando eu estava lendo um texto na internet sobre a = >resolucao da equacao completa do terceiro grau. Dada uma equacao do tipo = >x^3 -3px + q =3D 0 (apos devidas simplificacoes de uma eq. completa = >ax^3+bx^2+cx+d=3D0), o autor do texto sugeria a seguinte formula: > x =3D 2 p^(1/2) cos ((1/3) arccos (q)p^(-3/2)) , deduzida a partir = >do usual sistema {t + u=3Dq; tu=3Dp^3/3} do qual x=3Dt^1/3 + u^1/3 e = >solucao de x^3 + px + q} para encontrar uma das raizes dessa equacao (e = >consequentemente as outras duas). O que eu estranhei foi que o texto = >dizia que essa era uma maneira de contornar o problema que a maioria das = >calculadoras tem em lidar com operacoes do tipo exponenciacao com = >numeros complexos (ja que a formula comum envolve muitos radicais).=20 > > O autor disse ter se baseado na identidade cos 3x =3D 4(cos x)^3 - = >3(cos x) para chegar a isso. > > Abracos, > Marcio > >------=_NextPart_000_0006_01BE063E.FD5815E0 >Content-Type: text/html; > charset="iso-8859-1" >Content-Transfer-Encoding: quoted-printable > > > > > >http-equiv=3DContent-Type> > > > >
    Gostaria de fazer = >uma=20 >pergunta que me surgiu ha pouco tempo atras:
>
 
>
Existe algum conjunto que esteja = >para as funcoes=20 >trigonometricas assim como os numeros complexos estao para a=20 >"radiciacao" dos numeros reais? Nao sei se consegui expressar = >bem=20 >minha pergunta, mas o que eu quero saber e o seguinte:
>
podemos dizer que ((-1)^.5)^2 =3D = >i^2 =3D 1 pertence=20 >aos reais, embora (-1)^.5 nao pertenca.
>
Podemos, procedendo dessa forma, = >dizer por=20 >exemplo que cos(arccos 5) =3D 5?
>
 
>
A duvida surgiu quando eu estava = >lendo um texto=20 >na internet sobre a resolucao da equacao completa do terceiro grau. Dada = >uma=20 >equacao do tipo x^3 -3px + q =3D 0 (apos devidas simplificacoes de uma = >eq.=20 >completa ax^3+bx^2+cx+d=3D0), o autor do texto sugeria a seguinte=20 >formula:
>
    x =3D 2 p^(1/2) = >cos ((1/3)=20 >arccos (q)p^(-3/2))  , deduzida a partir do usual sistema {t + = >u=3Dq;=20 >tu=3Dp^3/3} do qual x=3Dt^1/3 + u^1/3 e solucao de x^3 + px + q} para = >encontrar uma=20 >das raizes dessa equacao (e consequentemente as outras duas). O que eu = >estranhei=20 >foi que o texto dizia que essa era uma = >maneira de=20 >contornar o problema que a maioria das calculadoras tem em lidar com = >operacoes=20 >do tipo exponenciacao com numeros complexos (ja que a formula comum = >envolve=20 >muitos radicais).
>
 
>
    O autor disse ter = >se baseado=20 >na identidade cos 3x =3D 4(cos x)^3 - 3(cos x) para chegar a = >isso.
>
 
>
    = >Abracos,
>
   =20 >Marcio
> >------=_NextPart_000_0006_01BE063E.FD5815E0-- > > From owner-obm-rj@saci.mat.puc-rio.br Tue Nov 3 23:05:27 1998 Received: by saci.mat.puc-rio.br (AIX 3.2/UCB 5.64/4.03) id AA23056; Tue, 3 Nov 1998 23:05:25 -0200 Received: from mail.iis.com.br by saci.mat.puc-rio.br (AIX 3.2/UCB 5.64/4.03) id AA18700; Tue, 3 Nov 1998 23:05:10 -0200 Received: from Rffsa.rffsa.gov.br (rio-as3-tty02.iis.com.br [200.202.97.50]) by mail.iis.com.br (8.8.7/8.8.14) with SMTP id XAA30795 for ; Tue, 3 Nov 1998 23:05:06 -0200 From: "Marcio" To: Subject: errata Date: Mon, 2 Nov 1998 13:05:41 -0300 Message-Id: <01be067a$a3db7d40$461bf0c8@Rffsa.rffsa.gov.br> Mime-Version: 1.0 Content-Type: multipart/alternative; boundary="----=_NextPart_000_0011_01BE0661.7E8E4540" X-Priority: 3 X-Msmail-Priority: Normal X-Mailer: Microsoft Outlook Express 4.71.1712.3 X-Mimeole: Produced By Microsoft MimeOLE V4.71.1712.3 Sender: owner-obm-rj@saci.mat.puc-rio.br Precedence: bulk Reply-To: obm-rj@saci.mat.puc-rio.br Status: RO X-Status: X-Keywords: X-UID: 18 This is a multi-part message in MIME format. ------=_NextPart_000_0011_01BE0661.7E8E4540 Content-Type: text/plain; charset="iso-8859-1" Content-Transfer-Encoding: quoted-printable onde esta escrito "x^3 -3px + q =3D 0" leia-se "x^3 -3px - 2q =3D 0" -----Original Message----- From: Marcio To: obm-rj@saci.mat.puc-rio.br Date: Segunda-feira, 2 de Novembro de 1998 12:32 Subject: cos (arccos 5) =3D 5? =20 =20 Gostaria de fazer uma pergunta que me surgiu ha pouco tempo = atras: =20 Existe algum conjunto que esteja para as funcoes trigonometricas = assim como os numeros complexos estao para a "radiciacao" dos numeros = reais? Nao sei se consegui expressar bem minha pergunta, mas o que eu = quero saber e o seguinte:=20 podemos dizer que ((-1)^.5)^2 =3D i^2 =3D 1 pertence aos reais, = embora (-1)^.5 nao pertenca. Podemos, procedendo dessa forma, dizer por exemplo que cos(arccos 5) = =3D 5?=20 =20 A duvida surgiu quando eu estava lendo um texto na internet sobre a = resolucao da equacao completa do terceiro grau. Dada uma equacao do tipo = x^3 -3px + q =3D 0 (apos devidas simplificacoes de uma eq. completa = ax^3+bx^2+cx+d=3D0), o autor do texto sugeria a seguinte formula: x =3D 2 p^(1/2) cos ((1/3) arccos (q)p^(-3/2)) , deduzida a = partir do usual sistema {t + u=3Dq; tu=3Dp^3/3} do qual x=3Dt^1/3 + = u^1/3 e solucao de x^3 + px + q} para encontrar uma das raizes dessa = equacao (e consequentemente as outras duas). O que eu estranhei foi que = o texto dizia que essa era uma maneira de contornar o problema que a = maioria das calculadoras tem em lidar com operacoes do tipo = exponenciacao com numeros complexos (ja que a formula comum envolve = muitos radicais).=20 =20 O autor disse ter se baseado na identidade cos 3x =3D 4(cos x)^3 = - 3(cos x) para chegar a isso. =20 Abracos, Marcio ------=_NextPart_000_0011_01BE0661.7E8E4540 Content-Type: text/html; charset="iso-8859-1" Content-Transfer-Encoding: quoted-printable
    onde esta escrito = "x^3=20 -3px + q =3D 0" leia-se "x^3 -3px - 2q =3D = 0"
-----Original = Message-----
From:=20 Marcio <mcohen@iis.com.br>
To: = obm-rj@saci.mat.puc-rio.br= =20 <obm-rj@saci.mat.puc-rio.br= >
Date:=20 Segunda-feira, 2 de Novembro de 1998 12:32
Subject: = cos=20 (arccos 5) =3D 5?

    Gostaria de = fazer uma=20 pergunta que me surgiu ha pouco tempo atras:
 
Existe algum conjunto que esteja = para as=20 funcoes trigonometricas assim como os numeros complexos estao para a = "radiciacao" dos numeros reais? Nao sei se consegui = expressar bem=20 minha pergunta, mas o que eu quero saber e o seguinte:
podemos dizer que ((-1)^.5)^2 = =3D i^2 =3D 1=20 pertence aos reais, embora (-1)^.5 nao pertenca.
Podemos, procedendo dessa forma, = dizer por=20 exemplo que cos(arccos 5) =3D 5?
 
A duvida surgiu quando eu estava = lendo um=20 texto na internet sobre a resolucao da equacao completa do terceiro = grau.=20 Dada uma equacao do tipo x^3 -3px + q =3D 0 (apos devidas = simplificacoes de=20 uma eq. completa ax^3+bx^2+cx+d=3D0), o autor do texto sugeria a = seguinte=20 formula:
    x =3D 2 = p^(1/2) cos ((1/3)=20 arccos (q)p^(-3/2))  , deduzida a partir do usual sistema {t + = u=3Dq;=20 tu=3Dp^3/3} do qual x=3Dt^1/3 + u^1/3 e solucao de x^3 + px + q} = para encontrar=20 uma das raizes dessa equacao (e consequentemente as outras duas). O = que eu=20 estranhei foi que o texto dizia que = essa era uma=20 maneira de contornar o problema que a maioria das calculadoras tem = em lidar=20 com operacoes do tipo exponenciacao com numeros complexos (ja que a = formula=20 comum envolve muitos radicais).
 
    O autor disse = ter se=20 baseado na identidade cos 3x =3D 4(cos x)^3 - 3(cos x) para chegar a = isso.
 
    = Abracos,
   =20 Marcio
------=_NextPart_000_0011_01BE0661.7E8E4540-- From nicolau@saci.mat.puc-rio.br Mon Nov 9 16:11:16 1998 Received: from mula.mat.puc-rio.br by saci.mat.puc-rio.br (AIX 3.2/UCB 5.64/4.03) id AA23533; Mon, 9 Nov 1998 16:11:15 -0200 Received: (from nicolau@localhost) by mula.mat.puc-rio.br (8.8.7/8.8.7) id QAA17744; Mon, 9 Nov 1998 16:10:58 -0200 Date: Mon, 9 Nov 1998 16:10:58 -0200 From: Nicolau Corcao Saldanha Message-Id: <199811091810.QAA17744@mula.mat.puc-rio.br> To: elon@impa.br, jpcarneiro@openlink.com.br, pcezar@Euler.impa.br, wagner@impa.br Subject: obm-rj Cc: nicolau@saci.mat.puc-rio.br Status: RO X-Status: X-Keywords: X-UID: 26 Caros, Gostaria de lembrá-los que existe a lista obm-rj@mat.puc-rio.br, para discussão de problemas de olimpíadas de matemática. Como uma forma de convidá-los a assinarem a lista estou mandando para majordomo@mat.puc-rio.br pedidos de SUBSCRIBE em nome de voces. Assim, voces devem receber do majordomo uma mensagem (gerada automaticamente) pedindo confirmação do SUBSCRIBE. Se preferirem ficar fora da lista, por favor ignorem a mensagem. Se estiverem interessados em entrar, respondam seguindo as instruções do majordomo. A lista ainda tem um número de participantes extremamente baixo e o número de mensagens também ainda é baixo. []s, N. From kpm@netmarket.com.br Mon Nov 9 19:09:21 1998 Received: from diamante.netmarket.com.br by saci.mat.puc-rio.br (AIX 3.2/UCB 5.64/4.03) id AA17110; Mon, 9 Nov 1998 19:07:44 -0200 Received: from default (tty113 [200.248.242.113]) by diamante.netmarket.com.br (8.8.8+Sun/8.8.8) with ESMTP id TAA05476 for ; Mon, 9 Nov 1998 19:08:31 -0200 (EDT) Message-Id: <199811092108.TAA05476@diamante.netmarket.com.br> Reply-To: <@netmarket.com.br> From: "Karin" To: Date: Thu, 10 Sep 1998 18:49:28 -0300 X-Msmail-Priority: Normal X-Priority: 3 X-Mailer: Microsoft Internet Mail 4.70.1161 Mime-Version: 1.0 Content-Type: text/plain; charset=ISO-8859-1 Content-Transfer-Encoding: quoted-printable X-Mime-Autoconverted: from 8bit to quoted-printable by diamante.netmarket.com.br id TAA05476 Status: RO X-Status: X-Keywords: X-UID: 27 I Olimp=EDada LPM de Matem=E1tica Brasil, 1998. Tempo: 4 horas e meia. Primeiro Dia. Problema 1 Num tri=E2ngulo acut=E2ngulo ABC, AB mede x2 e AC mede x3. Considere o po= nto M como m=E9dio do lado AC. Se AB =E9 congruente a MC, quanto vale o lado BC= ? =20 Problema 2 Seja x um n=FAmero natural que satisfaz =E0 inequa=E7=E3o = =20 __ =D6 xx =A3 x3 =A3 x2 + x + x + x + x + ... + x/1998. =20 `2 `4 `6 =20 (1) Quais s=E3o os poss=EDveis valores de x? =20 (2) Demonstre que 1998 =E9 um poss=EDvel valor de x. Problema 3 Uma caixa trancada possui cinco orif=EDcios circulares. Disp=F5e-se de ci= nco discos tamb=E9m circulares, cada um de uma cor diferente, que devem ser introduzidos nos orif=EDcios, um disco em cada orif=EDcio. Se os colocarm= os numa seq=FC=EAncia secreta, a caixa se abrir=E1, sendo que ao colocarmos = um disco em seu respectivo orif=EDcio, um mecanismo da caixa =E9 acionado; ao colo= carmos dois discos em seus respectivos orif=EDcios, dois mecanismos s=E3o aciona= dos e assim por diante, de forma que se todos os discos forem colocados na seq=FC=EAncia secreta, todos os cinco mecanismos ser=E3o acionados e a ca= ixa automaticamente se abrir=E1. Veja abaixo algumas poss=EDveis seq=FC=EAnci= as dos discos e o n=FAmero de mecanismos que elas acionam. Seq=FC=EAncia dos discos: N=FAmero de mec= anismos=20 = =20 acionados: =20 Amarelo, Verde, Azul, Vermelho e Laranja. 0 Laranja, Vermelho, Azul, Verde e Amarelo. 1 Azul, Amarelo, Laranja, Vermelho e Verde. 2 Vermelho, Laranja, Amarelo, Verde e Azul. 3 (1) Considere o disco amarelo como A, o verde como B, o azul como C, o vermelho como D e o laranja como E. Fa=E7a uma lista bem organizada de to= das as poss=EDveis seq=FC=EAncias desses cinco discos e diga quantas s=E3o el= as.=20 (a) Observe a primeira seq=FC=EAncia mostrada. Podemos compar=E1-la com t= odas as outras que devem estar citadas na lista do item (1), e a partir desta compara=E7=E3o, podemos eliminar algumas delas, ou seja, podemos conhecer algumas seq=FC=EAncias (relacionadas na lista pedida) que, de acordo com = a primeira seq=FC=EAncia mostrada, n=E3o podem ser a correta para abrir a c= aixa. Mostre o n=FAmero exato de seq=FC=EAncias que podem ser eliminadas se com= paradas com a primeira mostrada e relacione numa nova lista todas as que restam. (b) Agora, considere somente as seq=FC=EAncias que devem constar na lista pedida no item anterior. Observe a segunda seq=FC=EAncia mostrada e diga = o n=FAmero exato das que s=E3o eliminadas quando comparadas a ela, fazendo = uma nova rela=E7=E3o que conste somente as restantes.=20 (c) Repita o processo, comparando a terceira seq=FC=EAncia mostrada com a= s que devem estar contidas na rela=E7=E3o do item anterior.=20 (2) Terminadas todas as listas com as respectivas informa=E7=F5es pedidas= , ap=F3s ter eliminado o maior n=FAmero poss=EDvel de seq=FC=EAncias, observe a =FA= ltima seq=FC=EAncia mostrada. Compare-a com a sua =FAltima rela=E7=E3o, tente e= liminar mais seq=FC=EAncias (se isto for poss=EDvel) e responda: Quantas seq=FC=EAncia= s ainda podem ser a correta para abrir a caixa? Se voc=EA achou mais do que uma, cite-as. Se voc=EA encontrou apenas uma, mostre-a. Se voc=EA n=E3o encont= rou nenhuma seq=FC=EAncia que poderia ser a verdadeira, ent=E3o diga que a ca= ixa jamais poder=E1 ser aberta! ____________________ Valor de cada problema:10 pontos I Olimp=EDada LPM de Matem=E1tica Brasil, 1998. Tempo: 4 horas e meia. Segundo Dia. Problema 4 A + B + C =3D D, AB =3D D e C2 =3D D. A + B + C + D =3D X, X + Y =3D Z e = Z =A3 1998. Se determinado que Y =3D 2, prove que _________ Z =3D Y - BY - CY + DY + Y=D6A + B + C + AY + BY + CY =BE=BE=BE=BE=BE=BE A Obs.: Todas as inc=F3gnitas s=E3o n=FAmeros naturais n=E3o-nulos, diferen= tes entre si. Problema 5 Seja LMN um tri=E2ngulo is=F3sceles interior ao ret=E2ngulo ABCD. Conside= re o conjunto de pontos E, F, K, P, Q e S como os pontos m=E9dios de AL, LB, N= D, AC, BD e CM, respectivamente, e o ponto T, como o p=E9 da altura de L.. C= D mede 222 cm, AL mede 155 cm e TN mede 44 cm. Explique por que a =E1rea de= um poss=EDvel tri=E2ngulo a ser tra=E7ado no interior ao ret=E2ngulo ABCD (m= ostre-o) =E9 1998 se e somente se a altura deste tri=E2ngulo for um natural divisor po= r 1998. Problema 6 O matem=E1tico Mathwell elaborou um jogo para seus colegas de estudo. Inicialmente, ele exibiu um cartaz contendo as seguintes igualdades: __=20 x =3D =D6 z x =3D a + b + c x + y =3D z c + z =3D z + x - 1 b + a =3D b c =3D a + b + 1 Depois, ele explicou os procedimentos. Durante 1998 dias, cada matem=E1ti= co jogaria diariamente 98 vezes um dado de seis faces, sendo elas a, b, c, x= , y, z, todas presentes no cartaz mostrado. Cada inc=F3gnita representava u= m n=FAmero natural, que correspondia com o seu valor em pontos. Assim, as 9= 8 jogadas di=E1rias do dado deveriam ser anotadas e somadas, resultando num total de pontos a cada dia. O processo continuaria at=E9 o 1998=B0 dia, q= uando seria declarado o vencedor, aquele que teria feito mais pontos ao final d= o tempo especificado.=20 (1) Mostre o valor de cada uma das seis faces do dado. (2) Se um competidor, durante todos os dias do jogo, terminasse sempre co= m um valor exatamente m=E9dio de pontos, qual seria a sua pontua=E7=E3o ao = t=E9rmino do jogo? E se o dado utilizado pelos matem=E1ticos fosse comum, de faces 1,2,3,4,5 e 6, qual seria esta pontua=E7=E3o? ____________________ Valor de cada problema: 10 pontos. I Olimp=EDada LPM de Matem=E1tica Brasil, 1998. Tempo: 4 horas e meia. Terceiro Dia. Problema 7 Um tabuleiro ABCD =E9 dividido em 14 x 14 quadrados unit=E1rios. Uma fich= a =E9 colocada num dos v=E9rtices do tabuleiro e dever=E1 chegar at=E9 o v=E9rt= ice diagonal, atrav=E9s de movimentos iguais aos da pe=E7a "Cavalo" do jogo d= e xadrez. Prove que para a ficha chegar ao v=E9rtice oposto, s=E3o necess=E1= rios no m=EDnimo 10 movimentos. Problema 8 Utilizando os algarismos de 0 a 9 exatamente uma vez, construa dois n=FAm= eros (para cada item) tal que: (1) A diferen=E7a entre eles (o maior menos o menor) seja a menor poss=ED= vel. (2) A diferen=E7a entre eles (o maior menos o menor) seja a maior poss=ED= vel. (3) A soma deles seja a menor poss=EDvel. (4) A soma deles seja a maior poss=EDvel. (5) O produto deles seja o menor poss=EDvel. (6) O produto deles seja o maior poss=EDvel. Problema 9 Considere uma hex=E1gono de =E2ngulos congruentes. Tra=E7amos retas na fi= gura, sempre ligando um v=E9rtice a tr=EAs outros n=E3o adjacentes, de forma qu= e de cada v=E9rtice saem tr=EAs retas. Mostre 96 tri=E2ngulos que podem ser tr= a=E7ados interiormente ao hex=E1gono a partir dos v=E9rtices deste e dos pontos de intersec=E7=E3o das retas. ____________________ Valor de cada problema: 10 pontos. Meu nome =E9 Lucas Povarczuk Mocelim. Tenho 14 anos e amo a ci=EAncia dos n=FAmeros. Por esta raz=E3o, j=E1 criei dezenas de problemas, sendo que 9= deles constam na minha pr=F3pria olimp=EDada de matem=E1tica, a I Olimp=EDada L= PM de Matem=E1tica. Gostaria que o senhor resolvesse a prova, dando a sua opini= =E3o (fa=E7a cr=EDticas, d=EA sugest=F5es, revele a que gostou mais, o porqu=EA= , etc.). Mando esta mensagem porque teclei com a simp=E1tica Nelly, da OBM, e ela sugeriu que eu enviasse os problemas ao professor Nicolau. Moro em Porto Alegre, RS, e meu e-mail =E9 kpm@netmarket.com.br Prazer! =20 =20 From kpm@netmarket.com.br Mon Nov 9 20:38:43 1998 Received: from diamante.netmarket.com.br by saci.mat.puc-rio.br (AIX 3.2/UCB 5.64/4.03) id AA19444; Mon, 9 Nov 1998 20:37:45 -0200 Received: from default (tty129 [200.248.242.129]) by diamante.netmarket.com.br (8.8.8+Sun/8.8.8) with ESMTP id UAA07110 for ; Mon, 9 Nov 1998 20:38:51 -0200 (EDT) Message-Id: <199811092238.UAA07110@diamante.netmarket.com.br> Reply-To: <@netmarket.com.br> From: "Karin" To: Subject: Hello, Mr. Nicolau! Date: Thu, 10 Sep 1998 20:21:46 -0300 X-Msmail-Priority: Normal X-Priority: 3 X-Mailer: Microsoft Internet Mail 4.70.1161 Mime-Version: 1.0 Content-Type: text/plain; charset=ISO-8859-1 Content-Transfer-Encoding: quoted-printable X-Mime-Autoconverted: from 8bit to quoted-printable by diamante.netmarket.com.br id UAA07110 Status: RO X-Status: X-Keywords: X-UID: 28 Prezado Professor Nicolau, lembra de mim? Sou o Lucas, aquele que cria problemas matem=E1ticos. Enviei a pouco tempo a minha olimp=EDada, para que o senho= r a resolvesse e fizesse coment=E1rios. Escrevo esta mensagem com o intuito d= e conhec=EA-lo melhor. J=E1 leu O Homem que Calculava, de Malba Tahan? E o Racioc=EDnio L=F3gico, de Jonofon S=E9rates? Ensina h=E1 quanto tempo? On= de? Tem rela=E7=E3o com a OBM? Tem problemas para me enviar? Se quiser falar comi= go, perguntar alguma coisa a respeito dos meus problemas, encontrar erros, escreva para mim.=20 kpm@netmarket.com.br =20 From kpm@netmarket.com.br Mon Nov 9 19:09:21 1998 Received: from diamante.netmarket.com.br by saci.mat.puc-rio.br (AIX 3.2/UCB 5.64/4.03) id AA17110; Mon, 9 Nov 1998 19:07:44 -0200 Received: from default (tty113 [200.248.242.113]) by diamante.netmarket.com.br (8.8.8+Sun/8.8.8) with ESMTP id TAA05476 for ; Mon, 9 Nov 1998 19:08:31 -0200 (EDT) Message-Id: <199811092108.TAA05476@diamante.netmarket.com.br> Reply-To: <@netmarket.com.br> From: "Karin" To: Date: Thu, 10 Sep 1998 18:49:28 -0300 X-Msmail-Priority: Normal X-Priority: 3 X-Mailer: Microsoft Internet Mail 4.70.1161 Mime-Version: 1.0 Content-Type: text/plain; charset=ISO-8859-1 Content-Transfer-Encoding: quoted-printable X-Mime-Autoconverted: from 8bit to quoted-printable by diamante.netmarket.com.br id TAA05476 Status: RO X-Status: X-Keywords: X-UID: 29 I Olimp=EDada LPM de Matem=E1tica Brasil, 1998. Tempo: 4 horas e meia. Primeiro Dia. Problema 1 Num tri=E2ngulo acut=E2ngulo ABC, AB mede x2 e AC mede x3. Considere o po= nto M como m=E9dio do lado AC. Se AB =E9 congruente a MC, quanto vale o lado BC= ? =20 Problema 2 Seja x um n=FAmero natural que satisfaz =E0 inequa=E7=E3o = =20 __ =D6 xx =A3 x3 =A3 x2 + x + x + x + x + ... + x/1998. =20 `2 `4 `6 =20 (1) Quais s=E3o os poss=EDveis valores de x? =20 (2) Demonstre que 1998 =E9 um poss=EDvel valor de x. Problema 3 Uma caixa trancada possui cinco orif=EDcios circulares. Disp=F5e-se de ci= nco discos tamb=E9m circulares, cada um de uma cor diferente, que devem ser introduzidos nos orif=EDcios, um disco em cada orif=EDcio. Se os colocarm= os numa seq=FC=EAncia secreta, a caixa se abrir=E1, sendo que ao colocarmos = um disco em seu respectivo orif=EDcio, um mecanismo da caixa =E9 acionado; ao colo= carmos dois discos em seus respectivos orif=EDcios, dois mecanismos s=E3o aciona= dos e assim por diante, de forma que se todos os discos forem colocados na seq=FC=EAncia secreta, todos os cinco mecanismos ser=E3o acionados e a ca= ixa automaticamente se abrir=E1. Veja abaixo algumas poss=EDveis seq=FC=EAnci= as dos discos e o n=FAmero de mecanismos que elas acionam. Seq=FC=EAncia dos discos: N=FAmero de mec= anismos=20 = =20 acionados: =20 Amarelo, Verde, Azul, Vermelho e Laranja. 0 Laranja, Vermelho, Azul, Verde e Amarelo. 1 Azul, Amarelo, Laranja, Vermelho e Verde. 2 Vermelho, Laranja, Amarelo, Verde e Azul. 3 (1) Considere o disco amarelo como A, o verde como B, o azul como C, o vermelho como D e o laranja como E. Fa=E7a uma lista bem organizada de to= das as poss=EDveis seq=FC=EAncias desses cinco discos e diga quantas s=E3o el= as.=20 (a) Observe a primeira seq=FC=EAncia mostrada. Podemos compar=E1-la com t= odas as outras que devem estar citadas na lista do item (1), e a partir desta compara=E7=E3o, podemos eliminar algumas delas, ou seja, podemos conhecer algumas seq=FC=EAncias (relacionadas na lista pedida) que, de acordo com = a primeira seq=FC=EAncia mostrada, n=E3o podem ser a correta para abrir a c= aixa. Mostre o n=FAmero exato de seq=FC=EAncias que podem ser eliminadas se com= paradas com a primeira mostrada e relacione numa nova lista todas as que restam. (b) Agora, considere somente as seq=FC=EAncias que devem constar na lista pedida no item anterior. Observe a segunda seq=FC=EAncia mostrada e diga = o n=FAmero exato das que s=E3o eliminadas quando comparadas a ela, fazendo = uma nova rela=E7=E3o que conste somente as restantes.=20 (c) Repita o processo, comparando a terceira seq=FC=EAncia mostrada com a= s que devem estar contidas na rela=E7=E3o do item anterior.=20 (2) Terminadas todas as listas com as respectivas informa=E7=F5es pedidas= , ap=F3s ter eliminado o maior n=FAmero poss=EDvel de seq=FC=EAncias, observe a =FA= ltima seq=FC=EAncia mostrada. Compare-a com a sua =FAltima rela=E7=E3o, tente e= liminar mais seq=FC=EAncias (se isto for poss=EDvel) e responda: Quantas seq=FC=EAncia= s ainda podem ser a correta para abrir a caixa? Se voc=EA achou mais do que uma, cite-as. Se voc=EA encontrou apenas uma, mostre-a. Se voc=EA n=E3o encont= rou nenhuma seq=FC=EAncia que poderia ser a verdadeira, ent=E3o diga que a ca= ixa jamais poder=E1 ser aberta! ____________________ Valor de cada problema:10 pontos I Olimp=EDada LPM de Matem=E1tica Brasil, 1998. Tempo: 4 horas e meia. Segundo Dia. Problema 4 A + B + C =3D D, AB =3D D e C2 =3D D. A + B + C + D =3D X, X + Y =3D Z e = Z =A3 1998. Se determinado que Y =3D 2, prove que _________ Z =3D Y - BY - CY + DY + Y=D6A + B + C + AY + BY + CY =BE=BE=BE=BE=BE=BE A Obs.: Todas as inc=F3gnitas s=E3o n=FAmeros naturais n=E3o-nulos, diferen= tes entre si. Problema 5 Seja LMN um tri=E2ngulo is=F3sceles interior ao ret=E2ngulo ABCD. Conside= re o conjunto de pontos E, F, K, P, Q e S como os pontos m=E9dios de AL, LB, N= D, AC, BD e CM, respectivamente, e o ponto T, como o p=E9 da altura de L.. C= D mede 222 cm, AL mede 155 cm e TN mede 44 cm. Explique por que a =E1rea de= um poss=EDvel tri=E2ngulo a ser tra=E7ado no interior ao ret=E2ngulo ABCD (m= ostre-o) =E9 1998 se e somente se a altura deste tri=E2ngulo for um natural divisor po= r 1998. Problema 6 O matem=E1tico Mathwell elaborou um jogo para seus colegas de estudo. Inicialmente, ele exibiu um cartaz contendo as seguintes igualdades: __=20 x =3D =D6 z x =3D a + b + c x + y =3D z c + z =3D z + x - 1 b + a =3D b c =3D a + b + 1 Depois, ele explicou os procedimentos. Durante 1998 dias, cada matem=E1ti= co jogaria diariamente 98 vezes um dado de seis faces, sendo elas a, b, c, x= , y, z, todas presentes no cartaz mostrado. Cada inc=F3gnita representava u= m n=FAmero natural, que correspondia com o seu valor em pontos. Assim, as 9= 8 jogadas di=E1rias do dado deveriam ser anotadas e somadas, resultando num total de pontos a cada dia. O processo continuaria at=E9 o 1998=B0 dia, q= uando seria declarado o vencedor, aquele que teria feito mais pontos ao final d= o tempo especificado.=20 (1) Mostre o valor de cada uma das seis faces do dado. (2) Se um competidor, durante todos os dias do jogo, terminasse sempre co= m um valor exatamente m=E9dio de pontos, qual seria a sua pontua=E7=E3o ao = t=E9rmino do jogo? E se o dado utilizado pelos matem=E1ticos fosse comum, de faces 1,2,3,4,5 e 6, qual seria esta pontua=E7=E3o? ____________________ Valor de cada problema: 10 pontos. I Olimp=EDada LPM de Matem=E1tica Brasil, 1998. Tempo: 4 horas e meia. Terceiro Dia. Problema 7 Um tabuleiro ABCD =E9 dividido em 14 x 14 quadrados unit=E1rios. Uma fich= a =E9 colocada num dos v=E9rtices do tabuleiro e dever=E1 chegar at=E9 o v=E9rt= ice diagonal, atrav=E9s de movimentos iguais aos da pe=E7a "Cavalo" do jogo d= e xadrez. Prove que para a ficha chegar ao v=E9rtice oposto, s=E3o necess=E1= rios no m=EDnimo 10 movimentos. Problema 8 Utilizando os algarismos de 0 a 9 exatamente uma vez, construa dois n=FAm= eros (para cada item) tal que: (1) A diferen=E7a entre eles (o maior menos o menor) seja a menor poss=ED= vel. (2) A diferen=E7a entre eles (o maior menos o menor) seja a maior poss=ED= vel. (3) A soma deles seja a menor poss=EDvel. (4) A soma deles seja a maior poss=EDvel. (5) O produto deles seja o menor poss=EDvel. (6) O produto deles seja o maior poss=EDvel. Problema 9 Considere uma hex=E1gono de =E2ngulos congruentes. Tra=E7amos retas na fi= gura, sempre ligando um v=E9rtice a tr=EAs outros n=E3o adjacentes, de forma qu= e de cada v=E9rtice saem tr=EAs retas. Mostre 96 tri=E2ngulos que podem ser tr= a=E7ados interiormente ao hex=E1gono a partir dos v=E9rtices deste e dos pontos de intersec=E7=E3o das retas. ____________________ Valor de cada problema: 10 pontos. Meu nome =E9 Lucas Povarczuk Mocelim. Tenho 14 anos e amo a ci=EAncia dos n=FAmeros. Por esta raz=E3o, j=E1 criei dezenas de problemas, sendo que 9= deles constam na minha pr=F3pria olimp=EDada de matem=E1tica, a I Olimp=EDada L= PM de Matem=E1tica. Gostaria que o senhor resolvesse a prova, dando a sua opini= =E3o (fa=E7a cr=EDticas, d=EA sugest=F5es, revele a que gostou mais, o porqu=EA= , etc.). Mando esta mensagem porque teclei com a simp=E1tica Nelly, da OBM, e ela sugeriu que eu enviasse os problemas ao professor Nicolau. Moro em Porto Alegre, RS, e meu e-mail =E9 kpm@netmarket.com.br Prazer! =20 =20 From owner-obm-rj@saci.mat.puc-rio.br Mon Nov 9 20:45:18 1998 Received: by saci.mat.puc-rio.br (AIX 3.2/UCB 5.64/4.03) id AA20241; Mon, 9 Nov 1998 20:45:17 -0200 Received: from mail.iis.com.br by saci.mat.puc-rio.br (AIX 3.2/UCB 5.64/4.03) id AA04109; Mon, 9 Nov 1998 20:45:01 -0200 Received: from Rffsa.rffsa.gov.br (rio-as4-tty06.iis.com.br [200.202.97.70]) by mail.iis.com.br (8.8.7/8.8.14) with SMTP id UAA30090 for ; Mon, 9 Nov 1998 20:44:56 -0200 From: "Marcio" To: Subject: O problema dos cientistas Date: Mon, 9 Nov 1998 20:50:51 -0300 Message-Id: <01be0c3b$dd8d95c0$4661cac8@Rffsa.rffsa.gov.br> Mime-Version: 1.0 Content-Type: multipart/alternative; boundary="----=_NextPart_000_0016_01BE0C22.B8405DC0" X-Priority: 3 X-Msmail-Priority: Normal X-Mailer: Microsoft Outlook Express 4.71.1712.3 X-Mimeole: Produced By Microsoft MimeOLE V4.71.1712.3 Sender: owner-obm-rj@saci.mat.puc-rio.br Precedence: bulk Reply-To: obm-rj@saci.mat.puc-rio.br Status: RO X-Status: X-Keywords: X-UID: 30 This is a multi-part message in MIME format. ------=_NextPart_000_0016_01BE0C22.B8405DC0 Content-Type: text/plain; charset="iso-8859-1" Content-Transfer-Encoding: quoted-printable Aqui esta o problema que eu mencionei na aula de domingo tirado do = livro de combinatoria da colecao do professor de matematica: Nove cientistas trabalham num projeto sigiloso. Por questoes de = seguranca, os planos sao guardados em um cofre protegiso por muitos = cadeados de modo que so e possivel abri-los todos se houve pelo menos 5 = cientistas presentes. a)Qual o numero minimo possivel de cadeados b)Na situacao do item a), quantas chaves cada cientista deve ter? Esse problema eh semelhante ao que caiu na prova da olimpiada estadual = (o da olimp. porem era mais simples e podia ser resolvido por tentativa = e erro). ------=_NextPart_000_0016_01BE0C22.B8405DC0 Content-Type: text/html; charset="iso-8859-1" Content-Transfer-Encoding: quoted-printable
    Aqui esta=20 o problema que eu mencionei na aula de domingo tirado do livro de = combinatoria=20 da colecao do professor de matematica:
 
Nove = cientistas trabalham=20 num projeto sigiloso. Por questoes de seguranca, os planos sao guardados = em um=20 cofre protegiso por muitos cadeados de modo que so e possivel abri-los = todos se=20 houve pelo menos 5 cientistas presentes.
    a)Qual o=20 numero minimo possivel de cadeados
    b)Na=20 situacao do item a), quantas chaves cada cientista deve = ter?
 
 
Esse problema = eh semelhante=20 ao que caiu na prova da olimpiada estadual (o da olimp. porem era mais = simples e=20 podia ser resolvido por tentativa e = erro).
------=_NextPart_000_0016_01BE0C22.B8405DC0-- From owner-obm-rj@saci.mat.puc-rio.br Tue Nov 10 18:30:45 1998 Received: by saci.mat.puc-rio.br (AIX 3.2/UCB 5.64/4.03) id AA34211; Tue, 10 Nov 1998 18:30:44 -0200 Received: from mula.mat.puc-rio.br by saci.mat.puc-rio.br (AIX 3.2/UCB 5.64/4.03) id AA21405; Tue, 10 Nov 1998 18:30:29 -0200 Received: (from nicolau@localhost) by mula.mat.puc-rio.br (8.8.7/8.8.7) id SAA18664 for obm-rj@saci.mat.puc-rio.br; Tue, 10 Nov 1998 18:30:29 -0200 Date: Tue, 10 Nov 1998 18:30:29 -0200 From: Nicolau Corcao Saldanha Message-Id: <199811102030.SAA18664@mula.mat.puc-rio.br> To: obm-rj@saci.mat.puc-rio.br Subject: Re: O problema dos cientistas Sender: owner-obm-rj@saci.mat.puc-rio.br Precedence: bulk Reply-To: obm-rj@saci.mat.puc-rio.br Status: RO X-Status: X-Keywords: X-UID: 31 > Nove cientistas trabalham num projeto sigiloso. Por questoes de = > seguranca, os planos sao guardados em um cofre protegiso por muitos = > cadeados de modo que so e possivel abri-los todos se houve pelo menos 5 = > cientistas presentes. > a)Qual o numero minimo possivel de cadeados > b)Na situacao do item a), quantas chaves cada cientista deve ter? Segue a solução do problema. Se você preferir pensar sozinho, pare de ler aqui. []s, N. Interpreto o enunciado como dizendo que há vários cadeados, cada um tem uma chave diferente (com muitas cópias), os cadeados são independentes uns dos outros e o cofre só é aberto se todos os cadeados forem abertos. Assim, dados quatro cientistas quaisquer, sempre existe pelo menos um cadeado para o qual nenhum dos quatro tem a chave. Por outro lado, qualquer um dos cinco cientistas restantes deve ter a chave de todos estes cadeados. Assim, ter mais de um cadeado deste tipo é um desperdício (as mesmas pessoas teriam as chaves) e podemos supor que para quatro cientistas quaisquer há exatamente um cadeado tal que nenhum deles tem a chave. Também, para conjuntos de quatro cientistas distintos os cadeados devem ser diferentes, já que os outros cinco têm a chave. Assim, temos pelo menos um cadeado para cada conjunto de quatro cientistas (que não têm a chave) ou, equivalentemente, um cadeado para cada conjunto de cinco cientistas (que eles e só eles têm a chave). Ou seja, temos pelo menos binomial(9,4) = 126 cadeados. Mas esta configuração além de necessária é também suficiente. Ou seja, a resposta do item (a) é 126. Para o item (b), note que há 5 cópias de cada chave, igualmente divididas (no total) entre 9 cientistas. Assim, cada cientista tem 5*126/9 = 70 chaves. Mais geralmente, se o número total de cientistas for n e o número exigido para abrir o cofre for m o número de cadeados será binomial(n,m-1) e o número de chaves em poder de cada cientista (n-m+1)*binomial(n,m-1)/n = binomial(n-1,m-1). From owner-obm-rj@saci.mat.puc-rio.br Tue Nov 17 17:03:27 1998 Received: by saci.mat.puc-rio.br (AIX 3.2/UCB 5.64/4.03) id AA24792; Tue, 17 Nov 1998 17:03:15 -0200 Received: from euler.impa.br by saci.mat.puc-rio.br (AIX 3.2/UCB 5.64/4.03) id AA14801; Tue, 17 Nov 1998 17:02:59 -0200 Received: from Gauss.impa.br (gauss.impa.br [147.65.4.1]) by Euler.impa.br (8.8.6/8.8.6) with ESMTP id RAA16791 for ; Tue, 17 Nov 1998 17:02:59 -0200 (EDT) From: Carlos Gustavo Tamm de Araujo Moreira Received: by Gauss.impa.br (8.8.8) id RAA03597; Tue, 17 Nov 1998 17:02:58 -0200 (EDT) Message-Id: <199811171902.RAA03597@Gauss.impa.br> Subject: Re: =?iso-8859-1?Q?Sobre_a_terceira_fase_da_olimp=EDada.?= To: obm-rj@saci.mat.puc-rio.br Date: Tue, 17 Nov 1998 17:02:58 -0200 (EDT) In-Reply-To: <01be1152$152fd7c0$7861cac8@Rffsa.rffsa.gov.br> from "Marcio" at Nov 16, 98 08:13:05 am X-Mailer: ELM [version 2.4 PL25] Mime-Version: 1.0 Content-Type: text/plain; charset=US-ASCII Content-Transfer-Encoding: 7bit Sender: owner-obm-rj@saci.mat.puc-rio.br Precedence: bulk Reply-To: obm-rj@saci.mat.puc-rio.br Status: RO X-Status: X-Keywords: X-UID: 32 Caro Marcio, Vamos colocar em breve todas as solucoes na home-page.Sobre o problema 6,que e' baseado em fatos reais(a situacao ocorreu comigo e com o Nicolau, quando estavamos em Berlim no congresso internacional de matematica,em agosto),a ideia e' que os dois andassem sempre juntos,e a resposta correta e' 9.Uma estrategia possivel e' andarem 1 quarteirao para a direita,voltarem a origem e andarem 2 pra esquerda,voltarem pra origem e andarem 4 pra esquerda,etc.Nao tinhamos pensado na hipotese de eles se separarem,mas vamos estudar caso a caso solucoes coerentes com essa interpretacao. Abracos, Carlos Gustavo Moreira ( Gugu) From nicolau@saci.mat.puc-rio.br Tue Nov 17 18:20:59 1998 Received: from boto.mat.puc-rio.br by saci.mat.puc-rio.br (AIX 3.2/UCB 5.64/4.03) id AA14646; Tue, 17 Nov 1998 18:20:59 -0200 Received: from localhost (nicolau@localhost) by boto.mat.puc-rio.br (8.8.7/8.8.7) with SMTP id SAA21144; Tue, 17 Nov 1998 18:20:57 -0200 Date: Tue, 17 Nov 1998 18:20:57 -0200 (EDT) From: Nicolau Corcao Saldanha To: Olimpiada Brasileira de Matematica Cc: nicolau@saci.mat.puc-rio.br Subject: Re: Palestra Domingo In-Reply-To: <3.0.5.32.19981117145124.007ac300@pop.impa.br> Message-Id: Mime-Version: 1.0 Content-Type: TEXT/PLAIN; charset=iso-8859-1 Content-Transfer-Encoding: QUOTED-PRINTABLE Status: RO X-Status: X-Keywords: X-UID: 33 > =09=09Tem prevista alguma palestra para este Domingo?, o Edmilson vem par= a > ajudar a corrigir as provas, Gugu diz que Edmilsom eventualmente poderia > dar a palestra no Domingo, Que voce acha disso??? A sala est=E1 reservada e dispon=EDvel. Acho =F3tima a id=E9ia de convidarm= os o Edmilson para falar. Vou mandar mensagem neste sentido para obm-rj. []s, N. From owner-obm-rj@saci.mat.puc-rio.br Tue Nov 17 18:24:16 1998 Received: by saci.mat.puc-rio.br (AIX 3.2/UCB 5.64/4.03) id AA23111; Tue, 17 Nov 1998 18:23:57 -0200 Received: from boto.mat.puc-rio.br by saci.mat.puc-rio.br (AIX 3.2/UCB 5.64/4.03) id AA14659; Tue, 17 Nov 1998 18:23:43 -0200 Received: from localhost (nicolau@localhost) by boto.mat.puc-rio.br (8.8.7/8.8.7) with SMTP id SAA21181 for ; Tue, 17 Nov 1998 18:23:43 -0200 Date: Tue, 17 Nov 1998 18:23:43 -0200 (EDT) From: Nicolau Corcao Saldanha To: obm-rj@saci.mat.puc-rio.br Message-Id: Mime-Version: 1.0 Content-Type: TEXT/PLAIN; charset=iso-8859-1 Content-Transfer-Encoding: QUOTED-PRINTABLE Sender: owner-obm-rj@saci.mat.puc-rio.br Precedence: bulk Reply-To: obm-rj@saci.mat.puc-rio.br Status: RO X-Status: X-Keywords: X-UID: 34 Caros, Deve haver palestra este domingo, no lugar e hora de sempre: 14 =E0s 18, na sala 150L da PUC-Rio. Deve falar o professor Edmilson, do Col=E9gio Etapa, em S=E3o Paulo. Nicolau P.S.: Escrevam mais, mandem mais problemas, esta lista est=E1 muito parada. From owner-obm-rj@saci.mat.puc-rio.br Mon Nov 16 08:05:45 1998 Received: by saci.mat.puc-rio.br (AIX 3.2/UCB 5.64/4.03) id AA14638; Mon, 16 Nov 1998 08:05:44 -0200 Received: from mail.iis.com.br by saci.mat.puc-rio.br (AIX 3.2/UCB 5.64/4.03) id AA13354; Mon, 16 Nov 1998 08:05:28 -0200 Received: from Rffsa.rffsa.gov.br (rio-as7-tty08.iis.com.br [200.202.97.120]) by mail.iis.com.br (8.8.7/8.8.14) with SMTP id IAA01525; Mon, 16 Nov 1998 08:05:26 -0200 From: "Marcio" To: "Nicolau Corcao Saldanha" Cc: Subject: =?iso-8859-1?Q?Sobre_a_terceira_fase_da_olimp=EDada.?= Date: Mon, 16 Nov 1998 08:13:05 -0300 Message-Id: <01be1152$152fd7c0$7861cac8@Rffsa.rffsa.gov.br> Mime-Version: 1.0 Content-Type: text/plain; charset="iso-8859-1" X-Priority: 3 X-Msmail-Priority: Normal X-Mailer: Microsoft Outlook Express 4.71.1712.3 X-Mimeole: Produced By Microsoft MimeOLE V4.71.1712.3 Content-Transfer-Encoding: quoted-printable X-Mime-Autoconverted: from 8bit to quoted-printable by mail.iis.com.br id IAA01525 Sender: owner-obm-rj@saci.mat.puc-rio.br Precedence: bulk Reply-To: obm-rj@saci.mat.puc-rio.br Status: RO X-Status: X-Keywords: X-UID: 35 Gostaria de saber como =E9 que voces (alunos) foram nas provas.. Mas = na verdade o motivo da minha mensagem n=E3o =E9 bem esta. Eu estou mandando = ela por causa do sexto problema da terceira fase da olimpiada. Alguem esta 100% seguro que interpretou ele da maneira correta e acertou? Ser=E1 que voc=EAs (Nicolau e Gugu) poderiam por favor reproduzi-lo n= uma mensagem e mostrar a solu=E7=E3o? J=E1 conversei com 3 pessoas desde que sai da prova. Cada uma pensou = um neg=F3cio diferente. Todos dao respostas nao muito complicadas de se cheg= ar, e que diferem apenas no item "interpretacao do enunciado". Aqui vao algumas= : k =3D 2N +1 (considerando que os dois matematicos, juntos, andam uma quad= ra para a direita, depois duas para a esquerda, depois tres para direita, et= c.) k =3D 3 (considerando que o matematico simplesmente anda N para a direita= , e caso a rua nao seja essa, anda mais 2N para a esquerda). K =3D 3 + 2/n (considerando que h=E1 uma pequena diferenca 2/n devido aos matematicos andarem a quadra que eles ja estao). Gostaria tambem, se possivel de obter a solucao (mesmo que bastante simplificada) das outras questoes. Mas a que mais ficou batendo foi a sex= ta mesmo! Obrigado, Marcio From comissao@impa.br Tue Nov 17 18:53:20 1998 Received: from euler.impa.br by saci.mat.puc-rio.br (AIX 3.2/UCB 5.64/4.03) id AA26282; Tue, 17 Nov 1998 18:53:15 -0200 Received: from Euler (localhost.impa.br [127.0.0.1]) by Euler.impa.br (8.8.6/8.8.6) with SMTP id SAA19200; Tue, 17 Nov 1998 18:47:29 -0200 (EDT) Date: Tue, 17 Nov 1998 18:47:29 -0200 (EDT) Message-Id: Errors-To: pcezar@visgraf.impa.br Reply-To: comissao@impa.br Originator: comissao@obm.org.br Sender: comissao@impa.br Precedence: bulk From: Nicolau Corcao Saldanha To: Multiple recipients of list X-Listprocessor-Version: 6.0c -- ListProcessor by Anastasios Kotsikonas X-Comment: Comissao de Olimpiadas da SBM Status: RO X-Status: X-Keywords: X-UID: 36 Caros, Atendendo a sugestões, estou mandando para todos vocês este anúncio de uma lista para discussão de problemas de matemática. A lista é aberta a todos, e seu público alvo são professores e alunos de fim de 1o ou do 2o grau. O endereço da lista é: obm-rj@mat.puc-rio.br Para assinar, enviem uma mensagem para majordomo@mat.puc-rio.br com texto subscribe obm-rj Quem assim proceder deve receber um pedido de confirmação auto-explicativo (automaticamente gerado pelo majordomo, em inglês). Em caso de problemas, escrevam para mim: nicolau@mat.puc-rio.br Nicolau From owner-obm-rj@saci.mat.puc-rio.br Tue Nov 17 19:06:02 1998 Received: by saci.mat.puc-rio.br (AIX 3.2/UCB 5.64/4.03) id AA17110; Tue, 17 Nov 1998 19:06:00 -0200 Received: from diamante.netmarket.com.br by saci.mat.puc-rio.br (AIX 3.2/UCB 5.64/4.03) id AA26066; Tue, 17 Nov 1998 19:05:24 -0200 Received: from default (tty119 [200.248.242.119]) by diamante.netmarket.com.br (8.8.8+Sun/8.8.8) with ESMTP id SAA26285 for ; Tue, 17 Nov 1998 18:54:51 -0200 (EDT) Message-Id: <199811172054.SAA26285@diamante.netmarket.com.br> From: "Karin" To: Subject: =?ISO-8859-1?Q?Problema_da_Olimp=EDada_Brasileira?= Date: Wed, 16 Sep 1998 08:13:38 -0300 X-Msmail-Priority: Normal X-Priority: 3 X-Mailer: Microsoft Internet Mail 4.70.1161 Mime-Version: 1.0 Content-Type: text/plain; charset=ISO-8859-1 Content-Transfer-Encoding: quoted-printable X-Mime-Autoconverted: from 8bit to quoted-printable by diamante.netmarket.com.br id SAA26285 Sender: owner-obm-rj@saci.mat.puc-rio.br Precedence: bulk Reply-To: obm-rj@saci.mat.puc-rio.br Status: RO X-Status: X-Keywords: X-UID: 37 Oi M=E1rcio! hoje eu tive uma prova de matem=E1tica no col=E9gio e como aind= a faltavam uns 20 minutos pra acabar o per=EDodo, resolvi fazer aquela ques= t=E3o da Olimp=EDada Brasileira de Matem=E1tica do m=EDnimo m=FAltiplo de 1998 = de algarismos 0 e 3 (segunda fase de n=EDvel 3) e consegui resolver o proble= ma, utilizando o verso da folha do testa da escola. Por isso, se quiser, n=E3= o precisa me mostrar o desenvolvimento como eu tinha pedido. Valeu pela aten=E7=E3o.=20 Lucas P.M.=20 =20 From owner-obm-rj@saci.mat.puc-rio.br Tue Nov 17 19:07:14 1998 Received: by saci.mat.puc-rio.br (AIX 3.2/UCB 5.64/4.03) id AA15330; Tue, 17 Nov 1998 19:07:13 -0200 Received: from diamante.netmarket.com.br by saci.mat.puc-rio.br (AIX 3.2/UCB 5.64/4.03) id AA23743; Tue, 17 Nov 1998 19:01:10 -0200 Received: from default (tty119 [200.248.242.119]) by diamante.netmarket.com.br (8.8.8+Sun/8.8.8) with ESMTP id TAA26464 for ; Tue, 17 Nov 1998 19:00:25 -0200 (EDT) Message-Id: <199811172100.TAA26464@diamante.netmarket.com.br> From: "Karin" To: Subject: =?ISO-8859-1?Q?Desafio_aos_seus_Neur=F4nios.?= Date: Wed, 16 Sep 1998 08:19:14 -0300 X-Msmail-Priority: Normal X-Priority: 3 X-Mailer: Microsoft Internet Mail 4.70.1161 Mime-Version: 1.0 Content-Type: text/plain; charset=ISO-8859-1 Content-Transfer-Encoding: quoted-printable X-Mime-Autoconverted: from 8bit to quoted-printable by diamante.netmarket.com.br id TAA26464 Sender: owner-obm-rj@saci.mat.puc-rio.br Precedence: bulk Reply-To: obm-rj@saci.mat.puc-rio.br Status: RO X-Status: X-Keywords: X-UID: 38 Oi gente, aqui =E9 o Lucas. Prometi mandar uma outra prova minha. =C9 = bem maior que a outra, por=E9m =E9 brincadeira de crian=E7a resolv=EA-la, poi= s eu criei os problemas para jovens de no m=E1ximo 14 anos fazerem. O verdadeiro des= afio ser=E1 acertar todas as quest=F5es respeitando o tempo. A=ED vai chumbo: Desafio aos seus Neur=F4nios - 1998 1a Fase Instru=E7=F5es: - A dura=E7=E3o da prova =E9 de 3 horas. - Voc=EA pode rascunhar no verso das folhas da prova. - Mandem um e-mail com as respostas desta e da outra prova! = =20 Bo@ sorte! 01. No planeta X existem 600.000 litros de =E1gua. A cada dia est=E3o sen= do utilizados 1/250 litros desse volume. Hoje h=E1 apenas 25/50 dos litros q= ue haviam anteriormente. Daqui a quanto tempo n=E3o ter=E1 mais =E1gua nesse planeta? (1 m=EAs equivale a 30 dias.) =20 A) Um bimestre e 25 dias. B) Tr=EAs meses e 10 dias. =20 C) Um trimestre e 5 dias. =20 D) Cinco meses e meio. E) Um semestre e 1 dia.=20 02. Leonardo nasceu numa ter=E7a-feira, em 11/11/1997. Em 1998 ele far=E1 anivers=E1rio numa quarta-feira, e em 1 999 numa quinta-feira. Sabendo qu= e 10 000 ser=E1 um ano bissexto ( possui 366 dias ) e que os anos bissextos ocorrem de 4 em 4 anos, o 45=B0 anivers=E1rio de Leonardo ser=E1 num (a):= =20 A) domingo B) segunda-feira C) ter=E7a-feira = D) sexta-feira E) s=E1bado =20 03. Rafael e sua esposa Rachel ganham 10.000 reais mensais, juntos. Se Rafael gasta =BC dessa quantia (por m=EAs) com o filho, Ricardo, e esse g= asto equivale a 2/5 de seu sal=E1rio, Rachel ganha, por m=EAs, em reais: A) 3.750 B) 5.750 C) 850 D) 1.000 = =20 E) 9.000=20 =20 04. Num =F3rg=E3o eletr=F4nico, existem as teclas d=F3, r=E9, mi, f=E1, s= ol, l=E1 e si, nesta ordem, sendo que esta seq=FC=EAncia se repete 5 vezes e no final h=E1= mais um d=F3 (total de 36 teclas). Numa m=FAsica chamada "O Frio da Montanha",= h=E1 uma seq=FC=EAncia de 13 teclas:=20 Sol, d=F3, si, d=F3, l=E1, r=E9, d=F3, r=E9, si, mi, r=E9, mi, x. A tecla= x =E9: A) sol B) l=E1 C) f=E1 D) mi = E) d=F3 =20 05. Num torneio de v=F4lei, h=E1 20 equipes. Se todas jogarem contra toda= s, haver=E3o: A) 1.034 jogos C) 190 jogos E) 400 jogos = =20 =20 B) 100 jogos D) 380 jogos 06. Na seq=FC=EAncia 172 143 214 285 356 427 498 569 631 070 117 712 841 = 391 149... o pr=F3ximo algarismo =E9: A) 5 B) 3 C) 7 D) 2 E) 8 07. Quantos s=E3o os n=FAmeros naturais n compreendidos entre 250 e 500 t= al que 7n - 5 resulta em um n=FAmero m=FAltiplo de 4? A) 83 B) 63 C) 43 D) 73 E) 53 08. Quantas vezes pode-se escrever uma seq=FC=EAncia de 4 d=EDgitos com a= s letras a, b, c, d, sem repetir letras? A) 24 vezes B) 16 vezes C) 36 vezes D) 4= 8 vezes E) 12 vezes =20 09. No jogo de bicho voc=EA pode " pingar " n=FAmeros, ou seja, apostar n= os n=FAmeros que voc=EA escolheu sendo que os algarismos podem estar em qual= quer ordem. Ex.: se eu pingo os n=FAmeros 123, eu ganho se constar no resultad= o as centenas 123, 321, 213, 231, 312 e 132. Ent=E3o, se C=E9sar pingou os n=FA= meros 557799, e apostou 4 reais em cada n=FAmero escolhido, ele gastou, em centavos, A) 9.600 B) 96.000 C) 1.200 D) 12.000 = =20 E) 63.680 10. Existem 4 caminhos: A, B, C, e D, sendo que x =3D 2y, 2y =3D 4z e 4z = =3D 8abc2. Observe-os e, sabendo que todas as inc=F3gnitas utilizadas pertenc= em ao conjunto Q*, o caminho mais curto =E9:=20 A) 8abc2 + 4z + 2y - x B) 4z + 2y + x + 8abc2 - x - 2y + 4z - 8abc2=20 C) 2y + x + x - 4z - 8abc2 D) 1 + 2y=20 E) N=E3o se pode identificar com os dados apresentados. 11. Na reta abaixo, x e y s=E3o n=FAmeros reais. Podemos afirmar que: A) ( x + y )( x - y )( x5y5 )( x/y ) =3D 1 -3 -2 = -1 0 1 2 3 B) x - y =3D 1=20 C) x + y =3D 1.5 D) xy =3D 0. 3125 E) x/y =3D -0.2 = =20 x y 12. Observe os seguintes dados: a < b < c x < y < z x =3D =D6z x =3D a + b + c x + y =3D z c + z =3D z + x - 1 b + a =3D b c =3D a + b + 1 Podemos afirmar que ( x + y + z ) . ( a + b + c ) =E9 igual a: A) 54 B) 64 C) 74 D) 84 E= ) bcxyz =20 13. Meu irm=E3o tem a quarta parte da idade de Daniel que tem 5/6 da de Gabriel que tem a quinta parte da raiz quadrada da metade da idade de uma obra de arte de 7.200 anos. A idade de meu irm=E3o =E9:=20 A) 2 anos e 5 meses B) 2 anos e 1 semestre C) 2 anos e 485 horas D) 2 anos e tr=EAs bimestres E) 2 anos e 125 dias 14. Um navio que est=E1 em alto mar possui uma escada lateral de 25 degra= us, sendo que a cada hora a mar=E9 sobe 1 metro. Se o primeiro degrau est=E1 = a meio metro do ch=E3o do navio, cada degrau =E9 separado por essa mesma dist=E2= ncia e mar=E9 est=E1 atingindo exatamente o ch=E3o do navio, quantos degraus fi= car=E3o submersos depois de 2 horas e meia? A) 0 B) 4 C) 5 D) 6 E) 25=20 15. Num hotel 5 estrelas est=E3o hospedadas 225 pessoas: 47 homens, 48 mulheres e x crian=E7as. Se das x crian=E7as, 52 t=EAm menos de 10 anos, = as que t=EAm mais representam, do n=FAmero total de h=F3spedes, aproximadamente,= =20 A) 51,9% B) 43,7% C) 39,1% D) 34,6% = =20 E) 48,4% 16. Se no planeta Z, a2 =3D 5x2, b =3D 8y2, c =3D =D6x, d =3D =D6y2 e x =3D= y, nesse planeta a express=E3o =D6d2 equivale a:=20 A) x B) 2x C) 2y D) 2xy E) 2=D6abc=20 17. No planeta Malthys est=E1 em andamento uma Copa do Mundo, na qual participam 32 sele=E7=F5es e h=E1 8 grupos, como a da Fran=E7a, aqui na T= erra. No grupo J, est=E3o quatro sele=E7=F5es: Sonata, Wekinj, Drusdell e Rammel. = J=E1 ocorreram as seguintes partidas:=20 Sonata F x R Wekinj, Drusdell J x N Rammel, Sonata R x R Rammel, Drusdell= N x N Wekinj e=20 Wekinj N x S Rammel. Sabendo que S, R, N, F e J s=E3o n=FAmeros naturais,= que Sonata venceu Drusdell por N a R e que nessa partida houve apenas um gol, observe as equa=E7=F5es abaixo: J =3D R + N + 1 F =3D J + N -S =3D -F - J + N + R O valor correspondente =E0 letra desse grupo somado a S resulta em: A) 5 B) 6 C) 7 D) 8 E) 9 18. Nesse mesmo planeta h=E1 um jogo popular, o qual =E9 disputado por 2 = times. A equipe que faz 4 pontos primeiro marca 1 s=E9rie, e aquela que ganha 6 s=E9ries antes que a rival vence uma 1 etapa. O vencedor =E9 aquele que consegue completar 3 etapas primeiro. Se o Villon derrotou o Lamme por 3 etapas a 1 (veja o n=FAmero de s=E9ries conquistadas pelos times em cada = etapa) e marcou mais pontos que o advers=E1rio, a menor diferen=E7a poss=EDvel e= ntre o n=FAmero de pontos ganhos pela equipe vencedora e pela derrotada =E9:=20 A) 1 B) 7 C) 28 D) 33 E) 36 = =20 =20 1a etapa 2a etapa 3a etapa 4a etapa Villon 4 6 6 = =20 6 Lamme 6 2 3 4= =20 19. Houve uma maratona nacional numa pista do Rio de Janeiro. Ap=F3s a corrida, alguns matem=E1ticos analisaram os dois primeiros colocados e constataram que, em m=E9dia, o maratonista ga=FAcho completava uma volta = em 50 segundos e o mineiro em 52. Em qual volta os dois atletas ficaram lado a lado? Quanto tempo ap=F3s a largada isso aconteceu? A) Na 25a volta, 20 minutos e 8 segundos ap=F3s a largada. B) Na 25a volta, 20 minutos e 48 segundos ap=F3s a largada. C) Na 26a volta, 21 minutos e 36 segundos ap=F3s a largada. D) Na 26a volta, 22 minutos e 26 segundos ap=F3s a largada. E) Na 26a volta, 22 minutos e 16 segundos ap=F3s a largada. 20. Pensei num n=FAmero de 4 algarismos. O primeiro algarismo equivale ao quarto somado a 7, o segundo =E9 igual ao dobro do quarto e o terceiro po= ssui o valor do segundo subtra=EDdo de 1. Um dos poss=EDveis valores da soma d= e todos os algarismos que comp=F5e o n=FAmero que eu pensei =E9: A) 15 B) 18 C) 21 D) 24 E) 27= =20 21. O telefone de Gustavo =E9 34X-YXZY. Observe os dados abaixo e tente descobrir o valor das tr=EAs inc=F3gnitas: X =3D 1 + Y Y =3D 8 - Z=20 Z =3D X - 7 Podemos concluir que uma das somas de todos os algarismos que comp=F5e o n=FAmero do telefone de Gustavo =E9 igual a: A) 31 B) 32 C) 34 D) 36 E) 38 22. O n=FAmero 12345678909876543210012345678900098765432100x0 =E9 divis=ED= vel por 9. Podemos afirmar que: A) Este n=FAmero =E9 primo e x vale 0, 3 ou 9. B) Este n=FAmero =E9 divis=EDvel por 3 e x vale 3. C) Este n=FAmero =E9 divis=EDvel por 5 e x vale 6. D) Este n=FAmero =E9 divis=EDvel por 144 e x vale 0 ou 9. E) Este n=FAmero =E9 divis=EDvel por 72 e x vale 0, 3 ou 9. =20 23. Quantos s=E3o os n=FAmeros de tr=EAs d=EDgitos cuja soma de seus tr=EA= s algarismos seja 12? A) 144 B) 52 C) 66 D) 82 E) 3= 6 24. Observe os seguintes dados:=20 a > b > c z < y < x=20 x > a y =3D c Sabendo que as seis inc=F3gnitas pertencem ao conjunto dos n=FAmeros inte= iros, se a soma de todas as vari=E1veis =E9 85 e o valor de z =E9 -15, o valor = m=E1ximo de x =E9: A) 139 B) 153 C) 218 D) 327 E= ) 457 25. Considere um ret=E2ngulo de per=EDmetro 56cm (todos os lados medem x = cm, sendo x sempre um n=FAmero natural diferente de 0). A diferen=E7a entre a= maior =E1rea poss=EDvel e a menor =E1rea poss=EDvel desse ret=E2ngulo =E9 de: A) 165 B) 166 C) 167 D) 168 E= ) 169 =20 Espa=E7o para rascunho. From owner-obm-rj@saci.mat.puc-rio.br Tue Nov 17 19:07:32 1998 Received: by saci.mat.puc-rio.br (AIX 3.2/UCB 5.64/4.03) id AA26348; Tue, 17 Nov 1998 19:07:31 -0200 Received: from diamante.netmarket.com.br by saci.mat.puc-rio.br (AIX 3.2/UCB 5.64/4.03) id AA21975; Tue, 17 Nov 1998 19:06:01 -0200 Received: from default (tty119 [200.248.242.119]) by diamante.netmarket.com.br (8.8.8+Sun/8.8.8) with ESMTP id SAA26043 for ; Tue, 17 Nov 1998 18:46:46 -0200 (EDT) Message-Id: <199811172046.SAA26043@diamante.netmarket.com.br> From: "Karin" To: Subject: =?ISO-8859-1?Q?I_Olimp=EDada_LPM_de_Matem=E1tica.?= Date: Wed, 16 Sep 1998 08:05:36 -0300 X-Msmail-Priority: Normal X-Priority: 3 X-Mailer: Microsoft Internet Mail 4.70.1161 Mime-Version: 1.0 Content-Type: text/plain; charset=ISO-8859-1 Content-Transfer-Encoding: quoted-printable X-Mime-Autoconverted: from 8bit to quoted-printable by diamante.netmarket.com.br id SAA26043 Sender: owner-obm-rj@saci.mat.puc-rio.br Precedence: bulk Reply-To: obm-rj@saci.mat.puc-rio.br Status: RO X-Status: X-Keywords: X-UID: 39 Caros amigos, entrei h=E1 pouco tempo neste grupo, portanto vou me apresentar: meu nome =E9 Lucas Povarczuk Mocelim, tenho 14 anos e resido = em Porto Alegre RS. Amo matem=E1tica, tanto =E9 que aqui estou. Vou mostrar = a voc=EAs a I Olimp=EDada LPM de Matem=E1tica, criada por mim (9 problemas)= e mais tarde enviarei uma outra prova minha. Gostaria que se apresentassem no pr=F3ximo e-mail.=20 I Olimp=EDada LPM de Matem=E1tica Brasil, 1998. Tempo: 4 horas e meia. Primeiro Dia. Problema 1 Num tri=E2ngulo acut=E2ngulo ABC, AB mede x2 e AC mede x3. Considere o po= nto M como m=E9dio do lado AC. Se AB =E9 congruente a MC, quanto vale o lado BC= ? =20 Problema 2 Seja x um n=FAmero natural que satisfaz =E0 inequa=E7=E3o = =20 __ =D6 xx =B3 x3 =A3 x2 + x + x + x + x + ... + x/1998. =20 `2 `4 `6 =20 (1) Quais s=E3o os poss=EDveis valores de x? =20 (2) Demonstre que 1998 =E9 um poss=EDvel valor de x. Problema 3 Uma caixa trancada possui cinco orif=EDcios circulares. Disp=F5e-se de ci= nco discos tamb=E9m circulares, cada um de uma cor diferente, que devem ser introduzidos nos orif=EDcios, um disco em cada orif=EDcio. Se os colocarm= os numa seq=FC=EAncia secreta, a caixa se abrir=E1, sendo que ao colocarmos = um disco em seu respectivo orif=EDcio, um mecanismo da caixa =E9 acionado; ao colo= carmos dois discos em seus respectivos orif=EDcios, dois mecanismos s=E3o aciona= dos e assim por diante, de forma que se todos os discos forem colocados na seq=FC=EAncia secreta, todos os cinco mecanismos ser=E3o acionados e a ca= ixa automaticamente se abrir=E1. Veja abaixo algumas poss=EDveis seq=FC=EAnci= as dos discos e o n=FAmero de mecanismos que elas acionam. Seq=FC=EAncia dos discos: N=FAmero de mec= anismos acionados: =20 Amarelo, Verde, Azul, Vermelho e Laranja. 0 Laranja, Vermelho, Azul, Verde e Amarelo. 1 Azul, Amarelo, Laranja, Vermelho e Verde. 2 Vermelho, Laranja, Amarelo, Verde e Azul. 3 (1) Considere o disco amarelo como A, o verde como B, o azul como C, o vermelho como D e o laranja como E. Fa=E7a uma lista bem organizada de to= das as poss=EDveis seq=FC=EAncias desses cinco discos e diga quantas s=E3o el= as.=20 (a) Observe a primeira seq=FC=EAncia mostrada. Podemos compar=E1-la com t= odas as outras que devem estar citadas na lista do item (1), e a partir desta compara=E7=E3o, podemos eliminar algumas delas, ou seja, podemos conhecer algumas seq=FC=EAncias (relacionadas na lista pedida) que, de acordo com = a primeira seq=FC=EAncia mostrada, n=E3o podem ser a correta para abrir a c= aixa. Mostre o n=FAmero exato de seq=FC=EAncias que podem ser eliminadas se com= paradas com a primeira mostrada e relacione numa nova lista todas as que restam. (b) Agora, considere somente as seq=FC=EAncias que devem constar na lista pedida no item anterior. Observe a segunda seq=FC=EAncia mostrada e diga = o n=FAmero exato das que s=E3o eliminadas quando comparadas a ela, fazendo = uma nova rela=E7=E3o que conste somente as restantes.=20 (c) Repita o processo, comparando a terceira seq=FC=EAncia mostrada com a= s que devem estar contidas na rela=E7=E3o do item anterior.=20 (2) Terminadas todas as listas com as respectivas informa=E7=F5es pedidas= , ap=F3s ter eliminado o maior n=FAmero poss=EDvel de seq=FC=EAncias, observe a =FA= ltima seq=FC=EAncia mostrada. Compare-a com a sua =FAltima rela=E7=E3o, tente e= liminar mais seq=FC=EAncias (se isto for poss=EDvel) e responda: Quantas seq=FC=EAncia= s ainda podem ser a correta para abrir a caixa? Se voc=EA achou mais do que uma, cite-as. Se voc=EA encontrou apenas uma, mostre-a. Se voc=EA n=E3o encont= rou nenhuma seq=FC=EAncia que poderia ser a verdadeira, ent=E3o diga que a ca= ixa jamais poder=E1 ser aberta! ____________________ Valor de cada problema:10 pontos I Olimp=EDada LPM de Matem=E1tica Brasil, 1998. Tempo: 4 horas e meia. Segundo Dia. Problema 4 A + B + C =3D D, AB =3D D e C2 =3D D. A + B + C + D =3D X, X + Y =3D Z e = Z =A3 1998. Se determinado que Y =3D 2, prove que _________ Z =3D Y - BY - CY + DY + Y=D6A + B + C + AY + BY + CY =BE=BE=BE=BE=BE=BE A Obs.: Todas as inc=F3gnitas s=E3o n=FAmeros naturais n=E3o-nulos, diferen= tes entre si. Problema 5 Seja LMN um tri=E2ngulo is=F3sceles interior ao ret=E2ngulo ABCD. Conside= re o conjunto de pontos E, F, K, P, Q e S como os pontos m=E9dios de AL, LB, N= D, AC, BD e CM, respectivamente, e o ponto T, como o p=E9 da altura de L.. C= D mede 222 cm, AL mede 155 cm e TN mede 44 cm. Explique por que a =E1rea de= um poss=EDvel tri=E2ngulo a ser tra=E7ado no interior ao ret=E2ngulo ABCD (m= ostre-o) =E9 1998 se e somente se a altura deste tri=E2ngulo for um natural divisor po= r 1998. Problema 6 O matem=E1tico Mathwell elaborou um jogo para seus colegas de estudo. Inicialmente, ele exibiu um cartaz contendo as seguintes igualdades: __=20 x =3D =D6 z x =3D a + b + c x + y =3D z c + z =3D z + x - 1 b + a =3D b c =3D a + b + 1 Depois, ele explicou os procedimentos. Durante 1998 dias, cada matem=E1ti= co jogaria diariamente 98 vezes um dado de seis faces, sendo elas a, b, c, x= , y, z, todas presentes no cartaz mostrado. Cada inc=F3gnita representava u= m n=FAmero natural, que correspondia com o seu valor em pontos. Assim, as 9= 8 jogadas di=E1rias do dado deveriam ser anotadas e somadas, resultando num total de pontos a cada dia. O processo continuaria at=E9 o 1998=B0 dia, q= uando seria declarado o vencedor, aquele que teria feito mais pontos ao final d= o tempo especificado.=20 (1) Mostre o valor de cada uma das seis faces do dado. (2) Se um competidor, durante todos os dias do jogo, terminasse sempre co= m um valor exatamente m=E9dio de pontos, qual seria a sua pontua=E7=E3o ao = t=E9rmino do jogo? E se o dado utilizado pelos matem=E1ticos fosse comum, de faces 1,2,3,4,5 e 6, qual seria esta pontua=E7=E3o? ____________________ Valor de cada problema: 10 pontos. I Olimp=EDada LPM de Matem=E1tica Brasil, 1998. Tempo: 4 horas e meia. Terceiro Dia. Problema 7 Um tabuleiro ABCD =E9 dividido em 14 x 14 quadrados unit=E1rios. Uma fich= a =E9 colocada num dos v=E9rtices do tabuleiro e dever=E1 chegar at=E9 o v=E9rt= ice diagonal, atrav=E9s de movimentos iguais aos da pe=E7a "Cavalo" do jogo d= e xadrez. Prove que para a ficha chegar ao v=E9rtice oposto, s=E3o necess=E1= rios no m=EDnimo 10 movimentos. Problema 8 Utilizando os algarismos de 0 a 9 exatamente uma vez, construa dois n=FAm= eros (para cada item) tal que: (1) A diferen=E7a entre eles (o maior menos o menor) seja a menor poss=ED= vel. (2) A diferen=E7a entre eles (o maior menos o menor) seja a maior poss=ED= vel. (3) A soma deles seja a menor poss=EDvel. (4) A soma deles seja a maior poss=EDvel. (5) O produto deles seja o menor poss=EDvel. (6) O produto deles seja o maior poss=EDvel. Problema 9 Considere uma hex=E1gono de =E2ngulos congruentes. Tra=E7amos retas na fi= gura, sempre ligando um v=E9rtice a tr=EAs outros n=E3o adjacentes, de forma qu= e de cada v=E9rtice saem tr=EAs retas. Mostre 96 tri=E2ngulos que podem ser tr= a=E7ados interiormente ao hex=E1gono a partir dos v=E9rtices deste e dos pontos de intersec=E7=E3o das retas. ____________________ Valor de cada problema: 10 pontos. =20 =20 From owner-obm-rj@saci.mat.puc-rio.br Tue Nov 17 19:12:45 1998 Received: by saci.mat.puc-rio.br (AIX 3.2/UCB 5.64/4.03) id AA15357; Tue, 17 Nov 1998 19:12:32 -0200 Received: from diamante.netmarket.com.br by saci.mat.puc-rio.br (AIX 3.2/UCB 5.64/4.03) id AA26096; Tue, 17 Nov 1998 19:09:09 -0200 Received: from default (tty119 [200.248.242.119]) by diamante.netmarket.com.br (8.8.8+Sun/8.8.8) with ESMTP id TAA26729 for ; Tue, 17 Nov 1998 19:10:27 -0200 (EDT) Message-Id: <199811172110.TAA26729@diamante.netmarket.com.br> From: "Karin" To: Subject: Segunda Fase Date: Wed, 16 Sep 1998 08:29:15 -0300 X-Msmail-Priority: Normal X-Priority: 3 X-Mailer: Microsoft Internet Mail 4.70.1161 Mime-Version: 1.0 Content-Type: text/plain; charset=ISO-8859-1 Content-Transfer-Encoding: quoted-printable X-Mime-Autoconverted: from 8bit to quoted-printable by diamante.netmarket.com.br id TAA26729 Sender: owner-obm-rj@saci.mat.puc-rio.br Precedence: bulk Reply-To: obm-rj@saci.mat.puc-rio.br Status: RO X-Status: X-Keywords: X-UID: 40 Fal=F4 amizade, ai v=E3o os tr=EAs primeiros problemas da segunda fase do "Desafio aos seus Neur=F4nios" (tamb=E9m s=E3o uma barbada). Logo eu mand= o mais problemas, mas primeiro eu vou deixar voc=EAs respirarem um pouco com est= es e com as outras provas que eu mandei em e-mails anteriores. A=ED vai: Desafio aos seus Neur=F4nios - 1998 2a fase Lucas Povarczuk Mocelim 1. Preencha o quadro abaixo utilizando as instru=E7=F5es dadas. As posi=E7= =F5es das colunas onde se encontram o nome dos personagens correspondem =E0s posi=E7= =F5es de suas casas. Nome: Rafael Andr=E9 Jo=E3o Eduardo=09 Casa: =09 Carro: =09 Comida preferida: lasanha =09 Cor preferida: =09 Banda preferida: =09 Esporte que pratica: futebol =09 Time que torce: =09 Pessoa que admira: Carla Perez =09 Profiss=E3o: escritor=09 Caracter=EDstica: =09 =20 Quem tem uma mans=E3o adora os Beatles e o Gr=EAmio, mas detesta t=EAnis. Quem =E9 forte admira muito a Luana Piovani e a cor amarela, por=E9m odei= a camar=E3o. Quem joga golfe mora =E0 direita de quem n=E3o possui carro. Quem =E9 apaixonado pela Ana Paula Ar=F3sio jamais dispensa uma =E0 la mi= nuta e mora a tr=EAs casas de dist=E2ncia de quem tem uma Ferrari. A casa pequena fica =E0 direita da m=E9dia que fica =E0 direita da grande= que n=E3o fica ao lado da mans=E3o. Quem possui um Scort mora a duas casas de dist=E2ncia de quem =E9 um segu= ran=E7a e a uma casa de quem tem um Corsa. Quem =E9 m=E9dico mora ao lado direito de quem tem uma casa pequena. Os dois colorados moram lado a lado, ao contr=E1rio dos gremistas. Quem =E9 mec=E2nico mora ao lado de quem admira demasiadamente a Ana Paul= a Ar=F3sio. Quem aprecia a cor amarela mora ao lado direito de quem =E9 baixo e prati= ca automobilismo.=20 Quem =E9 seguran=E7a curte muito os Rolling Stones. Quem ama comer pizza n=E3o mora ao lado daquele que adora a cor azul cele= ste. Quem gosta do grupo =C9 o Tchan mora ao lado de quem =E9 magro. Quem =E9 homossexual mora ao lado de quem gosta de jogar futebol. Quem escuta muito a banda Legi=E3o Urbana mora ao lado de quem aprecia a = cor bord=F4. Quem ama comer pizza mora ao lado de quem =E9 f=E3 do Leonardo Di Caprio = e admira a cor rosa-choque. Quem pratica t=EAnis =E9 gremista. =20 2. Houve um roubo de meio milh=E3o de reais num grande shopping e h=E1 ci= nco suspeitos. Veja abaixo o nome, a idade e as declara=E7=F5es de cada um de= les: Bruno (18) - Marcos =E9 o culpado. Fernando (21) - N=E3o tenho a m=EDnima id=E9ia de quem roubou a quantia. Guilherme (27) - O ladr=E3o tem menos de 20 anos de idade. Marcos (16) - Fernando =E9 o culpado. Matheus (17) - N=E3o sei quem =E9 o culpado. Sabendo que apenas um mentiu e que o roubo dos R$ 500.000,00 foi cometido por uma =FAnica pessoa, revele quem =E9 o culpado, mostrando e justifican= do o racioc=EDnio elaborado por voc=EA para chegar a sua resposta. 3. Tonial, Juliana, Simone e Lucas s=E3o filhos de Malba Tahan. Observe a= s afirmativas de cada um deles, ap=F3s uma corrida: Tonial: A Juliana venceu e eu tirei segundo lugar. Lucas: A Simone venceu e eu tirei terceiro lugar. Simone: O Tonial foi o =FAltimo colocado e o Lucas venceu. Juliana: O Lucas e a Simone mentiram uma vez, enquanto que o Tonial n=E3o mentiu. Se cada um disse uma afirmativa verdadeira e outra falsa, qual foi a orde= m de chegada? Transcreva o seu racioc=EDnio. =20 From owner-obm-rj@saci.mat.puc-rio.br Tue Nov 17 20:48:39 1998 Received: by saci.mat.puc-rio.br (AIX 3.2/UCB 5.64/4.03) id AA21782; Tue, 17 Nov 1998 20:48:31 -0200 Received: from mail.iis.com.br by saci.mat.puc-rio.br (AIX 3.2/UCB 5.64/4.03) id AA23570; Tue, 17 Nov 1998 20:48:15 -0200 Received: from Rffsa.rffsa.gov.br (rio-as8-tty02.iis.com.br [200.202.97.130]) by mail.iis.com.br (8.8.7/8.8.14) with SMTP id UAA07093 for ; Tue, 17 Nov 1998 20:48:13 -0200 From: "Marcio" To: Subject: =?iso-8859-1?Q?Re:_Sobre_a_terceira_fase_da_olimp=EDada.?= Date: Tue, 17 Nov 1998 20:55:49 -0300 Message-Id: <01be1285$cd37f760$8261cac8@Rffsa.rffsa.gov.br> Mime-Version: 1.0 Content-Type: text/plain; charset="iso-8859-1" X-Priority: 3 X-Msmail-Priority: Normal X-Mailer: Microsoft Outlook Express 4.71.1712.3 X-Mimeole: Produced By Microsoft MimeOLE V4.71.1712.3 Content-Transfer-Encoding: quoted-printable X-Mime-Autoconverted: from 8bit to quoted-printable by mail.iis.com.br id UAA07093 Sender: owner-obm-rj@saci.mat.puc-rio.br Precedence: bulk Reply-To: obm-rj@saci.mat.puc-rio.br Status: RO X-Status: X-Keywords: X-UID: 41 Na verdade eu tmb nao pensei na opcao de eles se separarem. O que eu pensei, apesar de parecer trivial demais, =E9 que eles poderiam andar N quadras para a esquerda e caso nao encontrassem a rua desejada, andariam mais N quadras ate o local inicial e depois mais N para a direita, chegan= do entao com certeza a rua desejada. Acho que eu nao consegui interpretar o problema da maneira correta. Obrigado, Marcio -----Original Message----- From: Carlos Gustavo Tamm de Araujo Moreira To: obm-rj@saci.mat.puc-rio.br Date: Ter=E7a-feira, 17 de Novembro de 1998 16:04 Subject: Re: Sobre a terceira fase da olimp=EDada. > Caro Marcio, > Vamos colocar em breve todas as solucoes na home-page.Sobre o probl= ema >6,que e' baseado em fatos reais(a situacao ocorreu comigo e com o Nicola= u, >quando estavamos em Berlim no congresso internacional de matematica,em >agosto),a ideia e' que os dois andassem sempre juntos,e a resposta corre= ta >e' 9.Uma estrategia possivel e' andarem 1 quarteirao para a direita,voltarem >a origem e andarem 2 pra esquerda,voltarem pra origem e andarem 4 pra >esquerda,etc.Nao tinhamos pensado na hipotese de eles se separarem,mas vamos >estudar caso a caso solucoes coerentes com essa interpretacao. > Abracos, > Carlos Gustavo Moreira ( Gugu) > From owner-obm-rj@saci.mat.puc-rio.br Tue Nov 17 20:59:01 1998 Received: by saci.mat.puc-rio.br (AIX 3.2/UCB 5.64/4.03) id AA25896; Tue, 17 Nov 1998 20:58:59 -0200 Received: from mail.iis.com.br by saci.mat.puc-rio.br (AIX 3.2/UCB 5.64/4.03) id AA18980; Tue, 17 Nov 1998 20:58:47 -0200 Received: from Rffsa.rffsa.gov.br (rio-as7-tty01.iis.com.br [200.202.97.113]) by mail.iis.com.br (8.8.7/8.8.14) with SMTP id UAA08162 for ; Tue, 17 Nov 1998 20:58:46 -0200 From: "Marcio" To: Subject: Re: Date: Tue, 17 Nov 1998 21:05:51 -0300 Message-Id: <01be1287$34425580$8261cac8@Rffsa.rffsa.gov.br> Mime-Version: 1.0 Content-Type: text/plain; charset="iso-8859-1" X-Priority: 3 X-Msmail-Priority: Normal X-Mailer: Microsoft Outlook Express 4.71.1712.3 X-Mimeole: Produced By Microsoft MimeOLE V4.71.1712.3 Content-Transfer-Encoding: quoted-printable X-Mime-Autoconverted: from 8bit to quoted-printable by mail.iis.com.br id UAA08162 Sender: owner-obm-rj@saci.mat.puc-rio.br Precedence: bulk Reply-To: obm-rj@saci.mat.puc-rio.br Status: RO X-Status: X-Keywords: X-UID: 42 Infelizmente, domingo =E9 o dia da prova da UFRJ e eu acredito que mu= itos de nos (pelo menos os vestibulandos) nao poderiamos assistir a essa aula. []'s Marcio -----Original Message----- From: Nicolau Corcao Saldanha To: obm-rj@saci.mat.puc-rio.br Date: Ter=E7a-feira, 17 de Novembro de 1998 17:24 Caros, Deve haver palestra este domingo, no lugar e hora de sempre: 14 =E0s 18, na sala 150L da PUC-Rio. Deve falar o professor Edmilson, do Col=E9gio Etapa, em S=E3o Paulo. Nicolau P.S.: Escrevam mais, mandem mais problemas, esta lista est=E1 muito parad= a. From owner-obm-rj@saci.mat.puc-rio.br Wed Nov 18 11:56:04 1998 Received: by saci.mat.puc-rio.br (AIX 3.2/UCB 5.64/4.03) id AA16666; Wed, 18 Nov 1998 11:54:50 -0200 Received: from euler.impa.br by saci.mat.puc-rio.br (AIX 3.2/UCB 5.64/4.03) id AA14356; Wed, 18 Nov 1998 11:54:36 -0200 Received: from Descartes.impa.br (descartes.impa.br [147.65.13.34]) by Euler.impa.br (8.8.6/8.8.6) with ESMTP id LAA19924 for ; Wed, 18 Nov 1998 11:54:35 -0200 (EDT) From: Carlos Gustavo Tamm de Araujo Moreira Received: by Descartes.impa.br (8.8.8) id LAA15787; Wed, 18 Nov 1998 11:54:35 -0200 (EDT) Message-Id: <199811181354.LAA15787@Descartes.impa.br> Subject: Re: =?iso-8859-1?Q?Re:_Sobre_a_terceira_fase_da_olimp=EDada.?= To: obm-rj@saci.mat.puc-rio.br Date: Wed, 18 Nov 1998 11:54:34 -0200 (EDT) In-Reply-To: <01be1285$cd37f760$8261cac8@Rffsa.rffsa.gov.br> from "Marcio" at Nov 17, 98 08:55:49 pm X-Mailer: ELM [version 2.4 PL25] Mime-Version: 1.0 Content-Type: text/plain; charset=US-ASCII Content-Transfer-Encoding: 7bit Sender: owner-obm-rj@saci.mat.puc-rio.br Precedence: bulk Reply-To: obm-rj@saci.mat.puc-rio.br Status: RO X-Status: X-Keywords: X-UID: 43 Caro Marcio, Os matematicos NAO sabem o valor de N,e portanto devem ziguezaguear ate' acharem a Meiningerstrasse. Abracos, Gugu From owner-obm-rj@saci.mat.puc-rio.br Wed Nov 18 12:11:41 1998 Received: by saci.mat.puc-rio.br (AIX 3.2/UCB 5.64/4.03) id AA16744; Wed, 18 Nov 1998 12:11:29 -0200 Received: from mail.iis.com.br by saci.mat.puc-rio.br (AIX 3.2/UCB 5.64/4.03) id AA25183; Wed, 18 Nov 1998 12:11:14 -0200 Received: from Rffsa.rffsa.gov.br (rio-as6-tty06.iis.com.br [200.202.97.102]) by mail.iis.com.br (8.8.7/8.8.14) with SMTP id MAA21103 for ; Wed, 18 Nov 1998 12:11:12 -0200 From: "Marcio" To: Subject: =?iso-8859-1?Q?Re:_Re:_Sobre_a_terceira_fase_da_olimp=EDada.?= Date: Wed, 18 Nov 1998 12:18:48 -0300 Message-Id: <01be1306$bd6384e0$6661cac8@Rffsa.rffsa.gov.br> Mime-Version: 1.0 Content-Type: text/plain; charset="iso-8859-1" X-Priority: 3 X-Msmail-Priority: Normal X-Mailer: Microsoft Outlook Express 4.71.1712.3 X-Mimeole: Produced By Microsoft MimeOLE V4.71.1712.3 Content-Transfer-Encoding: quoted-printable X-Mime-Autoconverted: from 8bit to quoted-printable by mail.iis.com.br id MAA21103 Sender: owner-obm-rj@saci.mat.puc-rio.br Precedence: bulk Reply-To: obm-rj@saci.mat.puc-rio.br Status: RO X-Status: X-Keywords: X-UID: 44 Realmente, o problema inicialmente citava isso. =E9 que como dizia "o numero K tal que se existem N quadras entre as duas ruas..." eu considere= i como sabido que eram N quadras (proposicoes do tipo se-entao). Mas admito que foi uma consideracao bem fraca.. :) no caso, eu sei que sao N quadras, mas os matematicos nao sabem, correto? Nesse caso eu nao faco a menor ideia de como achar K.. vou ver s= e penso um pouco... Obrigado, Marcio -----Original Message----- From: Carlos Gustavo Tamm de Araujo Moreira To: obm-rj@saci.mat.puc-rio.br Date: Quarta-feira, 18 de Novembro de 1998 10:56 Subject: Re: Re: Sobre a terceira fase da olimp=EDada. > Caro Marcio, > Os matematicos NAO sabem o valor de N,e portanto devem ziguezaguea= r >ate' acharem a Meiningerstrasse. > Abracos, > Gugu > From owner-obm-rj@saci.mat.puc-rio.br Wed Nov 18 22:04:44 1998 Received: by saci.mat.puc-rio.br (AIX 3.2/UCB 5.64/4.03) id AA19149; Wed, 18 Nov 1998 22:04:31 -0200 Received: from diamante.netmarket.com.br by saci.mat.puc-rio.br (AIX 3.2/UCB 5.64/4.03) id AA17349; Wed, 18 Nov 1998 22:02:04 -0200 Received: from default (tty114 [200.248.242.114]) by diamante.netmarket.com.br (8.8.8+Sun/8.8.8) with ESMTP id WAA28207 for ; Wed, 18 Nov 1998 22:03:19 -0200 (EDT) Message-Id: <199811190003.WAA28207@diamante.netmarket.com.br> From: "Karin" To: Subject: =?ISO-8859-1?Q?TR=CAS_PROBLEMAS?= Date: Wed, 16 Sep 1998 23:12:49 -0300 X-Msmail-Priority: Normal X-Priority: 3 X-Mailer: Microsoft Internet Mail 4.70.1161 Mime-Version: 1.0 Content-Type: text/plain; charset=ISO-8859-1 Content-Transfer-Encoding: quoted-printable X-Mime-Autoconverted: from 8bit to quoted-printable by diamante.netmarket.com.br id WAA28207 Sender: owner-obm-rj@saci.mat.puc-rio.br Precedence: bulk Reply-To: obm-rj@saci.mat.puc-rio.br Status: RO X-Status: X-Keywords: X-UID: 47 Prezados amigos, quem se habilita a resolver estes tr=EAs problemas meus? (mostrando como chegou =E0 resolu=E7=E3o). Problema 1 Um grupo integrado por 98 escoteiros, sendo um deles voc=EA, senta e form= a um grande c=EDrculo. Trata-se de um encontro no qual um de voc=EAs receber=E1= um grande pr=EAmio, da seguinte forma: =E9 feito um sorteio; o sortiado esco= lher=E1 um integrante da roda e contar=E1 at=E9 7, a partir deste, no sentido hor= =E1rio. O s=E9timo escoteiro a ser contado =E9 eliminado da disputa e retira-se d= o c=EDrculo. Inicia-se ent=E3o uma nova contagem a partir daquele que estav= a =E0 direita do eliminado, e o s=E9timo escoteiro a ser contado =E9 retirado d= o grupo. O processo se repete at=E9 que apenas um sobre, e este ser=E1 o vencedor. Sendo voc=EA o sortiado, qual dos escoteiros deve ser escolhido para abrir a contagem, de forma que voc=EA receba o pr=EAmio? Obs.: Cada escoteiro veste uma camiseta com um n=FAmero natural de 1 a 98= , sendo que o de camiseta 1 est=E1 sentado entre os de camisetas 2 e 98, o = de camiseta 2 est=E1 sentado entre os de camiseta 1 e 3, e assim por diante. Voc=EA veste a de n=FAmero 1. Problema 2 Qual =E9 o valor da soma 1 + 2 + 3 + ... + 1998? =20 Problema 3 Crie uma fun=E7=E3o F tal que F(1998) =E9 123456789 e F(1) =E9 1998. lucas =20 =20 From crenfielt@usa.net Sun Nov 22 17:53:31 1998 Reply-To: From: "Crenfielt" To: Subject: =?ISO-8859-1?Q?Problema_de_an=E1lise_combinat=F3ria?= Date: Sun, 22 Nov 1998 18:05:36 -0500 X-Msmail-Priority: Normal X-Priority: 3 Content-Type: text/plain; charset=ISO-8859-1 X-Mime-Autoconverted: from 8bit to quoted-printable by mail.iis.com.br id RAA02362 Status: O X-Status: X-Keywords: X-UID: 48 =09 Gostaria de ajuda para resolver um problema de an=E1lise combinat=F3ria = que foi proposto numa prova de um curso preparat=F3rio=20 para o ime. Numa festa, cinco casais de amigos combinaram dan=E7ar as tres =FAltimas m=FAsicas da seguinte maneira: - os casais n=E3o poder=E3o dan=E7ar entre si, ou seja, haver=E1 sempre t= roca na forma=E7=E3o dos pares - cada par formado dever=E1 dan=E7ar exatamente uma m=FAsica. - poder=E1 existir m=FAsica sem qualquer um destes pares dan=E7ando. De quantas maneiras poder=E1 ser realizada a combina=E7=E3o acima descrit= a? p.s. Eu j=E1 vi uma solu=E7=E3o para este problema que era bem compacta, = mas que usava certos comceitos que eu nunca havia visto. From owner-obm-rj@saci.mat.puc-rio.br Mon Nov 23 13:18:52 1998 Date: Mon, 23 Nov 1998 13:17:12 -0200 From: Nicolau Corcao Saldanha To: crenfielt@usa.net Cc: obm-rj@saci.mat.puc-rio.br In-Reply-To: <199811221953.RAA02362@mail.iis.com.br> (crenfielt@usa.net) Subject: Re: Problema_de_an=E1lise_combinat=F3ria?= Sender: owner-obm-rj@saci.mat.puc-rio.br Precedence: bulk Reply-To: obm-rj@saci.mat.puc-rio.br Status: O X-Status: X-Keywords: X-UID: 49 Caro Crenfielt, Estou tomando a liberdade de mandar cópia do seu problema para o grupo de discussão obm-rj@mat.puc-rio.br; você também deve receber convite (auto-explicativo, em inglês) para entrar nesta lista de discussão. Caros membros de obm-rj, segue cópia do problema do Crenfielt. Devo escrever uma solução mais tarde. []s, Nicolau Gostaria de ajuda para resolver um problema de an=E1lise combinat=F3ria = que foi proposto numa prova de um curso preparat=F3rio=20 para o ime. Numa festa, cinco casais de amigos combinaram dan=E7ar as tres =FAltimas m=FAsicas da seguinte maneira: - os casais n=E3o poder=E3o dan=E7ar entre si, ou seja, haver=E1 sempre t= roca na forma=E7=E3o dos pares - cada par formado dever=E1 dan=E7ar exatamente uma m=FAsica. - poder=E1 existir m=FAsica sem qualquer um destes pares dan=E7ando. De quantas maneiras poder=E1 ser realizada a combina=E7=E3o acima descrit= a? p.s. Eu j=E1 vi uma solu=E7=E3o para este problema que era bem compacta, = mas que usava certos comceitos que eu nunca havia visto. From owner-obm-rj@saci.mat.puc-rio.br Mon Nov 23 18:25:35 1998 From: "Karin" To: Subject: Um problema para a Lista. Date: Sat, 19 Sep 1998 11:40:04 -0300 X-Msmail-Priority: Normal X-Priority: 3 Content-Type: text/plain; charset=ISO-8859-1 X-Mime-Autoconverted: from 8bit to quoted-printable by diamante.netmarket.com.br id SAA23716 Sender: owner-obm-rj@saci.mat.puc-rio.br Precedence: bulk Reply-To: obm-rj@saci.mat.puc-rio.br Status: O X-Status: X-Keywords: X-UID: 50 Problema ABC...: Escrevemos a sequ=EAncia de algarismos, sem separ=E1-los, 123456789101112131415161718192021222324... Qual =E9 o algarismo de lugar 123456789? E de lugar 1998? From mcohen@iis.com.br Tue Nov 24 09:17:41 1998 Reply-To: "Marcio" From: "Marcio" To: "Nicolau Corcao Saldanha" Subject: Re: Problema_de_an=E1lise_combinat=F3ria?= Date: Tue, 24 Nov 1998 09:24:47 -0300 Content-Type: text/plain; charset="iso-8859-1" X-Priority: 3 X-Msmail-Priority: Normal X-Mimeole: Produced By Microsoft MimeOLE V4.71.1712.3 X-Mime-Autoconverted: from 8bit to quoted-printable by mail.iis.com.br id JAA26510 Status: O X-Status: X-Keywords: X-UID: 51 Desculpe Nicolau. Esse problema foi mandado por mim (Marcio Cohen). E= h que eu usei outro computador para manda-lo e ele tava configurado com o email que eu usava antigamente (crenfielt@usa.net). Desculpe o transtorno. -----Original Message----- From: Nicolau Corcao Saldanha To: crenfielt@usa.net Cc: obm-rj@mat.puc-rio.br Date: Segunda-feira, 23 de Novembro de 1998 12:17 Subject: Re: Problema_de_an=3DE1lise_combinat=3DF3ria?=3D >Caro Crenfielt, > >Estou tomando a liberdade de mandar c=F3pia do seu problema para o grupo >de discuss=E3o obm-rj@mat.puc-rio.br; voc=EA tamb=E9m deve receber convi= te >(auto-explicativo, em ingl=EAs) para entrar nesta lista de discuss=E3o. > >Caros membros de obm-rj, segue c=F3pia do problema do Crenfielt. >Devo escrever uma solu=E7=E3o mais tarde. []s, Nicolau > > Gostaria de ajuda para resolver um problema de an=3DE1lise combinat=3D= F3ria =3D > que > foi proposto numa prova de um curso preparat=3DF3rio=3D20 > para o ime. > > > Numa festa, cinco casais de amigos combinaram dan=3DE7ar as tres =3DF= Altimas > m=3DFAsicas da seguinte maneira: > > - os casais n=3DE3o poder=3DE3o dan=3DE7ar entre si, ou seja, haver=3D= E1 sempre t=3D > roca na > forma=3DE7=3DE3o dos pares > > > - cada par formado dever=3DE1 dan=3DE7ar exatamente uma m=3DFAsica. > > - poder=3DE1 existir m=3DFAsica sem qualquer um destes pares dan=3DE7= ando. > > De quantas maneiras poder=3DE1 ser realizada a combina=3DE7=3DE3o aci= ma descrit=3D > a? > > > p.s. Eu j=3DE1 vi uma solu=3DE7=3DE3o para este problema que era bem = compacta, =3D > mas que > usava certos comceitos que eu nunca havia visto. > > From kpm@netmarket.com.br Tue Nov 24 14:07:52 1998 Reply-To: <@netmarket.com.br> From: "Karin" To: Subject: =?ISO-8859-1?Q?Matem=E1tica=2C_Legi=E3o=2C_T=EAnis_=2C_Gr=EAmio?= Date: Sun, 20 Sep 1998 00:38:33 -0300 X-Msmail-Priority: Normal X-Priority: 3 Content-Type: text/plain; charset=ISO-8859-1 X-Mime-Autoconverted: from 8bit to quoted-printable by diamante.netmarket.com.br id OAA23393 Status: O X-Status: X-Keywords: X-UID: 52 Nicolau, a=ED vai o problema 4 da minha mais recente olimp=EDada (os tr=EAs primeiros eu mandei em um e-mail para a lista, no qual eu coloquei no assunto "TR=CAS PROBLEMAS") Problema 4 a) ABC + ABC + ABC =3D BBB=20 b) ACD + ACD + ACD =3D DDD Prove que BBB + DDD =3D EEE, com EEE divis=EDvel por 1998.=20 Nicolau, t=E1 na hora de tu me mandar as solu=E7=F5es de alguns de meus p= roblemas j=E1 enviados!!!!!!!!!!! Faz um temp=E3o j=E1, hem?Ahh, outra coisa: resp= onde o meu e-mail anterior, pois n=E3o entendi os teus exemplos de cobrir o diam= ante asteca com domin=F3s. Logo logo eu descubro a f=F3rmula! From owner-obm-rj@saci.mat.puc-rio.br Wed Dec 9 09:57:21 1998 Received: by saci.mat.puc-rio.br (AIX 3.2/UCB 5.64/4.03) id AA19744; Wed, 9 Dec 1998 09:56:52 -0200 Received: from euler.impa.br by saci.mat.puc-rio.br (AIX 3.2/UCB 5.64/4.03) id AA09244; Wed, 9 Dec 1998 09:56:29 -0200 Received: from Obm-01.impa.br (obm-01.impa.br [147.65.2.170]) by Euler.impa.br (8.8.6/8.8.6) with SMTP id JAA28910 for ; Wed, 9 Dec 1998 09:56:28 -0200 (EDT) Message-Id: <3.0.5.32.19981209095727.007bdaa0@pop.impa.br> X-Sender: obm@pop.impa.br X-Mailer: QUALCOMM Windows Eudora Light Version 3.0.5 (32) Date: Wed, 09 Dec 1998 09:57:27 -0200 To: obm-rj@saci.mat.puc-rio.br From: Olimpiada Brasileira de Matematica Subject: Resultados da XX OBM. Mime-Version: 1.0 Content-Type: text/plain; charset="us-ascii" Sender: owner-obm-rj@saci.mat.puc-rio.br Precedence: bulk Reply-To: obm-rj@saci.mat.puc-rio.br Status: RO X-Status: X-Keywords: X-UID: 53 Caros: O resultado final da XX Olimpiada Brasileira de Matematica ja' esta' publicado na home-page da OBM. http//www.obm.org.br/ (veja novidades). Um abraco, Nelly. From owner-obm-rj@saci.mat.puc-rio.br Wed Dec 9 11:02:06 1998 Received: by saci.mat.puc-rio.br (AIX 3.2/UCB 5.64/4.03) id AA27326; Wed, 9 Dec 1998 11:01:54 -0200 Received: from mula.mat.puc-rio.br by saci.mat.puc-rio.br (AIX 3.2/UCB 5.64/4.03) id AA24506; Wed, 9 Dec 1998 11:01:34 -0200 Received: (from nicolau@localhost) by mula.mat.puc-rio.br (8.8.7/8.8.7) id LAA07196 for obm-rj@saci.mat.puc-rio.br; Wed, 9 Dec 1998 11:01:32 -0200 Date: Wed, 9 Dec 1998 11:01:32 -0200 From: Nicolau Corcao Saldanha Message-Id: <199812091301.LAA07196@mula.mat.puc-rio.br> To: obm-rj@saci.mat.puc-rio.br Subject: Re: Resultados da XX OBM. Sender: owner-obm-rj@saci.mat.puc-rio.br Precedence: bulk Reply-To: obm-rj@saci.mat.puc-rio.br Status: RO X-Status: X-Keywords: X-UID: 54 > O resultado final da XX Olimpiada Brasileira de Matematica ja' esta' > publicado na home-page da OBM. > http//www.obm.org.br/ (veja novidades). Ja? Mas jah eh oficial? Nos nao iamos aguardar que os coordenadores regionais se manifestassem? Ou este prazo jah passou? Nicolau From owner-obm-rj@saci.mat.puc-rio.br Wed Dec 9 12:42:54 1998 Received: by saci.mat.puc-rio.br (AIX 3.2/UCB 5.64/4.03) id AA17582; Wed, 9 Dec 1998 12:42:41 -0200 Received: from ns-3.dglnet.com.br by saci.mat.puc-rio.br (AIX 3.2/UCB 5.64/4.03) id AA21390; Wed, 9 Dec 1998 12:36:51 -0200 Received: from juhu3.dglnet.com.br (pm1p22.jd.dglnet.com.br [200.246.57.55]) by ns-3.dglnet.com.br (8.9.1a/8.9.1/Debian/GNU) with SMTP id LAA23952 for ; Wed, 9 Dec 1998 11:48:06 -0200 Message-Id: <366E7E9B.2CEB@dglnet.com.br> Date: Wed, 09 Dec 1998 11:43:55 -0200 From: Arconcher X-Mailer: Mozilla 3.01 (Win95; I) Mime-Version: 1.0 To: obm-rj@saci.mat.puc-rio.br Subject: Re: Resultados da XX OBM. References: <3.0.5.32.19981209112311.007bfc90@pop.impa.br> Content-Type: text/plain; charset=iso-8859-1 Content-Transfer-Encoding: quoted-printable X-Mime-Autoconverted: from 8bit to quoted-printable by ns-3.dglnet.com.br id LAA23952 Sender: owner-obm-rj@saci.mat.puc-rio.br Precedence: bulk Reply-To: obm-rj@saci.mat.puc-rio.br Status: RO X-Status: X-Keywords: X-UID: 55 Olimpiada Brasileira de Matematica wrote: >=20 > At 11:01 09/12/98 -0200, you wrote: > >> O resultado final da XX Olimpiada Brasileira de Matematica ja' esta' > >> publicado na home-page da OBM. > >> http//www.obm.org.br/ (veja novidades). > > > >Ja? Mas jah eh oficial? Nos nao iamos aguardar que os coordenadores re= gionais > >se manifestassem? Ou este prazo jah passou? Nicolau > > > Oi Nicolau: >=20 > O Prazo para os coordenadores se manifestarem era ate' = segunda feira, eu > recebi varias manifestacoes deles, tanto em e-mail como por telefone, > achando o resultado otimo e parabenizando a comissao pelo bom > trabalho,(alguns deles fizeram correcoes nos nomes dos premiados), so'a > nossa coordenadora do Mato Grosso do Sul ficou um tanto triste por nao = ter > tido premiados na cidade dela. >=20 > Sendo assim o Professor Wagner autorizou a colocacao do= s resultados na > Home-Page. >=20 > Espero hoje chegue a tua passagem para B. Aires, assim = como esta chegue > as minhas maos eu peco para Eduardo passar pela PUC. >=20 > Um abraco, Nelly. Boa Tarde Sra. secret=E1ria Nelly: As escolas que tiveram alunos premiados na OBM est=E3o me perguntando se receber=E3o essas medalhas pelo correio. N=E3o sei responder. Outra coisa, uma das escolas est=E1 pensando e fazer a premia=E7=E3o com as medalhas na formatura da sua 8=AA s=E9rie. Ocorreu-me que essa =E9 uma= =20 excelente oportunidade de promo=E7=E3o para a OBM! Todos os familiares reunidos, amigos, imprensa local, etc. Talvez seja interessante sugerir que isso seja feito em todos os lugares. Por hora =E9 s=F3, obrigado, Arconcher(Jundia=ED) From owner-obm-rj@saci.mat.puc-rio.br Wed Dec 9 13:54:05 1998 Received: by saci.mat.puc-rio.br (AIX 3.2/UCB 5.64/4.03) id AA30960; Wed, 9 Dec 1998 13:53:45 -0200 Received: from mail.iis.com.br by saci.mat.puc-rio.br (AIX 3.2/UCB 5.64/4.03) id AA34537; Wed, 9 Dec 1998 13:53:18 -0200 Received: from default (rio-as6-tty10.iis.com.br [200.202.97.106]) by mail.iis.com.br (8.8.7/8.8.14) with SMTP id NAA00744 for ; Wed, 9 Dec 1998 13:53:13 -0200 From: "Marcio" To: Subject: Ultimo problema da ultima prova Date: Fri, 18 Dec 1998 13:55:44 -0300 Message-Id: <01be2aa7$40e1d6c0$LocalHost@default> Mime-Version: 1.0 Content-Type: multipart/alternative; boundary="----=_NextPart_000_0011_01BE2A8E.1B949EC0" X-Priority: 3 X-Msmail-Priority: Normal X-Mailer: Microsoft Outlook Express 4.71.1712.3 X-Mimeole: Produced By Microsoft MimeOLE V4.71.1712.3 Sender: owner-obm-rj@saci.mat.puc-rio.br Precedence: bulk Reply-To: obm-rj@saci.mat.puc-rio.br Status: RO X-Status: X-Keywords: X-UID: 56 This is a multi-part message in MIME format. ------=_NextPart_000_0011_01BE2A8E.1B949EC0 Content-Type: text/plain; charset="iso-8859-1" Content-Transfer-Encoding: quoted-printable Alguem aqui tem a solucao do ultimo problema da ultima prova da OBM = (Terceira fase - problema 6). Ha algum tempo, o Nicolau colocou aqui na = lista a resposta mas eu ainda nao faco a menor ideia de como se chegar a = ela.=20 Por favor, coloquem uma solucao na lista (tambem apreciaria as = solucoes completas das outras questoes da ultima prova). Grato, Marcio ------=_NextPart_000_0011_01BE2A8E.1B949EC0 Content-Type: text/html; charset="iso-8859-1" Content-Transfer-Encoding: quoted-printable
    Alguem=20 aqui tem a solucao do ultimo problema da ultima prova da OBM (Terceira = fase -=20 problema 6). Ha algum tempo, o Nicolau colocou aqui na lista a resposta = mas eu=20 ainda nao faco a menor ideia de como se chegar a ela. =
 
    Por=20 favor, coloquem uma solucao na lista (tambem apreciaria as solucoes = completas=20 das outras questoes da ultima prova).
 
   =20 Grato,
   =20 Marcio
------=_NextPart_000_0011_01BE2A8E.1B949EC0-- From owner-obm-rj@saci.mat.puc-rio.br Wed Dec 9 17:19:11 1998 Received: by saci.mat.puc-rio.br (AIX 3.2/UCB 5.64/4.03) id AA21183; Wed, 9 Dec 1998 17:19:07 -0200 Received: from boto.mat.puc-rio.br by saci.mat.puc-rio.br (AIX 3.2/UCB 5.64/4.03) id AA30394; Wed, 9 Dec 1998 17:18:47 -0200 Received: (from nicolau@localhost) by boto.mat.puc-rio.br (8.8.7/8.8.7) id RAA14364 for obm-rj@saci.mat.puc-rio.br; Wed, 9 Dec 1998 17:18:45 -0200 Date: Wed, 9 Dec 1998 17:18:45 -0200 From: "Nicolau C. Saldanha" Message-Id: <199812091918.RAA14364@boto.mat.puc-rio.br> To: obm-rj@saci.mat.puc-rio.br Subject: Re: Ultimo problema da ultima prova Sender: owner-obm-rj@saci.mat.puc-rio.br Precedence: bulk Reply-To: obm-rj@saci.mat.puc-rio.br Status: RO X-Status: X-Keywords: X-UID: 57 > Alguem aqui tem a solucao do ultimo problema da ultima prova da OBM > (Terceira fase - problema 6). Ha algum tempo, o Nicolau colocou aqui na > lista a resposta mas eu ainda nao faco a menor ideia de como se chegar a > ela. A resposta eh 9 (interpretando que os matematicos andem juntos). Chamando o ponto de partida de rua 0, eles andam ateh a rua 1, depois ateh a rua -2, depois ateh a rua 4 e assim por diante com as potencias de -2. O pior caso possivel ocorre se a rua procurada for a 2, -3, 5, -9,.. ou em geral (-2)^n + (-1)^n. Neste caso os matematicos andam 2*(2^0 + 2^1 + ... + 2^n + 2^(n+1)) + 2^n + 1 = 2^(n+3) + 2^n - 1. Dividindo isto por 2^n + 1 (a distancia) obtemos um numero um pouco menor do que 9. Agora pense um pouco sobre por que 9 eh o melhor numero possivel, depois eu escrevo a solucao desta outra parte. []s, Nicolau From owner-obm-rj@saci.mat.puc-rio.br Wed Dec 9 17:49:58 1998 Received: by saci.mat.puc-rio.br (AIX 3.2/UCB 5.64/4.03) id AA21139; Wed, 9 Dec 1998 17:49:42 -0200 Received: from mail.iis.com.br by saci.mat.puc-rio.br (AIX 3.2/UCB 5.64/4.03) id AA15501; Wed, 9 Dec 1998 17:49:17 -0200 Received: from default (rio-tc0-tty45.iis.com.br [200.202.98.45]) by mail.iis.com.br (8.8.7/8.8.14) with SMTP id RAA24552 for ; Wed, 9 Dec 1998 17:49:11 -0200 From: "Marcio" To: Subject: Re: Ultimo problema da ultima prova Date: Fri, 18 Dec 1998 17:53:01 -0300 Message-Id: <01be2ac8$66826540$2d62cac8@default> Mime-Version: 1.0 Content-Type: text/plain; charset="iso-8859-1" Content-Transfer-Encoding: 7bit X-Priority: 3 X-Msmail-Priority: Normal X-Mailer: Microsoft Outlook Express 4.71.1712.3 X-Mimeole: Produced By Microsoft MimeOLE V4.71.1712.3 Sender: owner-obm-rj@saci.mat.puc-rio.br Precedence: bulk Reply-To: obm-rj@saci.mat.puc-rio.br Status: RO X-Status: X-Keywords: X-UID: 58 Vou pensar mais sobre esse final. E quanto a pontuacao. Como foram distribuidos os pontos? -----Original Message----- From: Nicolau C. Saldanha To: obm-rj@saci.mat.puc-rio.br Date: Quarta-feira, 9 de Dezembro de 1998 16:19 Subject: Re: Ultimo problema da ultima prova >> Alguem aqui tem a solucao do ultimo problema da ultima prova da OBM >> (Terceira fase - problema 6). Ha algum tempo, o Nicolau colocou aqui na >> lista a resposta mas eu ainda nao faco a menor ideia de como se chegar a >> ela. > >A resposta eh 9 (interpretando que os matematicos andem juntos). >Chamando o ponto de partida de rua 0, eles andam ateh a rua 1, >depois ateh a rua -2, depois ateh a rua 4 e assim por diante >com as potencias de -2. O pior caso possivel ocorre se a rua >procurada for a 2, -3, 5, -9,.. ou em geral (-2)^n + (-1)^n. >Neste caso os matematicos andam >2*(2^0 + 2^1 + ... + 2^n + 2^(n+1)) + 2^n + 1 = >2^(n+3) + 2^n - 1. >Dividindo isto por 2^n + 1 (a distancia) obtemos um numero >um pouco menor do que 9. > >Agora pense um pouco sobre por que 9 eh o melhor numero possivel, >depois eu escrevo a solucao desta outra parte. > >[]s, Nicolau > > From owner-obm-rj@saci.mat.puc-rio.br Wed Dec 9 23:23:41 1998 Received: by saci.mat.puc-rio.br (AIX 3.2/UCB 5.64/4.03) id AA29728; Wed, 9 Dec 1998 23:23:37 -0200 Received: from euler.impa.br by saci.mat.puc-rio.br (AIX 3.2/UCB 5.64/4.03) id AA08476; Wed, 9 Dec 1998 23:23:13 -0200 Received: from [147.65.11.5] (dial05.impa.br [147.65.11.5]) by Euler.impa.br (8.8.6/8.8.6) with ESMTP id XAA12798 for ; Wed, 9 Dec 1998 23:23:10 -0200 (EDT) Message-Id: In-Reply-To: <01be2aa7$40e1d6c0$LocalHost@default> Mime-Version: 1.0 Content-Type: text/plain; charset="us-ascii" Date: Wed, 9 Dec 1998 22:27:24 -0300 To: obm-rj@saci.mat.puc-rio.br From: Eduardo Wagner Subject: Re: Ultimo problema da ultima prova Sender: owner-obm-rj@saci.mat.puc-rio.br Precedence: bulk Reply-To: obm-rj@saci.mat.puc-rio.br Status: RO X-Status: X-Keywords: X-UID: 59 > Alguem aqui tem a solucao do ultimo problema da ultima prova da >OBM (Terceira fase - problema 6). Ha algum tempo, o Nicolau colocou aqui >na lista a resposta mas eu ainda nao faco a menor ideia de como se chegar >a ela. Por favor, coloquem uma solucao na lista (tambem apreciaria >as solucoes completas das outras questoes da ultima prova). Grato, > Marcio Prezado colega: Em breve publicaremos as solucoes dos problemas da terceira fase da OBM nos tres niveis. Aguarde. Abraco, Wagner. From owner-obm-rj@saci.mat.puc-rio.br Wed Dec 16 18:23:01 1998 Received: by saci.mat.puc-rio.br (AIX 3.2/UCB 5.64/4.03) id AA18593; Wed, 16 Dec 1998 18:21:55 -0200 Received: from euler.impa.br by saci.mat.puc-rio.br (AIX 3.2/UCB 5.64/4.03) id AA20381; Wed, 16 Dec 1998 18:21:42 -0200 Received: from Gauss.impa.br (gauss.impa.br [147.65.4.1]) by Euler.impa.br (8.8.6/8.8.6) with ESMTP id PAA28430 for ; Wed, 16 Dec 1998 15:44:51 -0200 (EDT) From: Carlos Gustavo Tamm de Araujo Moreira Received: by Gauss.impa.br (8.8.8) id PAA13791; Wed, 16 Dec 1998 15:43:55 -0200 (EDT) Message-Id: <199812161743.PAA13791@Gauss.impa.br> Subject: Re: problemas (fwd) To: obm-rj@saci.mat.puc-rio.br Date: Wed, 16 Dec 1998 15:43:54 -0200 (EDT) X-Mailer: ELM [version 2.4 PL25] Mime-Version: 1.0 Content-Type: text/plain; charset=US-ASCII Content-Transfer-Encoding: quoted-printable X-Mime-Autoconverted: from 8bit to quoted-printable by Euler.impa.br id PAA28430 Sender: owner-obm-rj@saci.mat.puc-rio.br Precedence: bulk Reply-To: obm-rj@saci.mat.puc-rio.br Status: RO X-Status: X-Keywords: X-UID: 60 Forwarded message: >From gugu Tue Dec 15 20:03:28 1998 Subject: Re: problemas To: bene@digi.com.br (benedito) Date: Tue, 15 Dec 1998 20:03:28 -0200 (EDT) In-Reply-To: <3.0.3.32.19981213154521.007987d0@digi.com.br> from "benedit= o" at Dec 13, 98 03:45:21 pm X-Mailer: ELM [version 2.4 PL25] MIME-Version: 1.0 Content-Type: text/plain; charset=3DUS-ASCII Content-Transfer-Encoding: 7bit Content-Length: 2059 =20 Caro Benedito, Vamos la': Problema 1:Suponha que nao.Entao para n grande,se houver n^2+n carocos de feijao na mesa o primeiro jogador ganha,isto e',existe k quadrado perfeit= o menor ou igual a n^2+n(e portanto menor ou igual a n^2) tal que o primeir= o jogador,retirando k carocos e devolvendo n^2+n-k (que vale pelo menos n) carocos para o segundo jogador,este segundo jogador estara' perdido,mas i= sso mostra que quem comeca com n^2+n-k carocos perde,mas n^2+n-k>=3Dn pode se= r arbitrariamente grande,cqd. Problema 2:Os racionais sao um conjunto enumeravel,digamos Q=3D{r_1,r_2,r_3,...}.O jogador A pode escolher sempre o intervalo I(2k-1= ) de modo que o racional r_k nao pertenca a ele,e no fim nenhum racional r_k c= om k natural(e portanto nenhum racional) estara' na intersecao de todos os intervalos In,que portanto deve ser irracional.\ Abracos,\ Gugu > >Caro Gustavo,=20 >Voc=EA pode dar uma a solu=E7=E3o para cada um dos dois problemas abaixo= ? > >Problema1 >Considere o seguinte jogo para duas pessoas. Um n=FAmero M de caro=E7o= s de >feij=E3o est=E3o sobre uma mesa. Dois jogadores jogam alternadamente. Um >movimento consiste em retirar da mesa um n=FAmero n de caro=E7os de feij= =E3o, >onde n =E9 um quadrado de um n=FAmero inteiro positivo. O jogador que = n=E3o for >capaz de fazer um movimento perde. Prove que existe infinitas situa=E7=F5= es >iniciais onde o segundo jogador vence, n=E3o importando como o primeiro >jogador fa=E7a sua jogada. > >Problema 2 >Considere o seguinte jogo para dois jogadores A e B.O jogador A esco= lhe >um intervalo fechado arbitr=E1rio I1 de comprimento menor do que 1. E= m >seguida, o jogador B escolhe um intervalo fechado I2 contido em I1 e = de >comprimento menor do que 1/2. Ent=E3o A escolhe um intervalo fechado = I3 >contido em I2 de comprimento menor do que 1/3, e assim por diante. >Se a interse=E7=E3o de todos esse intervalos I1, I2, I3, ... for irraci= onal, o >jogador A vence a partida. Caso contr=E1rio, vence B. Mostre que A sempr= e >pode ganhar, independente de como B jogue. > >Obrigado antecipadamente. >Benedito > > From owner-obm-rj@saci.mat.puc-rio.br Sat Dec 19 00:44:05 1998 Received: by saci.mat.puc-rio.br (AIX 3.2/UCB 5.64/4.03) id AA19395; Sat, 19 Dec 1998 00:43:38 -0200 Received: from srv4-poa.zaz.com.br by saci.mat.puc-rio.br (AIX 3.2/UCB 5.64/4.03) id AA18111; Sat, 19 Dec 1998 00:43:24 -0200 Received: from default (dl-tnt1-179.poa.zaz.com.br [200.248.211.179]) by srv4-poa.nutecnet.com.br (8.9.1/8.7.3) with ESMTP id AAA08675 for ; Sat, 19 Dec 1998 00:43:23 -0200 (GMT+2) Message-Id: <199812190243.AAA08675@srv4-poa.nutecnet.com.br> From: "Lucas" To: Subject: =?ISO-8859-1?Q?Vote_En=E9as=2C_56?= Date: Sun, 27 Sep 1998 01:33:24 -0300 X-Msmail-Priority: Normal X-Priority: 3 X-Mailer: Microsoft Internet Mail 4.70.1161 Mime-Version: 1.0 Content-Type: text/plain; charset=ISO-8859-1 Content-Transfer-Encoding: quoted-printable X-Mime-Autoconverted: from 8bit to quoted-printable by srv4-poa.nutecnet.com.br id AAA08675 Sender: owner-obm-rj@saci.mat.puc-rio.br Precedence: bulk Reply-To: obm-rj@saci.mat.puc-rio.br Status: RO X-Status: X-Keywords: X-UID: 61 Oi gente boa! A=ED v=E3o algumas tarefas pra voc=EAs. Desafio!! Continue a letra: "=C0s vezes parecia=20 Que de tanto acreditar Em tudo o que ach=E1vamos t=E3o certo, Ter=EDamos um mundo inteiro=20 E at=E9 um pouco mais, Far=EDamos floresta do deserto E diamantes de peda=E7os de vidro. " ............................................... Mate a cobra e mostre o porrete!: 1. Um pai morreu e deixou uma heran=E7a para que seus dois filhos a disputassem. No testamento ele disse: "...dever=E1 ser realizada uma corr= ida de cavalo entre meus dois filhos; aquele que chegar por =FAltimo na est=E1= tua de S=E3o Lucas ser=E1 o vencedor". Ent=E3o um dos filhos montou em seu ca= valo e o outro fez o mesmo com o seu cavalo. Por=E9m, eles iam t=E3o devagar (pois queriam receber a grana, chegando em =FAltimo) que nunca chegariam ao seu destino. No meio do caminho, por sorte, eles encontram o s=E1bio Chico Xa= vier e decidiram lhe pedir ajuda. Ent=E3o ele cochichou uma frase no ouvido de cada um dos irm=E3os e, em menos de cinco segundos, os dois montaram nos cavalos e disputaram a mais veloz das corridas. Qual foi a s=E1bia frase = dita por Chico Xavier, sendo que ela deve respeitar a condi=E7=E3o do testamen= to? 2. Temos tr=EAs jarras de capacidades 5, 6 e 8 litros cada. As duas prime= iras est=E3o cheios, enquanto que restante est=E1 completamente vazio. Como de= vemos proceder a fim de obtermos um =FAnico litro na jarra de capacidade 8 litr= os, se n=E3o temos as demarca=E7=F5es (medidas) de litro? Espera as suas manifesta=E7=F5es! Lucas P. M. From owner-obm-rj@saci.mat.puc-rio.br Sat Dec 19 01:15:24 1998 Received: by saci.mat.puc-rio.br (AIX 3.2/UCB 5.64/4.03) id AA18148; Sat, 19 Dec 1998 01:15:17 -0200 Received: from [200.248.17.2] by saci.mat.puc-rio.br (AIX 3.2/UCB 5.64/4.03) id AA22752; Sat, 19 Dec 1998 01:15:06 -0200 Received: from server [200.248.17.36] by sinos.net [200.248.17.2] with SMTP (MDaemon.v2.7.SP4.R) for ; Sat, 19 Dec 1998 01:13:42 -0200 From: "Benjamin Hinrichs" To: Subject: =?iso-8859-1?Q?Re:_Vote_En=E9as=2C_56?= Date: Sat, 19 Dec 1998 01:10:02 -0200 Message-Id: <01be2afd$1d75bb60$2411f8c8@server> Mime-Version: 1.0 Content-Type: multipart/alternative; boundary="----=_NextPart_000_000C_01BE2AEC.59ECEB60" X-Priority: 3 X-Msmail-Priority: Normal X-Mailer: Microsoft Outlook Express 4.71.1712.3 X-Mimeole: Produced By Microsoft MimeOLE V4.71.1712.3 X-Mdaemon-Deliver-To: obm-rj@saci.mat.puc-rio.br X-Return-Path: hinsoft@sinos.net Sender: owner-obm-rj@saci.mat.puc-rio.br Precedence: bulk Reply-To: obm-rj@saci.mat.puc-rio.br Status: RO X-Status: X-Keywords: X-UID: 62 This is a multi-part message in MIME format. ------=_NextPart_000_000C_01BE2AEC.59ECEB60 Content-Type: text/plain; charset="iso-8859-1" Content-Transfer-Encoding: quoted-printable Lucas, consegui resolver apenas seu segundo problema, que ali=E1s n=E3o =E9 = muito dif=EDcil. Basta derramar o jarro de 6 litros no de 8 litros, = derramar o de 5 no de 6 e do de 8 no de 5 at=E9 que quase transborde. = Vai restar um litro no jarro de 8 litros, como pedia a quest=E3o. Benjamin Hinrichs ------=_NextPart_000_000C_01BE2AEC.59ECEB60 Content-Type: text/html; charset="iso-8859-1" Content-Transfer-Encoding: quoted-printable
Lucas,
 
consegui resolver apenas seu segundo problema, que=20 aliás não é muito difícil. Basta derramar o = jarro de=20 6 litros no de 8 litros, derramar o de 5 no de 6 e do de 8 no de 5 = até=20 que quase transborde. Vai restar um litro no jarro de 8 litros, como = pedia a=20 questão.
 
 
Benjamin=20 Hinrichs
 
------=_NextPart_000_000C_01BE2AEC.59ECEB60-- From owner-obm-rj@saci.mat.puc-rio.br Sun Dec 20 00:37:45 1998 Received: by saci.mat.puc-rio.br (AIX 3.2/UCB 5.64/4.03) id AA22726; Sun, 20 Dec 1998 00:37:41 -0200 Received: from srv4-poa.zaz.com.br by saci.mat.puc-rio.br (AIX 3.2/UCB 5.64/4.03) id AA16578; Sun, 20 Dec 1998 00:37:27 -0200 Received: from default (dl-pm3-14.poa.zaz.com.br [200.248.210.78]) by srv4-poa.nutecnet.com.br (8.9.1/8.7.3) with ESMTP id AAA17354 for ; Sun, 20 Dec 1998 00:37:24 -0200 (GMT+2) Message-Id: <199812200237.AAA17354@srv4-poa.nutecnet.com.br> From: "Lucas" To: Subject: Muito bem, cara! Date: Sun, 27 Sep 1998 04:49:38 -0300 X-Msmail-Priority: Normal X-Priority: 3 X-Mailer: Microsoft Internet Mail 4.70.1161 Mime-Version: 1.0 Content-Type: text/plain; charset=ISO-8859-1 Content-Transfer-Encoding: quoted-printable X-Mime-Autoconverted: from 8bit to quoted-printable by srv4-poa.nutecnet.com.br id AAA17354 Sender: owner-obm-rj@saci.mat.puc-rio.br Precedence: bulk Reply-To: obm-rj@saci.mat.puc-rio.br Status: RO X-Status: X-Keywords: X-UID: 63 =C9, Benjamin, realmente existem quest=F5es mais dif=EDceis do que esta, = mas mesmo assim, meus parab=E9ns! Voc=EA matou a cobra e mostrou o porrete! B= em, tente os outros desafios; s=E3o bem legais. A=ED v=E3o mais alguns probleminhas: 3. Duas torneiras enchem completamente um tanque em 3 horas e 45 minutos. Uma delas, quando ligada s=F3, completa o tanque 4 horas a menos que a ou= tra. Quanto tempo cada uma delas leva para encher o tanque, quando ligadas sozinhas ? 4. Na biblioteca do col=E9gio, um estudante contava a seu colega: " Imagi= ne s=F3, perdi, ontem, uma c=E9dula de 100 reais e s=F3 hoje a encontrei, de= ntro do dicion=E1rio entre as p=E1ginas 99 e 100. Por=E9m, como o colega era bem = esperto, logo viu que o "amigo" mentia. Como? 5. Temos a soma ABCD BCD =20 CD D que resulta em BAAAC. Mostre o valor de cada letra. Lucas P. M. From owner-obm-rj@saci.mat.puc-rio.br Sun Dec 20 16:25:06 1998 Received: by saci.mat.puc-rio.br (AIX 3.2/UCB 5.64/4.03) id AA05278; Sun, 20 Dec 1998 16:25:02 -0200 Received: from [200.248.17.2] by saci.mat.puc-rio.br (AIX 3.2/UCB 5.64/4.03) id AA22682; Sun, 20 Dec 1998 16:24:46 -0200 Received: from server [200.248.17.37] by sinos.net [200.248.17.2] with SMTP (MDaemon.v2.7.SP4.R) for ; Sun, 20 Dec 1998 16:22:43 -0200 From: "Benjamin Hinrichs" To: Subject: Lucas Date: Sun, 20 Dec 1998 16:20:13 -0200 Message-Id: <01be2c45$63115b40$0100007f@localhost> Mime-Version: 1.0 Content-Type: text/plain; charset="iso-8859-1" Content-Transfer-Encoding: 8bit X-Priority: 3 X-Msmail-Priority: Normal X-Mailer: Microsoft Outlook Express 4.71.1712.3 X-Mimeole: Produced By Microsoft MimeOLE V4.71.1712.3 X-Mdaemon-Deliver-To: obm-rj@saci.mat.puc-rio.br X-Return-Path: hinsoft@sinos.net Sender: owner-obm-rj@saci.mat.puc-rio.br Precedence: bulk Reply-To: obm-rj@saci.mat.puc-rio.br Status: RO X-Status: X-Keywords: X-UID: 64 Lucas, >3. Duas torneiras enchem completamente um tanque em 3 horas e 45 minutos. >Uma delas, quando ligada só, completa o tanque 4 horas a menos que a outra. >Quanto tempo cada uma delas leva para encher o tanque, quando ligadas >sozinhas ? Odeio perguntas de torneiras. Além de pouco criativas, pedem muito pensamento e eu também não sei as fazer. Peço ajuda. >4. Na biblioteca do colégio, um estudante contava a seu colega: " Imagine >só, perdi, ontem, uma cédula de 100 reais e só hoje a encontrei, dentro do >dicionário entre as páginas 99 e 100. Porém, como o colega era bem esperto, >logo viu que o "amigo" mentia. Como? Esta é fácil. Eu li na Super Interessante. É um costume colocar as página impares à direita. Logo ele mentiu porque a página 99 fica à direita e a página 100 fica do outro lado, ou seja, são uma folha só. Ele deveria ter dito entre as páginas 98 e 99 ou entre 100 e 101. Diga ao colega para que procure amigos menos mentirosos. >5. Temos a soma ABCD > BCD > CD > D > > que resulta em BAAAC. > >Mostre o valor de cada letra. Provavelmente não é bem esta resposta a que vc queria, porém é a única que eu achei. É que é domingo e não estou disposto a pensar. Deixo esta para os demais leitores. A minha resposta é A=B=C=D=0 E aqui vai o meu problema. Em um pote de ouro do rei da Matelândia existiam 9 bolas de ouro de igual tamanho e igual peso. Veio então o malvado ladrão e roubou uma das 9 bolas. Para que não ficasse tão evidente o roubo e para que ele tivesse tempo para fugir, ele projetou uma bola de chumbo (que é mais leve que as de ouro) e a pintou. Mas o rei notou naquela mesma manhã o roubo, pois a tinta dourada ainda estava fresca. Eles prenderam o sujeito que roubou a bola de ouro e pegaram a bola de volta. Mas no meio tempo a tinta da bola de chumbo já havia secado e o rei havia jogado ela de volta ao pote de ouro. Veio então um matemático com uma balança e disse. Medindo apenas duas vezes uma certa quantidade de bolas, posso descobrir qual é a bola falsa. Como? Benjamin Hinrichs From owner-obm-rj@saci.mat.puc-rio.br Sun Dec 20 16:57:23 1998 Received: by saci.mat.puc-rio.br (AIX 3.2/UCB 5.64/4.03) id AA20175; Sun, 20 Dec 1998 16:57:21 -0200 Received: from [200.248.17.2] by saci.mat.puc-rio.br (AIX 3.2/UCB 5.64/4.03) id AA09931; Sun, 20 Dec 1998 16:57:07 -0200 Received: from server [200.248.17.41] by sinos.net [200.248.17.2] with SMTP (MDaemon.v2.7.SP4.R) for ; Sun, 20 Dec 1998 16:56:11 -0200 From: "Benjamin Hinrichs" To: Subject: Lucas Date: Sun, 20 Dec 1998 16:53:41 -0200 Message-Id: <01be2c4a$0fbe89e0$0100007f@localhost> Mime-Version: 1.0 Content-Type: text/plain; charset="iso-8859-1" Content-Transfer-Encoding: 8bit X-Priority: 3 X-Msmail-Priority: Normal X-Mailer: Microsoft Outlook Express 4.71.1712.3 X-Mimeole: Produced By Microsoft MimeOLE V4.71.1712.3 X-Mdaemon-Deliver-To: obm-rj@saci.mat.puc-rio.br X-Return-Path: hinsoft@sinos.net Sender: owner-obm-rj@saci.mat.puc-rio.br Precedence: bulk Reply-To: obm-rj@saci.mat.puc-rio.br Status: RO X-Status: X-Keywords: X-UID: 65 Lucas, >3. Duas torneiras enchem completamente um tanque em 3 horas e 45 minutos. >Uma delas, quando ligada só, completa o tanque 4 horas a menos que a outra. >Quanto tempo cada uma delas leva para encher o tanque, quando ligadas >sozinhas ? Odeio perguntas de torneiras. Além de pouco criativas, pedem muito pensamento e eu também não sei as fazer. Peço ajuda. >4. Na biblioteca do colégio, um estudante contava a seu colega: " Imagine >só, perdi, ontem, uma cédula de 100 reais e só hoje a encontrei, dentro do >dicionário entre as páginas 99 e 100. Porém, como o colega era bem esperto, >logo viu que o "amigo" mentia. Como? Esta é fácil. Eu li na Super Interessante. É um costume colocar as página impares à direita. Logo ele mentiu porque a página 99 fica à direita e a página 100 fica do outro lado, ou seja, são uma folha só. Ele deveria ter dito entre as páginas 98 e 99 ou entre 100 e 101. Diga ao colega para que procure amigos menos mentirosos. >5. Temos a soma ABCD > BCD > CD > D > > que resulta em BAAAC. > >Mostre o valor de cada letra. Provavelmente não é bem esta resposta a que vc queria, porém é a única que eu achei. É que é domingo e não estou disposto a pensar. Deixo esta para os demais leitores. A minha resposta é A=B=C=D=0 E aqui vai o meu problema. Em um pote de ouro do rei da Matelândia existiam 9 bolas de ouro de igual tamanho e igual peso. Veio então o malvado ladrão e roubou uma das 9 bolas. Para que não ficasse tão evidente o roubo e para que ele tivesse tempo para fugir, ele projetou uma bola de chumbo (que é mais leve que as de ouro) e a pintou. Mas o rei notou naquela mesma manhã o roubo, pois a tinta dourada ainda estava fresca. Eles prenderam o sujeito que roubou a bola de ouro e pegaram a bola de volta. Mas no meio tempo a tinta da bola de chumbo já havia secado e o rei havia jogado ela de volta ao pote de ouro. Veio então um matemático com uma balança e disse. Medindo apenas duas vezes uma certa quantidade de bolas, posso descobrir qual é a bola falsa. Como? Benjamin Hinrichs From owner-obm-rj@saci.mat.puc-rio.br Sun Dec 20 20:17:17 1998 Received: by saci.mat.puc-rio.br (AIX 3.2/UCB 5.64/4.03) id AA20259; Sun, 20 Dec 1998 20:17:10 -0200 Received: from gb6-rio.rio.nutecnet.com.br by saci.mat.puc-rio.br (AIX 3.2/UCB 5.64/4.03) id AA05151; Sun, 20 Dec 1998 20:16:59 -0200 Received: from joaodias (dl1081-rio.rio.nutecnet.com.br [200.240.23.81]) by gb6-rio.rio.nutecnet.com.br (8.8.5/SCA-6.6) with SMTP id UAA08989 for ; Sun, 20 Dec 1998 20:19:16 -0200 (BRV) Message-Id: <199812202219.UAA08989@gb6-rio.rio.nutecnet.com.br> X-Sender: joaodias@pop.rio.nutecnet.com.br X-Mailer: QUALCOMM Windows Eudora Pro Version 4.0 Date: Sun, 20 Dec 1998 20:07:39 -0200 To: obm-rj@saci.mat.puc-rio.br From: Joao Dias Subject: Bolas de ouro In-Reply-To: <01be2c4a$0fbe89e0$0100007f@localhost> Mime-Version: 1.0 Content-Type: text/plain; charset="iso-8859-1" Content-Transfer-Encoding: quoted-printable Sender: owner-obm-rj@saci.mat.puc-rio.br Precedence: bulk Reply-To: obm-rj@saci.mat.puc-rio.br Status: RO X-Status: X-Keywords: X-UID: 66 E aqui vai o meu problema. Em um pote de ouro do rei da Matel=E2ndia= existiam 9 bolas de ouro de igual tamanho e igual peso. Veio ent=E3o o malvado ladr= =E3o e roubou uma das 9 bolas. Para que n=E3o ficasse t=E3o evidente o roubo e para= que ele tivesse tempo para fugir, ele projetou uma bola de chumbo (que =E9 mais leve que as de ouro) e a pintou. Mas o rei notou naquela mesma manh=E3 o roubo, pois a tinta dourada ainda estava fresca. Eles prenderam o sujeito que roubou a bola de ouro e pegaram a bola de volta. Mas no meio tempo a tinta da bola de chumbo j=E1 havia secado e o rei havia jogado ela de volta= ao pote de ouro. Veio ent=E3o um matem=E1tico com uma balan=E7a e disse.= Medindo apenas duas vezes uma certa quantidade de bolas, posso descobrir qual =E9 a bola falsa. Como? Benjamin Hinrichs Primeiro o matem=E1tico separou as bolas em 3 grupos de 3 bolas cada. Ap=F3s isso, ele pesou o primeiro grupo com o segundo. Caso a balan=E7a se desequilibre, a balan=E7a indicar=E1 qual o grupo com a bola de chumbo. Caso isso n=E3o aconte=E7a, o terceiro grupo tem a bola de chumbo. Ap=F3s isso, pegamos as 3 bolas do grupo com a bola falsa e pesamos duas bolas quaisquer entre si. Caso a balan=E7a se desequilibre, temos a resposta. Em caso contr=E1rio, a bola que n=E3o pesamos =E9 a resposta. Este problema apresenta uma pequena varia=E7=E3o no livro "O homem que calculava", de Malba Tahan. No problema do livro s=E3o 8 p=E9rolas que devem ser pesadas. []s, F=E1bio Dias From owner-obm-rj@saci.mat.puc-rio.br Sun Dec 20 21:12:13 1998 Received: by saci.mat.puc-rio.br (AIX 3.2/UCB 5.64/4.03) id AA09799; Sun, 20 Dec 1998 21:12:11 -0200 Received: from mail.iis.com.br by saci.mat.puc-rio.br (AIX 3.2/UCB 5.64/4.03) id AA16451; Sun, 20 Dec 1998 21:11:58 -0200 Received: from default (rio-as5-tty07.iis.com.br [200.202.97.87]) by mail.iis.com.br (8.8.7/8.8.14) with SMTP id VAA20830 for ; Sun, 20 Dec 1998 21:11:54 -0200 From: "Marcio" To: Subject: Re: Lucas Date: Tue, 29 Dec 1998 21:15:25 -0300 Message-Id: <01be3389$7f9ec400$5761cac8@default> Mime-Version: 1.0 Content-Type: text/plain; charset="iso-8859-1" X-Priority: 3 X-Msmail-Priority: Normal X-Mailer: Microsoft Outlook Express 4.71.1712.3 X-Mimeole: Produced By Microsoft MimeOLE V4.71.1712.3 Content-Transfer-Encoding: quoted-printable X-Mime-Autoconverted: from 8bit to quoted-printable by mail.iis.com.br id VAA20830 Sender: owner-obm-rj@saci.mat.puc-rio.br Precedence: bulk Reply-To: obm-rj@saci.mat.puc-rio.br Status: RO X-Status: X-Keywords: X-UID: 67 Do mesmo jeito que voce odeia problemas de torneiras, eu nao gosto mu= ito de problemas de pesagens desse tipo.. costumam exigir muito de mim :) (e = eu em geral nao resolvo). Ha alguns desses problemas no livro "The USSR Olympiad Problem Book". Aqui vai um (=E9 apenas uma versao um pouco maior desse problema): Ha uma moeda falsa entre 1000 moedas similares. Nao se sabe se a moed= a eh mais leve ou mais pesada do que a moeda verdadeira. Qual =E9 o menor n= =FAmero de pesagens, numa balanca de bracos, necessario para descobrir (na pior d= as hipoteses) qual eh a moeda falsa e determinar se ela eh mais leve ou mais pesada que as outras. Se nao me engano, ha uma generalizacao desse problema na Eureka no. 3 (nao sei se eh desse ou de outro, quando ja eh sabido que a moeda falsa e= h mais leve que as outras). Tem um outro, que eu acho que =E9 o problema classico desse tipo que = eh o mesmo problema anterior porem com 12 moedas. Determinar como descobrir a moeda falsa (e se ela eh mais leve ou mais pesada) com apenas 3 pesagens.= .. Se voces quiserem, eu posso mandar outros do tipo. Mas talvez, no fundo, os outros sejam iguais a esses. Abracos, Marcio -----Original Message----- From: Benjamin Hinrichs To: obm-rj@saci.mat.puc-rio.br Date: Domingo, 20 de Dezembro de 1998 15:57 Subject: Lucas >Lucas, > >>3. Duas torneiras enchem completamente um tanque em 3 horas e 45 minuto= s. >>Uma delas, quando ligada s=F3, completa o tanque 4 horas a menos que a outra. >>Quanto tempo cada uma delas leva para encher o tanque, quando ligadas >>sozinhas ? > >Odeio perguntas de torneiras. Al=E9m de pouco criativas, pedem muito >pensamento e eu tamb=E9m n=E3o sei as fazer. Pe=E7o ajuda. > >>4. Na biblioteca do col=E9gio, um estudante contava a seu colega: " Ima= gine >>s=F3, perdi, ontem, uma c=E9dula de 100 reais e s=F3 hoje a encontrei, = dentro do >>dicion=E1rio entre as p=E1ginas 99 e 100. Por=E9m, como o colega era be= m esperto, >>logo viu que o "amigo" mentia. Como? > > >Esta =E9 f=E1cil. Eu li na Super Interessante. =C9 um costume colocar as= p=E1gina >impares =E0 direita. Logo ele mentiu porque a p=E1gina 99 fica =E0 direi= ta e a >p=E1gina 100 fica do outro lado, ou seja, s=E3o uma folha s=F3. Ele deve= ria ter >dito entre as p=E1ginas 98 e 99 ou entre 100 e 101. >Diga ao colega para que procure amigos menos mentirosos. > >>5. Temos a soma ABCD >> BCD >> CD >> D >> >> que resulta em BAAAC. >> >>Mostre o valor de cada letra. > > >Provavelmente n=E3o =E9 bem esta resposta a que vc queria, por=E9m =E9 a= =FAnica que >eu achei. =C9 que =E9 domingo e n=E3o estou disposto a pensar. Deixo est= a para os >demais leitores. A minha resposta =E9 A=3DB=3DC=3DD=3D0 > > >E aqui vai o meu problema. Em um pote de ouro do rei da Matel=E2ndia exi= stiam >9 bolas de ouro de igual tamanho e igual peso. Veio ent=E3o o malvado la= dr=E3o e >roubou uma das 9 bolas. Para que n=E3o ficasse t=E3o evidente o roubo e = para que >ele tivesse tempo para fugir, ele projetou uma bola de chumbo (que =E9 m= ais >leve que as de ouro) e a pintou. Mas o rei notou naquela mesma manh=E3 o >roubo, pois a tinta dourada ainda estava fresca. Eles prenderam o sujeit= o >que roubou a bola de ouro e pegaram a bola de volta. Mas no meio tempo a >tinta da bola de chumbo j=E1 havia secado e o rei havia jogado ela de vo= lta ao >pote de ouro. Veio ent=E3o um matem=E1tico com uma balan=E7a e disse. Me= dindo >apenas duas vezes uma certa quantidade de bolas, posso descobrir qual =E9= a >bola falsa. Como? > >Benjamin Hinrichs > > > > From owner-obm-rj@saci.mat.puc-rio.br Sun Dec 20 22:11:09 1998 Received: by saci.mat.puc-rio.br (AIX 3.2/UCB 5.64/4.03) id AA16480; Sun, 20 Dec 1998 22:10:58 -0200 Received: from [200.248.17.2] by saci.mat.puc-rio.br (AIX 3.2/UCB 5.64/4.03) id AA22620; Sun, 20 Dec 1998 22:10:46 -0200 Received: from server [200.248.17.55] by sinos.net [200.248.17.2] with SMTP (MDaemon.v2.7.SP4.R) for ; Sun, 20 Dec 1998 22:09:43 -0200 From: "Benjamin Hinrichs" To: Subject: Bons livros Date: Sun, 20 Dec 1998 22:07:01 -0200 Message-Id: <01be2c75$d53a0de0$0100007f@localhost> Mime-Version: 1.0 Content-Type: text/plain; charset="iso-8859-1" Content-Transfer-Encoding: 8bit X-Priority: 3 X-Msmail-Priority: Normal X-Mailer: Microsoft Outlook Express 4.71.1712.3 X-Mimeole: Produced By Microsoft MimeOLE V4.71.1712.3 X-Mdaemon-Deliver-To: obm-rj@saci.mat.puc-rio.br X-Return-Path: hinsoft@sinos.net Sender: owner-obm-rj@saci.mat.puc-rio.br Precedence: bulk Reply-To: obm-rj@saci.mat.puc-rio.br Status: RO X-Status: X-Keywords: X-UID: 68 Vamos ajudar a campanha da Rede Bobo, digo Globo: Leitura nas Férias, indicando os melhores livros de matemática. Que tal "O Homem que Calculava" ou "Diabo dos Números", da Cia. das Letras. Se alguém tem uma boa recomendação a fazer, estou de ouvindo. Benjamin Hinrichs From owner-obm-rj@saci.mat.puc-rio.br Mon Dec 21 04:24:55 1998 Received: by saci.mat.puc-rio.br (AIX 3.2/UCB 5.64/4.03) id AA09875; Mon, 21 Dec 1998 04:24:51 -0200 Received: from mail.iis.com.br by saci.mat.puc-rio.br (AIX 3.2/UCB 5.64/4.03) id AA18063; Mon, 21 Dec 1998 04:24:37 -0200 Received: from default (rio-as5-tty04.iis.com.br [200.202.97.84]) by mail.iis.com.br (8.8.7/8.8.14) with SMTP id EAA22927 for ; Mon, 21 Dec 1998 04:24:36 -0200 From: "Marcio" To: Subject: Lucas Date: Wed, 30 Dec 1998 04:27:33 -0300 Message-Id: <01be33c5$ddeca540$LocalHost@default> Mime-Version: 1.0 Content-Type: text/plain; charset="iso-8859-1" X-Priority: 3 X-Msmail-Priority: Normal X-Mailer: Microsoft Outlook Express 4.71.1712.3 X-Mimeole: Produced By Microsoft MimeOLE V4.71.1712.3 Content-Transfer-Encoding: quoted-printable X-Mime-Autoconverted: from 8bit to quoted-printable by mail.iis.com.br id EAA22927 Sender: owner-obm-rj@saci.mat.puc-rio.br Precedence: bulk Reply-To: obm-rj@saci.mat.puc-rio.br Status: RO X-Status: X-Keywords: X-UID: 69 -----Original Message----- From: Lucas To: obm-rj@saci.mat.puc-rio.br Date: S=E1bado, 19 de Dezembro de 1998 23:37 Subject: Muito bem, cara! 3. Duas torneiras enchem completamente um tanque em 3 horas e 45 minutos. Uma delas, quando ligada s=F3, completa o tanque 4 horas a menos que a ou= tra. Quanto tempo cada uma delas leva para encher o tanque, quando ligadas sozinhas ? Perdi um tempo com esse problema, mas acho que, na verdade, eu ja o tinha visto.. Ele parece um problema de fisica.. Seja "x" a vazao com que a torneira 1 enche o tanque, e "y" a vazao c= om que 2 enche o tanque (x e y em l/s). Seja "t" o tempo gasto para encher o tanque com a torneira 1 apenas (logo, t+240 eh o tempo, em segundos, que leva para encher o tanque com a torneira 2). Seja "V" o volume do recipiente. Entao: as duas juntas: 225*x + 225*y =3D V 1 sozinha: t*x =3D V 2 sozinha: (t+240)*y =3D V V/(x+y) =3D 225 , logo (x+y)/V =3D 1/225 e consequentemente: x/V + y/= V =3D 1/225 mas: x/V =3D 1/t e y/V =3D 1/(t+240) Substituindo: 1/t + 1/(t+240) =3D 1/225 Resolvendo a equacao do segundo grau: (2t + 240)*225 =3D t(t+240) 450t + 54000 =3D t^2 + 240t t^2 - 210t - 54000 =3D 0; t =3D 360s =3D 6h (A outra levara 10h) 5. Temos a soma ABCD BCD CD D que resulta em BAAAC. Nesse problema, eu nao consegui ver nenhuma nova solucao.. Na minha opiniao, aquela dada pelo Beijamim eh a unica.. Mas eu posso estar errado= :) O meu raciocinio foi que como o resultado eh um numero de 5 digitos (caso= B !=3D 0) entao A deveria ser um algarismo grande tipo 9 (de repente ate 8)= . O resultado (BAAAC) deveria ser um numero pequeno de cinco algarismos (do t= ipo 1****) e portanto, B =3D 1. mas nesse caso, A =3D 2. O meu raciocinio deve ter ficado meio confuso.. Eu deveria ter tentad= o expo-lo com um rigor maior .. espero que ele esteja certo... para o caso em que B=3D0, chega-se a conclusao de que A=3DB=3DC=3DD=3D= 0 como disse o Beijamim. []'s Marcio From owner-obm-rj@saci.mat.puc-rio.br Mon Dec 21 09:33:49 1998 Received: by saci.mat.puc-rio.br (AIX 3.2/UCB 5.64/4.03) id AA18265; Mon, 21 Dec 1998 09:33:27 -0200 Received: from ns-3.dglnet.com.br by saci.mat.puc-rio.br (AIX 3.2/UCB 5.64/4.03) id AA16981; Mon, 21 Dec 1998 09:33:01 -0200 Received: from juhu3.dglnet.com.br (pm1p29.jd.dglnet.com.br [200.246.57.62]) by ns-3.dglnet.com.br (8.9.1a/8.9.1/Debian/GNU) with SMTP id JAA11332 for ; Mon, 21 Dec 1998 09:32:41 -0200 Message-Id: <367E30DD.7018@dglnet.com.br> Date: Mon, 21 Dec 1998 09:28:29 -0200 From: Arconcher X-Mailer: Mozilla 3.01 (Win95; I) Mime-Version: 1.0 To: obm-rj@saci.mat.puc-rio.br Subject: Re: Bons livros References: <01be2c75$d53a0de0$0100007f@localhost> Content-Type: text/plain; charset=iso-8859-1 Content-Transfer-Encoding: quoted-printable X-Mime-Autoconverted: from 8bit to quoted-printable by ns-3.dglnet.com.br id JAA11332 Sender: owner-obm-rj@saci.mat.puc-rio.br Precedence: bulk Reply-To: obm-rj@saci.mat.puc-rio.br Status: RO X-Status: X-Keywords: X-UID: 70 Benjamin Hinrichs wrote: >=20 > Vamos ajudar a campanha da Rede Bobo, digo Globo: Leitura nas F=E9rias, > indicando os melhores livros de matem=E1tica. Que tal "O Homem que Calc= ulava" > ou "Diabo dos N=FAmeros", da Cia. das Letras. >=20 > Se algu=E9m tem uma boa recomenda=E7=E3o a fazer, estou de ouvindo. >=20 > Benjamin Hinrichs ------------------------ Problem-Solving Strategies.......Arthur Engel(Springer) (talvez esteja um pouquinho acima das possibilidades da apresentadora Ang=E9lica(se n=E3o estou enganado)) ------------------------------------------------------------------------ From owner-obm-rj@saci.mat.puc-rio.br Mon Dec 21 19:32:59 1998 Received: by saci.mat.puc-rio.br (AIX 3.2/UCB 5.64/4.03) id AA18226; Mon, 21 Dec 1998 19:32:47 -0200 Received: from srv4-poa.zaz.com.br by saci.mat.puc-rio.br (AIX 3.2/UCB 5.64/4.03) id AA20013; Mon, 21 Dec 1998 19:32:27 -0200 Received: from default (dl-max4-02.poa.zaz.com.br [200.248.77.66]) by srv4-poa.nutecnet.com.br (8.9.1/8.7.3) with ESMTP id TAA17758 for ; Mon, 21 Dec 1998 19:32:07 -0200 (GMT+2) Message-Id: <199812212132.TAA17758@srv4-poa.nutecnet.com.br> From: "Lucas" To: Subject: =?ISO-8859-1?Q?Reformula=E7=E3o_do_problema?= Date: Sun, 27 Sep 1998 14:13:32 -0300 X-Msmail-Priority: Normal X-Priority: 3 X-Mailer: Microsoft Internet Mail 4.70.1161 Mime-Version: 1.0 Content-Type: text/plain; charset=ISO-8859-1 Content-Transfer-Encoding: quoted-printable X-Mime-Autoconverted: from 8bit to quoted-printable by srv4-poa.nutecnet.com.br id TAA17758 Sender: owner-obm-rj@saci.mat.puc-rio.br Precedence: bulk Reply-To: obm-rj@saci.mat.puc-rio.br Status: RO X-Status: X-Keywords: X-UID: 71 Agora sim as coisas est=E3o indo bem. T=E1 todo mundo ligado nos problema= s, legal. Ser=E1 que algu=E9m consegue resolver o problema do Chico Xavier? = E o desafio de continuar a letra da m=FAsica? Ah, mais uma coisa: acabo de constatar que eu sou um idiota, pois me enganei num problema. O correto =E9= : ABCD mais BCD mais CD mais D igual a AAAA =C9 claro que existem outros bem parecidos, como: AAAA mais BBBB mais CCCC igual a BAAAC O que eu achei mais dif=EDcil foi este: ABC=20 x DC ________ DECB F I F _________ I EBB E temos este aqui tamb=E9m, que =E9 bem divertido. Tenho 4 solu=E7=F5es a= t=E9 agora... ABC mais DEF igual a GHIJ(ou seja, =E9 uma soma de centenas que resulta e= m um n=FAmero de 4 algarismos, sendo que empregamos todos os d=EDgitos de 0 a = 9 uma =FAnica vez. Tentem...) Lucas P. M. From owner-obm-rj@saci.mat.puc-rio.br Mon Dec 21 19:47:54 1998 Received: by saci.mat.puc-rio.br (AIX 3.2/UCB 5.64/4.03) id AA21608; Mon, 21 Dec 1998 19:47:43 -0200 Received: from srv4-poa.zaz.com.br by saci.mat.puc-rio.br (AIX 3.2/UCB 5.64/4.03) id AA02660; Mon, 21 Dec 1998 19:47:24 -0200 Received: from default (dl-max4-02.poa.zaz.com.br [200.248.77.66]) by srv4-poa.nutecnet.com.br (8.9.1/8.7.3) with ESMTP id TAA18516 for ; Mon, 21 Dec 1998 19:47:18 -0200 (GMT+2) Message-Id: <199812212147.TAA18516@srv4-poa.nutecnet.com.br> From: "Lucas" To: Subject: Well, well, well,... Date: Sun, 27 Sep 1998 14:28:44 -0300 X-Msmail-Priority: Normal X-Priority: 3 X-Mailer: Microsoft Internet Mail 4.70.1161 Mime-Version: 1.0 Content-Type: text/plain; charset=ISO-8859-1 Content-Transfer-Encoding: quoted-printable X-Mime-Autoconverted: from 8bit to quoted-printable by srv4-poa.nutecnet.com.br id TAA18516 Sender: owner-obm-rj@saci.mat.puc-rio.br Precedence: bulk Reply-To: obm-rj@saci.mat.puc-rio.br Status: RO X-Status: X-Keywords: X-UID: 72 Quem resolver este problema =E9 candidato a g=EAnio do ano. Vejamos: * * 5 * x * * _________ * * * * * * * * * __________ 8645 7 Descobrir 2 solu=E7=F5es. E este aqui =E9 bem legal: S=F3crates: diga-me, Plat=E3o, quais s=E3o as idades de teus tr=EAs filho= s ? Plat=E3o: o produto delas =E9 36. S=F3crates: com esta informa=E7=E3o n=E3o =E9 poss=EDvel saber as idades. Plat=E3o: a soma delas resulta no n=FAmero desta casa a=ED na frente. S=F3crates: ainda n=E3o d=E1 pra saber. Plat=E3o: o mais velho =E9 uma bixa louca! S=F3crates: ah, agora eu j=E1 sei as idades de cada um deles! Como =E9 que ele sabe? Lucas P. M. =20 From owner-obm-rj@saci.mat.puc-rio.br Mon Dec 21 20:09:10 1998 Received: by saci.mat.puc-rio.br (AIX 3.2/UCB 5.64/4.03) id AA17087; Mon, 21 Dec 1998 20:08:57 -0200 Received: from [200.248.17.2] by saci.mat.puc-rio.br (AIX 3.2/UCB 5.64/4.03) id AA05050; Mon, 21 Dec 1998 20:08:42 -0200 Received: from server [200.248.17.29] by sinos.net [200.248.17.2] with SMTP (MDaemon.v2.7.SP4.R) for ; Mon, 21 Dec 1998 20:06:44 -0200 From: "Benjamin Hinrichs" To: Subject: Marcio Date: Mon, 21 Dec 1998 20:00:44 -0200 Message-Id: <01be2d2d$5b592660$0100007f@localhost> Mime-Version: 1.0 Content-Type: text/plain; charset="iso-8859-1" Content-Transfer-Encoding: 8bit X-Priority: 3 X-Msmail-Priority: Normal X-Mailer: Microsoft Outlook Express 4.71.1712.3 X-Mimeole: Produced By Microsoft MimeOLE V4.71.1712.3 X-Mdaemon-Deliver-To: obm-rj@saci.mat.puc-rio.br X-Return-Path: hinsoft@sinos.net Sender: owner-obm-rj@saci.mat.puc-rio.br Precedence: bulk Reply-To: obm-rj@saci.mat.puc-rio.br Status: RO X-Status: X-Keywords: X-UID: 73 Marcio, acho que tudo o que dissestes no e-mail está certo. Ainda não tive tempo de entender a tua equação das torneiras. Quanto ao outro problema, quero dizer que pensei o mesmo. Não sei se isto é um grande avanço, mas cheguei a grande e fantástica conclusão que o problema pode ser representado assim: 1000 x A + 200 x B + 30 x C +4 x D = 10000 x B + 1110 A + 1 x C -110A -9800B -29E +4D = 0 110A +9800B +29E -4D = 0 Como se avança a partir daí não é mais o meu departamento. Abraço para todos, Benjamin Hinrichs From owner-obm-rj@saci.mat.puc-rio.br Mon Dec 21 21:34:59 1998 Received: by saci.mat.puc-rio.br (AIX 3.2/UCB 5.64/4.03) id AA21095; Mon, 21 Dec 1998 21:34:49 -0200 Received: from mail.iis.com.br by saci.mat.puc-rio.br (AIX 3.2/UCB 5.64/4.03) id AA20321; Mon, 21 Dec 1998 21:34:29 -0200 Received: from default (rio-as6-tty08.iis.com.br [200.202.97.104]) by mail.iis.com.br (8.8.7/8.8.14) with SMTP id VAA19128 for ; Mon, 21 Dec 1998 21:34:25 -0200 From: "Marcio" To: Subject: Re: Well, well, well,... Date: Wed, 30 Dec 1998 21:36:11 -0300 Message-Id: <01be3455$90a2b780$LocalHost@default> Mime-Version: 1.0 Content-Type: text/plain; charset="iso-8859-1" X-Priority: 3 X-Msmail-Priority: Normal X-Mailer: Microsoft Outlook Express 4.71.1712.3 X-Mimeole: Produced By Microsoft MimeOLE V4.71.1712.3 Content-Transfer-Encoding: quoted-printable X-Mime-Autoconverted: from 8bit to quoted-printable by mail.iis.com.br id VAA19128 Sender: owner-obm-rj@saci.mat.puc-rio.br Precedence: bulk Reply-To: obm-rj@saci.mat.puc-rio.br Status: RO X-Status: X-Keywords: X-UID: 74 Esse segundo problema eu ja vi. Ele realmente eh bastante interessant= e. Quanto ao primeiro, nao tenho certeza se ja vi um igual. Vou tentar o primeiro depois. []'s Marcio -----Original Message----- From: Lucas To: obm-rj@saci.mat.puc-rio.br Date: Segunda-feira, 21 de Dezembro de 1998 18:49 Subject: Well, well, well,... Quem resolver este problema =E9 candidato a g=EAnio do ano. Vejamos: * * 5 * x * * _________ * * * * * * * * * __________ 8645 7 Descobrir 2 solu=E7=F5es. E este aqui =E9 bem legal: S=F3crates: diga-me, Plat=E3o, quais s=E3o as idades de teus tr=EAs filho= s ? Plat=E3o: o produto delas =E9 36. S=F3crates: com esta informa=E7=E3o n=E3o =E9 poss=EDvel saber as idades. Plat=E3o: a soma delas resulta no n=FAmero desta casa a=ED na frente. S=F3crates: ainda n=E3o d=E1 pra saber. Plat=E3o: o mais velho =E9 uma bixa louca! S=F3crates: ah, agora eu j=E1 sei as idades de cada um deles! Como =E9 que ele sabe? Lucas P. M. From nicolau@saci.mat.puc-rio.br Tue Dec 22 20:56:09 1998 Received: from dial05.impa.br by saci.mat.puc-rio.br (AIX 3.2/UCB 5.64/4.03) id AA17363; Tue, 22 Dec 1998 20:56:06 -0200 Received: from localhost (nicolau@localhost) by washoe.domus (8.8.7/8.8.7) with ESMTP id UAA28087; Tue, 22 Dec 1998 20:50:31 -0200 X-Authentication-Warning: washoe.domus: nicolau owned process doing -bs Date: Tue, 22 Dec 1998 20:50:26 -0200 (EDT) From: "Nicolau C. Saldanha" X-Sender: nicolau@washoe To: Luiz Gustavo Nogara Cc: nicolau@saci.mat.puc-rio.br Subject: Re: Programa de Fractais In-Reply-To: <000701be2dbf$ede89a00$386ae0c8@lgnogara> Message-Id: Mime-Version: 1.0 Content-Type: TEXT/PLAIN; charset=iso-8859-1 Content-Transfer-Encoding: QUOTED-PRINTABLE Status: RO X-Status: X-Keywords: X-UID: 75 On Tue, 22 Dec 1998, Luiz Gustavo Nogara wrote: > Oi Nicolau >=20 > Desculpe-nos... a Luiza acabou viajando e nao deve ter checado seus e-mai= ls. > E eu fui ontem abrir minha caixa de correio, depois de alguns dias sem > entrar na internet. Nessa epoca de natal eu tive que trabalhar na loja da > minha mae. Portanto, eu nem vi seu mail. Desculpe-nos por nao ter dado > qualquer resposta. >=20 > A gente vai estar por aqui em janeiro, para fazer as matriculas na puc. > Estamos realmente interessados em ver esse programa funcionando ai na sua > maquina. Nao sei se esses dias entre natal e ano novo seriam muito propic= ios > para qualquer encontro, isso depende de voce, mas em janeiro e fevereiro > estaremos disponiveis para qualquer encontro. >=20 > Esperamos resposta. > Um abraco. >=20 > Luiz Gustavo e Luiza Nenhum problema, aguardo mensagem de voc=EAs em janeiro.=20 Bom Natal e Ano Novo, Nicolau From owner-obm-rj@saci.mat.puc-rio.br Tue Dec 22 22:57:54 1998 Received: by saci.mat.puc-rio.br (AIX 3.2/UCB 5.64/4.03) id AA04902; Tue, 22 Dec 1998 22:57:45 -0200 Received: from [200.248.17.2] by saci.mat.puc-rio.br (AIX 3.2/UCB 5.64/4.03) id AA18978; Tue, 22 Dec 1998 22:57:28 -0200 Received: from server [200.248.17.12] by sinos.net [200.248.17.2] with SMTP (MDaemon.v2.7.SP4.R) for ; Tue, 22 Dec 1998 22:55:31 -0200 From: "Benjamin Hinrichs" To: Subject: Re: Well, well, well,... Date: Tue, 22 Dec 1998 22:39:40 -0200 Message-Id: <01be2e0c$b9b0a780$0100007f@localhost> Mime-Version: 1.0 Content-Type: text/plain; charset="iso-8859-1" Content-Transfer-Encoding: 8bit X-Priority: 3 X-Msmail-Priority: Normal X-Mailer: Microsoft Outlook Express 4.71.1712.3 X-Mimeole: Produced By Microsoft MimeOLE V4.71.1712.3 X-Mdaemon-Deliver-To: obm-rj@saci.mat.puc-rio.br X-Return-Path: hinsoft@sinos.net Sender: owner-obm-rj@saci.mat.puc-rio.br Precedence: bulk Reply-To: obm-rj@saci.mat.puc-rio.br Status: RO X-Status: X-Keywords: X-UID: 76 Mais uma vez vamos por partes. É que eu tenho muitos problemas (um deles é a mentira constante). O primeiro problema eu deixo para um outro gênio do ano. Daqui a uma semana eu resolvo. Quem resolver este problema é candidato a gênio do ano. Vejamos: * * 5 * x * * _________ * * * * * * * * * __________ 8645 7 Descobrir 2 soluções. E este aqui é bem legal: Sócrates: diga-me, Platão, quais são as idades de teus três filhos ? Platão: o produto delas é 36. Sócrates: com esta informação não é possível saber as idades. Platão: a soma delas resulta no número desta casa aí na frente. Sócrates: ainda não dá pra saber. Platão: o mais velho é uma bixa louca! Sócrates: ah, agora eu já sei as idades de cada um deles! Como é que ele sabe? Pensava que tinha mais a ver com o comportamento sexual, porém meu pai me mostrou como resolver. Era só saber quais as possíveis idades para os filhos. São elas com a soma entre elas em parênteses: 36; 1; 1; (38) 18; 2; 1; (21) 12; 3; 1; (16) 9; 4; 1; (14) 9; 2; 2; (13) 6; 6; 1; (13) 6; 3; 2; (11) 4; 3; 3; (10) Como o sábio Sócrates disse que mesmo assim não dava para saber, então só pode ser aqueles, cujas somas são iguais, ou seja 9; 2; 2; e 6; 6; 1;. Coube então à Platão dizer à Sócrates que há um mais velho entre eles, para que Sócrates dissesse à Platão a idade correta dos filhos dele. Só queria saber do Marcio por que ele acha este problema tão "interessante"... Um abraço e um feliz natal para todos. Benjamin Hinrichs From owner-obm-rj@saci.mat.puc-rio.br Wed Dec 23 01:04:02 1998 Received: by saci.mat.puc-rio.br (AIX 3.2/UCB 5.64/4.03) id AA17289; Wed, 23 Dec 1998 01:03:56 -0200 Received: from mail.iis.com.br by saci.mat.puc-rio.br (AIX 3.2/UCB 5.64/4.03) id AA17284; Wed, 23 Dec 1998 01:03:34 -0200 Received: from default (rio-tc0-tty19.iis.com.br [200.202.98.19]) by mail.iis.com.br (8.8.7/8.8.14) with SMTP id BAA26271 for ; Wed, 23 Dec 1998 01:03:30 -0200 From: "Marcio" To: Subject: Re: Well, well, well,... Date: Fri, 1 Jan 1999 01:05:31 -0300 Message-Id: <01be353b$f925cfe0$LocalHost@default> Mime-Version: 1.0 Content-Type: text/plain; charset="iso-8859-1" X-Priority: 3 X-Msmail-Priority: Normal X-Mailer: Microsoft Outlook Express 4.71.1712.3 X-Mimeole: Produced By Microsoft MimeOLE V4.71.1712.3 Content-Transfer-Encoding: quoted-printable X-Mime-Autoconverted: from 8bit to quoted-printable by mail.iis.com.br id BAA26271 Sender: owner-obm-rj@saci.mat.puc-rio.br Precedence: bulk Reply-To: obm-rj@saci.mat.puc-rio.br Status: RO X-Status: X-Keywords: X-UID: 77 Bom, se realmente ha um espaco entre o "5" e o "7" do resultado, enta= o eu nao sei fazer (mas pelo que eu vi, rapido, num programa para fatoracao= , o problema nao admitiria solucao exceto 0,0,0,...). Caso nao haja o espaco, a unica solucao nao-trivial que eu achei foi 3759*23. Agora eu vou aproveitar esses dias de ferias pra dar uma descansada e provavelmente nao vou tentar resolver os proximos problemas :) Mas se voc= es quiserem um problema no estilo desse ultimo pra tentar resolver: * * * * * * * | * * * * * * * 8 * * ___ * * * * __ * * * * * * ____ 0 Nao sei se consegui deixar o problema claro, mas trata-se de uma cont= a de dividir na qual os "*" representam algarismos nao necessariamente igua= is de 0 a 9. O divisor tem dois algarismos e o dividendo tem 7. Marcio -----Original Message----- From: Lucas To: obm-rj@saci.mat.puc-rio.br Date: Segunda-feira, 21 de Dezembro de 1998 18:49 Subject: Well, well, well,... Quem resolver este problema =E9 candidato a g=EAnio do ano. Vejamos: * * 5 * x * * _________ * * * * * * * * * __________ 8645 7 Descobrir 2 solu=E7=F5es. E este aqui =E9 bem legal: S=F3crates: diga-me, Plat=E3o, quais s=E3o as idades de teus tr=EAs filho= s ? Plat=E3o: o produto delas =E9 36. S=F3crates: com esta informa=E7=E3o n=E3o =E9 poss=EDvel saber as idades. Plat=E3o: a soma delas resulta no n=FAmero desta casa a=ED na frente. S=F3crates: ainda n=E3o d=E1 pra saber. Plat=E3o: o mais velho =E9 uma bixa louca! S=F3crates: ah, agora eu j=E1 sei as idades de cada um deles! Como =E9 que ele sabe? Lucas P. M. From owner-obm-rj@saci.mat.puc-rio.br Wed Dec 23 01:04:06 1998 Received: by saci.mat.puc-rio.br (AIX 3.2/UCB 5.64/4.03) id AA19080; Wed, 23 Dec 1998 01:03:54 -0200 Received: from mail.iis.com.br by saci.mat.puc-rio.br (AIX 3.2/UCB 5.64/4.03) id AA17279; Wed, 23 Dec 1998 01:03:31 -0200 Received: from default (rio-tc0-tty19.iis.com.br [200.202.98.19]) by mail.iis.com.br (8.8.7/8.8.14) with SMTP id BAA26267 for ; Wed, 23 Dec 1998 01:03:29 -0200 From: "Marcio" To: Subject: =?iso-8859-1?Q?Re:_Reformula=E7=E3o_do_problema?= Date: Fri, 1 Jan 1999 00:57:02 -0300 Message-Id: <01be353a$ca0edea0$LocalHost@default> Mime-Version: 1.0 Content-Type: text/plain; charset="iso-8859-1" X-Priority: 3 X-Msmail-Priority: Normal X-Mailer: Microsoft Outlook Express 4.71.1712.3 X-Mimeole: Produced By Microsoft MimeOLE V4.71.1712.3 Content-Transfer-Encoding: quoted-printable X-Mime-Autoconverted: from 8bit to quoted-printable by mail.iis.com.br id BAA26267 Sender: owner-obm-rj@saci.mat.puc-rio.br Precedence: bulk Reply-To: obm-rj@saci.mat.puc-rio.br Status: RO X-Status: X-Keywords: X-UID: 78 Ahh.. :) Aqui estao as solucoes que eu encontrei: 1) A=3D4; B=3D1; C=3D8; D=3D1 2) A=3D2; B=3D0; C=3D7; D=3D3 3) A=3D8; B=3D4; C=3D2; D=3D7 4) A=3D6; B=3D3; C=3D1; D=3D9 5) A=3DB=3DC=3DD=3D0 Para o da multiplicacao : 348 * 28 =3D 9744 Os outros eu nao tentei. Quanto a letra da musica, eu nao faco a menor ideia.. :)) Respondendo ao Benjamim, o que eu acho interessante no problema das idades eh que a primeira vista, nao faz muito sentido o outro descobrir a idade a partir da ultima informacao.. Quando eu vi esse problema pela primeira vez, eu perdi um bom tempo tentando resolve-lo (Nao lembro se consegui ou se alguem me disse a resposta). -----Original Message----- From: Lucas To: obm-rj@saci.mat.puc-rio.br Date: Segunda-feira, 21 de Dezembro de 1998 18:33 Subject: Reformula=E7=E3o do problema Agora sim as coisas est=E3o indo bem. T=E1 todo mundo ligado nos problema= s, legal. Ser=E1 que algu=E9m consegue resolver o problema do Chico Xavier? = E o desafio de continuar a letra da m=FAsica? Ah, mais uma coisa: acabo de constatar que eu sou um idiota, pois me enganei num problema. O correto =E9= : ABCD mais BCD mais CD mais D igual a AAAA =C9 claro que existem outros bem parecidos, como: AAAA mais BBBB mais CCCC igual a BAAAC O que eu achei mais dif=EDcil foi este: ABC x DC ________ DECB F I F _________ I EBB E temos este aqui tamb=E9m, que =E9 bem divertido. Tenho 4 solu=E7=F5es a= t=E9 agora... ABC mais DEF igual a GHIJ(ou seja, =E9 uma soma de centenas que resulta e= m um n=FAmero de 4 algarismos, sendo que empregamos todos os d=EDgitos de 0 a = 9 uma =FAnica vez. Tentem...) Lucas P. M. From owner-obm-rj@saci.mat.puc-rio.br Thu Dec 24 10:43:14 1998 Received: by saci.mat.puc-rio.br (AIX 3.2/UCB 5.64/4.03) id AA22312; Thu, 24 Dec 1998 10:43:03 -0200 Received: from srv4-poa.zaz.com.br by saci.mat.puc-rio.br (AIX 3.2/UCB 5.64/4.03) id AA17444; Thu, 24 Dec 1998 10:42:26 -0200 Received: from default (dl-tnt1-185.poa.zaz.com.br [200.248.211.185]) by srv4-poa.nutecnet.com.br (8.9.1/8.7.3) with ESMTP id KAA12071 for ; Thu, 24 Dec 1998 10:42:17 -0200 (GMT+2) Message-Id: <199812241242.KAA12071@srv4-poa.nutecnet.com.br> From: "Lucas" To: Subject: Ho ho ho! Date: Mon, 28 Sep 1998 09:34:00 -0300 X-Msmail-Priority: Normal X-Priority: 3 X-Mailer: Microsoft Internet Mail 4.70.1161 Mime-Version: 1.0 Content-Type: text/plain; charset=ISO-8859-1 Content-Transfer-Encoding: quoted-printable X-Mime-Autoconverted: from 8bit to quoted-printable by srv4-poa.nutecnet.com.br id KAA12071 Sender: owner-obm-rj@saci.mat.puc-rio.br Precedence: bulk Reply-To: obm-rj@saci.mat.puc-rio.br Status: RO X-Status: X-Keywords: X-UID: 79 Fala gurizada medonha,... Hoje, ou melhor, daqui a 15 minutos, eu saio de casa e viajo pra Cidreira= , minha praia. =C9 um lixo, mas eu a amo. Tudo bem, n=E3o importa. Eu s=F3 = queria dar um Feliz Natal pra todos e que todos os problemas que n=F3s conhecemo= s sejam resolvidos no ano que vem... Um abra=E7o amigo, Lucas P. M. From owner-obm-rj@saci.mat.puc-rio.br Thu Dec 24 16:15:12 1998 Received: by saci.mat.puc-rio.br (AIX 3.2/UCB 5.64/4.03) id AA03223; Thu, 24 Dec 1998 16:15:09 -0200 Received: from [200.248.17.2] by saci.mat.puc-rio.br (AIX 3.2/UCB 5.64/4.03) id AA05267; Thu, 24 Dec 1998 16:14:54 -0200 Received: from server [200.248.17.29] by sinos.net [200.248.17.2] with SMTP (MDaemon.v2.7.SP4.R) for ; Thu, 24 Dec 1998 16:13:02 -0200 From: "Benjamin Hinrichs" To: Subject: =?iso-8859-1?Q?Gra=E7as_a_Deus?= Date: Thu, 24 Dec 1998 14:31:54 -0200 Message-Id: <01be2f5a$eab75c80$0100007f@localhost> Mime-Version: 1.0 Content-Type: text/plain; charset="iso-8859-1" Content-Transfer-Encoding: 8bit X-Priority: 3 X-Msmail-Priority: Normal X-Mailer: Microsoft Outlook Express 4.71.1712.3 X-Mimeole: Produced By Microsoft MimeOLE V4.71.1712.3 X-Mdaemon-Deliver-To: obm-rj@saci.mat.puc-rio.br X-Return-Path: hinsoft@sinos.net Sender: owner-obm-rj@saci.mat.puc-rio.br Precedence: bulk Reply-To: obm-rj@saci.mat.puc-rio.br Status: RO X-Status: X-Keywords: X-UID: 80 Lucas, que bom que vais à praia por um momento, assim teremos um pouco mais tempo para resolver todos os problemas. Um abraço e Feliz Natal, Benjamin Hinrichs From owner-obm-rj@saci.mat.puc-rio.br Wed Dec 30 00:57:28 1998 Received: by saci.mat.puc-rio.br (AIX 3.2/UCB 5.64/4.03) id AA18540; Wed, 30 Dec 1998 00:56:58 -0200 Received: from [200.248.17.2] by saci.mat.puc-rio.br (AIX 3.2/UCB 5.64/4.03) id AA02664; Wed, 30 Dec 1998 00:56:37 -0200 Received: from server [200.248.17.20] by sinos.net [200.248.17.2] with SMTP (MDaemon.v2.7.SP4.R) for ; Wed, 30 Dec 1998 00:55:49 -0200 From: "Benjamin Hinrichs" To: Subject: =?iso-8859-1?Q?O_g=EAnio_do_ano?= Date: Wed, 30 Dec 1998 00:49:47 -0200 Message-Id: <01be339f$0fc8ff40$0100007f@localhost> Mime-Version: 1.0 Content-Type: text/plain; charset="iso-8859-1" Content-Transfer-Encoding: 8bit X-Priority: 3 X-Msmail-Priority: Normal X-Mailer: Microsoft Outlook Express 4.71.1712.3 X-Mimeole: Produced By Microsoft MimeOLE V4.71.1712.3 X-Mdaemon-Deliver-To: obm-rj@saci.mat.puc-rio.br X-Return-Path: hinsoft@sinos.net Sender: owner-obm-rj@saci.mat.puc-rio.br Precedence: bulk Reply-To: obm-rj@saci.mat.puc-rio.br Status: RO X-Status: X-Keywords: X-UID: 81 >>>>Lucas wrote: Quem resolver este problema é candidato a gênio do ano. Vejamos: * * 5 * x * * _________ * * * * * * * * * __________ 8645 7 Descobrir 2 soluções. E este aqui é bem legal: Sócrates: diga-me, Platão, quais são as idades de teus três filhos ? Platão: o produto delas é 36. Sócrates: com esta informação não é possível saber as idades. Platão: a soma delas resulta no número desta casa aí na frente. Sócrates: ainda não dá pra saber. Platão: o mais velho é uma bixa louca! Sócrates: ah, agora eu já sei as idades de cada um deles! Como é que ele sabe? Lucas P. M. <<<< >>>>Benjamin Hinrichs wrote: Olá amigos matemáticos, já vimos a solução do segundo problema, porém nenhum dos candidatos a gênio do ano quiseram tentar a primeira pergunta. Pois acabo de matá-la. A so~lução é mais fácil do que pensava: basta decompor o número em fatores primos (escrevi um programinha em basic que faz isto para mim) e então tentar todas as onze possibilidades de multiplicação e então ver quais podem estar certas (o dígito do dez do primeiro número deve ser cinco). A decomposição de 86457 é 3 x 7 x 23 x 179. Aí foi só testar que 3 x 7 x 179 é 3759, logo 3759 | e 7 x 179 1253 x 23 | 3 x 23 x 69 ______ | _____ 11277 | 11277 75180 | 75180 _______ | ______ 86457 | 86457 Só gostaria de saber do Lucas quando vai ser escolhido o gênio do ano e se as perguntas vão começar a se tornar mais fáceis. Aliás, manda aí a resposta do problema da canção do Chico Xavier. Um grande abraço, Benjamin Hinrichs (o modesto gênio do ano) From owner-obm-rj@saci.mat.puc-rio.br Mon Jan 4 18:07:47 1999 Received: by saci.mat.puc-rio.br (AIX 3.2/UCB 5.64/4.03) id AA22945; Mon, 4 Jan 1999 18:04:38 -0200 Received: from srv4-poa.zaz.com.br by saci.mat.puc-rio.br (AIX 3.2/UCB 5.64/4.03) id AA23453; Mon, 4 Jan 1999 18:04:17 -0200 Received: from default (dl-tnt1-094.poa.zaz.com.br [200.248.211.94]) by srv4-poa.nutecnet.com.br (8.9.1/8.7.3) with ESMTP id SAA12774 for ; Mon, 4 Jan 1999 18:04:06 -0200 (GMT+2) Message-Id: <199901042004.SAA12774@srv4-poa.nutecnet.com.br> From: "Lucas" To: Subject: O primeiro problema de 1999. Date: Mon, 28 Sep 1998 10:21:04 -0300 X-Msmail-Priority: Normal X-Priority: 3 X-Mailer: Microsoft Internet Mail 4.70.1161 Mime-Version: 1.0 Content-Type: text/plain; charset=ISO-8859-1 Content-Transfer-Encoding: quoted-printable X-Mime-Autoconverted: from 8bit to quoted-printable by srv4-poa.nutecnet.com.br id SAA12774 Sender: owner-obm-rj@saci.mat.puc-rio.br Precedence: bulk Reply-To: obm-rj@saci.mat.puc-rio.br Status: RO X-Status: X-Keywords: X-UID: 82 Caros amigos, temos um g=EAnio do ano. E ele se chama Benjamin. Parab=E9ns, cara ! Est=E1 correta a sua resposta. Mudando de assunto, a can=E7=E3o =E9 do Renato Ru= sso, do Legi=E3o Urbana. E o problema do Chico Xavier tem a seguinte solu=E7=E3o: Chico falou ao ouvido de cada um dos irm=E3os: "Monte no cavalo de teu ir= m=E3o e corra o mais r=E1pido que puder". Quanto =E0 dificuldade dos problemas de 1999, caro G=EAnio do Ano, acho q= ue ela n=E3o ser=E1 alterada. A=ED vai um probleminha para aquecer os vossos c=E9rebros para este ano: Vamos supor que os n=FAmeros naturais 0, 1, 2, 3, 4, ... , 1000 foram escritos por extenso, a saber:=20 Zero, um, dois, tr=EAs, quatro, ..., mil.=20 a) Qual destes n=FAmeros se escreve com o menor n=FAmero de letras? b) E com maior n=FAmero de letras? c) Qual =E9 o n=FAmero que exprime o seu pr=F3prio n=FAmero de letras? d) Qual =E9 o maior n=FAmero que se escreve com 4 letras? e) Escrevendo-se por extenso e em ordem alfab=E9tica todos estes n=FAmero= s, quais s=E3o os tr=EAs primeiros e os tr=EAs =FAltimos? =20 From owner-obm-rj@saci.mat.puc-rio.br Tue Jan 5 01:24:37 1999 Received: by saci.mat.puc-rio.br (AIX 3.2/UCB 5.64/4.03) id AA21586; Tue, 5 Jan 1999 01:24:08 -0200 Received: from [200.248.17.2] by saci.mat.puc-rio.br (AIX 3.2/UCB 5.64/4.03) id AA20814; Tue, 5 Jan 1999 01:23:53 -0200 Received: from server [200.248.17.63] by sinos.net [200.248.17.2] with SMTP (MDaemon.v2.7.SP4.R) for ; Tue, 05 Jan 1999 01:22:06 -0200 From: "Benjamin Hinrichs" To: Subject: =?iso-8859-1?Q?Re:_Vote_En=E9as=2C_56?= Date: Tue, 5 Jan 1999 01:18:58 -0200 Message-Id: <01be385a$22064180$3f11f8c8@server> Mime-Version: 1.0 Content-Type: text/plain; charset="iso-8859-1" Content-Transfer-Encoding: 8bit X-Priority: 3 X-Msmail-Priority: Normal X-Mailer: Microsoft Outlook Express 4.71.1712.3 X-Mimeole: Produced By Microsoft MimeOLE V4.71.1712.3 X-Mdaemon-Deliver-To: obm-rj@saci.mat.puc-rio.br X-Return-Path: hinsoft@sinos.net Sender: owner-obm-rj@saci.mat.puc-rio.br Precedence: bulk Reply-To: obm-rj@saci.mat.puc-rio.br Status: RO X-Status: X-Keywords: X-UID: 83 Olá, O problema do Chico Chavier está na mensagem "Vote Enéas, 56", por volta de dia 18/12 ou 19/12, conforme a data de recebimento. Mate a cobra e mostre o porrete!: 1. Um pai morreu e deixou uma herança para que seus dois filhos a disputassem. No testamento ele disse: "...deverá ser realizada uma corrida de cavalo entre meus dois filhos; aquele que chegar por último na estátua de São Lucas será o vencedor". Então um dos filhos montou em seu cavalo e o outro fez o mesmo com o seu cavalo. Porém, eles iam tão devagar (pois queriam receber a grana, chegando em último) que nunca chegariam ao seu destino. No meio do caminho, por sorte, eles encontram o sábio Chico Xavier e decidiram lhe pedir ajuda. Então ele cochichou uma frase no ouvido de cada um dos irmãos e, em menos de cinco segundos, os dois montaram nos cavalos e disputaram a mais veloz das corridas. Qual foi a sábia frase dita por Chico Xavier, sendo que ela deve respeitar a condição do testamento? Não sei se é má interpretação minha, mas pede-se para que pessoas cheguem por último na tal estátua, não cavalos. Meu pai até me disse que é para os cavalos chegarem por último, mas não é realmente o que o problema afirma. Um grande abraço de ano novo, Benjamin Hinrichs From owner-obm-rj@saci.mat.puc-rio.br Wed Jan 6 16:11:41 1999 Received: by saci.mat.puc-rio.br (AIX 3.2/UCB 5.64/4.03) id AA18065; Wed, 6 Jan 1999 16:10:57 -0200 Received: from [200.248.17.2] by saci.mat.puc-rio.br (AIX 3.2/UCB 5.64/4.03) id AA21642; Wed, 6 Jan 1999 16:10:13 -0200 Received: from server [200.248.17.22] by sinos.net [200.248.17.2] with SMTP (MDaemon.v2.7.SP4.R) for ; Wed, 06 Jan 1999 16:09:36 -0300 From: "Benjamin Hinrichs" To: Subject: O primeiro problema resolvido de 1999. Date: Wed, 6 Jan 1999 16:02:10 -0200 Message-Id: <01be399e$ae8636c0$0100007f@localhost> Mime-Version: 1.0 Content-Type: text/plain; charset="iso-8859-1" Content-Transfer-Encoding: 8bit X-Priority: 3 X-Msmail-Priority: Normal X-Mailer: Microsoft Outlook Express 4.71.1712.3 X-Mimeole: Produced By Microsoft MimeOLE V4.71.1712.3 X-Mdaemon-Deliver-To: obm-rj@saci.mat.puc-rio.br X-Return-Path: hinsoft@sinos.net Sender: owner-obm-rj@saci.mat.puc-rio.br Precedence: bulk Reply-To: obm-rj@saci.mat.puc-rio.br Status: RO X-Status: X-Keywords: X-UID: 84 Vamos supor que os números naturais 0, 1, 2, 3, 4, ... , 1000 foram escritos por extenso, a saber: Zero, um, dois, três, quatro, ..., mil. a) Qual destes números se escreve com o menor número de letras? creio que, como não há número com uma única letra, deve ser o um. b) E com maior número de letras? basta escrever os múltiplos de cem por extenso, em seguida os de dez e então a maior unidade. De forma destacada vemos logo que é quatrocentos e cinqüenta e quatro, se não me falha a ortografia. c) Qual é o número que exprime o seu próprio número de letras? esta é barbada; é o cinco. nem ''e muito difícil de se chegar a tal resposta. É só ir experimentando. d) Qual é o maior número que se escreve com 4 letras? eu já ia escrever nove, mas é doze. e) Escrevendo-se por extenso e em ordem alfabética todos estes números, quais são os três primeiros e os três últimos? Os primeiros são cem, cinco e cinqüenta, se não me engano. Os últimos são vinte e um, vinte e três e vinte e sete. esta vai de volta para o autor: qual é o maior número que se escreve com 3 letras? Curiosidade: no francês não existem os números de 70 até 79 e de 90 até 99. Eles são expressos como sessenta e dez, sessenta e onze, etc. e oitenta e dez, oitenta e onze, etc. Um abraço, Benjamin Hinrichs From owner-obm-rj@saci.mat.puc-rio.br Wed Jan 6 16:24:01 1999 Received: by saci.mat.puc-rio.br (AIX 3.2/UCB 5.64/4.03) id AA20186; Wed, 6 Jan 1999 16:23:40 -0200 Received: from dns.digi.com.br by saci.mat.puc-rio.br (AIX 3.2/UCB 5.64/4.03) id AA05325; Wed, 6 Jan 1999 16:21:03 -0200 Received: from ppp1117.digi.com.br (unverified [200.241.111.7]) by vop.digi.com.br (Vircom SMTPRS 1.0.169) with SMTP id for ; Wed, 06 Jan 1999 15:20:13 -0300 Message-Id: <3.0.3.32.19990106151131.00860170@digi.com.br> X-Sender: bene@digi.com.br X-Mailer: QUALCOMM Windows Eudora Light Version 3.0.3 (32) Date: Wed, 06 Jan 1999 15:11:31 -0200 To: obm-rj@saci.mat.puc-rio.br From: benedito Subject: Problemas Mime-Version: 1.0 Content-Type: text/plain; charset="iso-8859-1" Content-Transfer-Encoding: quoted-printable Sender: owner-obm-rj@saci.mat.puc-rio.br Precedence: bulk Reply-To: obm-rj@saci.mat.puc-rio.br Status: RO X-Status: X-Keywords: X-UID: 85 Segue abaixo quatro problemas 1) Ache todos os pares de inteiros (x, y) satisfazendo a seguinte equa=E7= =E3o 1 + 1996x + 1998y =3D xy. 2) Seja A um subconjunto de {0, 1, 2, 3, 4, ...., 1996, 1997} contendo mais que 1.000 elementos. Prove que: ou A cont=E9m uma pot=EAncia de 2 (isto =E9, um n=FAmero da forma 2k com k um inteiro n=E3o negativo) ou ex= istem dois elementos distintos a, b em A tais que a soma a + b =E9 uma pot=EAncia de 2. 3) Seja S o conjunto de todos os n=FAmeros naturais n satisfazendo as condi=E7=F5e seguintes: (a) n possui 1.000 algarismos; (b) todos os algarismos de n s=E3o =EDmpares; (c) o valor absoluto da diferen=E7a entre algarismos adjacentes de n =E9 = 2. Determine o n=FAmero de elementos do conjunto S. 4) Uma cidade possui 10.000 linhas telef=F4nicas, cada uma acessada por um n=FAmero de quatro algarismos. Mais da metade das linhas telef=F4nicas est= =E3o no centro da cidade. Prove que: no centro da cidade, existe duas linhas telef=F4nicas cuja soma desses respectivos n=FAmeros =E9 o n=FAmero de uma l= inha telef=F4nica no centro da cidade.=20 From owner-obm-rj@saci.mat.puc-rio.br Wed Jan 6 19:18:58 1999 Received: by saci.mat.puc-rio.br (AIX 3.2/UCB 5.64/4.03) id AA21098; Wed, 6 Jan 1999 19:18:41 -0200 Received: from srv4-poa.zaz.com.br by saci.mat.puc-rio.br (AIX 3.2/UCB 5.64/4.03) id AA03174; Wed, 6 Jan 1999 19:18:27 -0200 Received: from default (dl-pm2-13.poa.zaz.com.br [200.248.210.45]) by srv4-poa.nutecnet.com.br (8.9.1/8.7.3) with ESMTP id TAA28940 for ; Wed, 6 Jan 1999 19:18:22 -0200 (GMT+2) Message-Id: <199901062118.TAA28940@srv4-poa.nutecnet.com.br> From: "Lucas" To: Subject: he he, cornholio! Date: Tue, 29 Sep 1998 01:34:21 -0300 X-Msmail-Priority: Normal X-Priority: 3 X-Mailer: Microsoft Internet Mail 4.70.1161 Mime-Version: 1.0 Content-Type: text/plain; charset=ISO-8859-1 Content-Transfer-Encoding: quoted-printable X-Mime-Autoconverted: from 8bit to quoted-printable by srv4-poa.nutecnet.com.br id TAA28940 Sender: owner-obm-rj@saci.mat.puc-rio.br Precedence: bulk Reply-To: obm-rj@saci.mat.puc-rio.br Status: RO X-Status: X-Keywords: X-UID: 86 Caro Benjamin, a resposta do seu problema =E9 mil, e at=E9 um retardado saberia, eu acho. Ali=E1s, voc=EA est=E1 r=E1pido no gatilho, n=E3o =E9? Uma curiosidade: q= uantos anos voc=EA tem? A resposta do meu problema era a seguinte: a) um b) 454 c) cinco d) doze e) Os tr=EAs primeiros s=E3o: cem, cento e cinco e cento e cinquenta. Os tr=EAs =FAltimos s=E3o: vinte, vinte e um e zero. A=ED v=E3o alguns problemas quentinhos, saindo do forno: 1. Tr=EAs mulheres t=EAm cada uma duas filhas e todas entram num restaura= nte para almo=E7ar. S=F3 h=E1 sete lugares vagos no restaurante mas cada uma = tem o seu lugar. Como conseguiram? 2. Qual =E9 o nome completo do taxista que trabalha diariamente dirigindo= o seu carro? 3. Qual =E9 a palavra que nunca diz a verdade? 4. Usando apenas os sinais das quatro opera=E7=F5es fundamentais, torne e= sta igualdade verdadeira (se vc quiser, pode trocar a ordem dos n=FAmeros). 9 8 7 6 5 4 3 2 1 =3D 1= 000=20 =20 From owner-obm-rj@saci.mat.puc-rio.br Wed Jan 6 22:00:22 1999 Received: by saci.mat.puc-rio.br (AIX 3.2/UCB 5.64/4.03) id AA21525; Wed, 6 Jan 1999 22:00:13 -0200 Received: from [200.248.17.2] by saci.mat.puc-rio.br (AIX 3.2/UCB 5.64/4.03) id AA03089; Wed, 6 Jan 1999 21:59:58 -0200 Received: from server [200.248.17.13] by sinos.net [200.248.17.2] with SMTP (MDaemon.v2.7.SP4.R) for ; Wed, 06 Jan 1999 21:59:43 -0300 From: "Benjamin Hinrichs" To: Subject: Curiosisdade Date: Wed, 6 Jan 1999 21:55:22 -0200 Message-Id: <01be39d0$0585fb00$0d11f8c8@server> Mime-Version: 1.0 Content-Type: text/plain; charset="iso-8859-1" Content-Transfer-Encoding: 8bit X-Priority: 3 X-Msmail-Priority: Normal X-Mailer: Microsoft Outlook Express 4.71.1712.3 X-Mimeole: Produced By Microsoft MimeOLE V4.71.1712.3 X-Mdaemon-Deliver-To: obm-rj@saci.mat.puc-rio.br X-Return-Path: hinsoft@sinos.net Sender: owner-obm-rj@saci.mat.puc-rio.br Precedence: bulk Reply-To: obm-rj@saci.mat.puc-rio.br Status: RO X-Status: X-Keywords: X-UID: 87 Caro Lucas, Curiosidade: tenho 15 aninhos. Um abraço, Benjamin Hinrichs From owner-obm-rj@saci.mat.puc-rio.br Wed Jan 6 23:12:33 1999 Received: by saci.mat.puc-rio.br (AIX 3.2/UCB 5.64/4.03) id AA20359; Wed, 6 Jan 1999 23:12:24 -0200 Received: from srv4-poa.zaz.com.br by saci.mat.puc-rio.br (AIX 3.2/UCB 5.64/4.03) id AA08835; Wed, 6 Jan 1999 23:12:12 -0200 Received: from default (dl-tnt1-190.poa.zaz.com.br [200.248.211.190]) by srv4-poa.nutecnet.com.br (8.9.1/8.7.3) with ESMTP id XAA26034 for ; Wed, 6 Jan 1999 23:12:05 -0200 (GMT+2) Message-Id: <199901070112.XAA26034@srv4-poa.nutecnet.com.br> From: "Lucas" To: Subject: =?ISO-8859-1?Q?Erguei_as_m=E3os_e_dai_agora_a_Deus=2C...?= Date: Tue, 29 Sep 1998 05:27:44 -0300 X-Msmail-Priority: Normal X-Priority: 3 X-Mailer: Microsoft Internet Mail 4.70.1161 Mime-Version: 1.0 Content-Type: text/plain; charset=ISO-8859-1 Content-Transfer-Encoding: quoted-printable X-Mime-Autoconverted: from 8bit to quoted-printable by srv4-poa.nutecnet.com.br id XAA26034 Sender: owner-obm-rj@saci.mat.puc-rio.br Precedence: bulk Reply-To: obm-rj@saci.mat.puc-rio.br Status: RO X-Status: X-Keywords: X-UID: 88 Caro Benjamin, tu participaste da Olimp=EDada Brasileira de Matem=E1tica? C= omo vc foi? Ah, vc viu os problemas do Benedito? S=E3o muito interessantes, mas = s=F3 deu tempo de tentar resolver um deles, porque eu tava de sa=EDda. Ah, n=E3= o esquece de mandar o teu e-mail.=20 Lucas P. M. =20 From owner-obm-rj@saci.mat.puc-rio.br Thu Jan 7 10:02:45 1999 Received: by saci.mat.puc-rio.br (AIX 3.2/UCB 5.64/4.03) id AA20233; Thu, 7 Jan 1999 10:02:41 -0200 Received: from [200.248.17.2] by saci.mat.puc-rio.br (AIX 3.2/UCB 5.64/4.03) id AA20995; Thu, 7 Jan 1999 10:02:25 -0200 Received: from server [200.248.17.37] by sinos.net [200.248.17.2] with SMTP (MDaemon.v2.7.SP4.R) for ; Thu, 07 Jan 1999 10:01:50 -0300 From: "Benjamin Hinrichs" To: Subject: =?iso-8859-1?Q?Re:_Erguei_as_m=E3os_e_dai_agora_a_Deus=2C...?= Date: Thu, 7 Jan 1999 09:57:52 -0200 Message-Id: <01be3a34$f4449bc0$2511f8c8@server> Mime-Version: 1.0 Content-Type: text/plain; charset="iso-8859-1" Content-Transfer-Encoding: 8bit X-Priority: 3 X-Msmail-Priority: Normal X-Mailer: Microsoft Outlook Express 4.71.1712.3 X-Mimeole: Produced By Microsoft MimeOLE V4.71.1712.3 X-Mdaemon-Deliver-To: obm-rj@saci.mat.puc-rio.br X-Return-Path: hinsoft@sinos.net Sender: owner-obm-rj@saci.mat.puc-rio.br Precedence: bulk Reply-To: obm-rj@saci.mat.puc-rio.br Status: RO X-Status: X-Keywords: X-UID: 89 Caro Lucas, Sim, participei das olimpíadas pela primeira vez. Fui até a terceira fase, mas... até agora não sei minha colocação mais precisa. Até agora não me lembro de nenhum problema do Benedito. Logo que chegeui me intrometi, o que faz menos de um mês. Meu e-mail é hinsoft@sinos.net Um abraço, Benjamin Hinrichs From owner-obm-rj@saci.mat.puc-rio.br Thu Jan 7 15:38:54 1999 Received: by saci.mat.puc-rio.br (AIX 3.2/UCB 5.64/4.03) id AA19598; Thu, 7 Jan 1999 15:38:35 -0200 Received: from mail.iis.com.br by saci.mat.puc-rio.br (AIX 3.2/UCB 5.64/4.03) id AA23178; Thu, 7 Jan 1999 15:38:19 -0200 Received: from default (rio-as6-tty03.iis.com.br [200.202.97.99]) by mail.iis.com.br (8.8.7/8.8.14) with SMTP id PAA16413 for ; Thu, 7 Jan 1999 15:38:06 -0200 From: "Marcio" To: Subject: Site com problemas na Internet Date: Thu, 7 Jan 1999 15:39:53 -0300 Message-Id: <01be3a6d$1d906080$LocalHost@default> Mime-Version: 1.0 Content-Type: multipart/alternative; boundary="----=_NextPart_000_0006_01BE3A53.F8432880" X-Priority: 3 X-Msmail-Priority: Normal X-Mailer: Microsoft Outlook Express 4.71.1712.3 X-Mimeole: Produced By Microsoft MimeOLE V4.71.1712.3 Sender: owner-obm-rj@saci.mat.puc-rio.br Precedence: bulk Reply-To: obm-rj@saci.mat.puc-rio.br Status: RO X-Status: X-Keywords: X-UID: 90 This is a multi-part message in MIME format. ------=_NextPart_000_0006_01BE3A53.F8432880 Content-Type: text/plain; charset="iso-8859-1" Content-Transfer-Encoding: quoted-printable Tem um site interessante na Internet com diversos problemas de = olimpiadas Russas, das IMO (olimpiada internacional), Putnam Exams (que = sao bem mais pesados, acho que tem algo a ver com a field medal, mas nao = tenho certeza e gostaria ateh que alguem me esclarecesse o que eh esse = Putnam Exams) e um outro lah que parecem ser do tipo do Putnam.. =20 Grande parte desses problemas tem a resolucao no site.. Se a pessoa = quiser, parece ser um preparo muito bom para IMO ou OBM.. Eu ainda nao = vi quase nenhum problema de lah (pq eu estou de ferias :) )e portanto = nao tenho mais informacoes.. O endereco eh http://www.kalva.demon.co.uk/=20 Quanto aos problemas do Benedito, alguem resolveu o problema 3? Ele = parece ser bem dificil..=20 Abracos, Marcio ------=_NextPart_000_0006_01BE3A53.F8432880 Content-Type: text/html; charset="iso-8859-1" Content-Transfer-Encoding: quoted-printable
    Tem um=20 site interessante na Internet com diversos problemas de olimpiadas = Russas, das=20 IMO (olimpiada internacional), Putnam Exams (que sao bem mais pesados, = acho que=20 tem algo a ver com a field medal, mas nao tenho certeza e gostaria ateh = que=20 alguem me esclarecesse o que eh esse Putnam Exams) e um outro lah que = parecem=20 ser do tipo do Putnam..
   
    Grande=20 parte desses problemas tem a resolucao no site.. Se a pessoa quiser, = parece ser=20 um preparo muito bom para IMO ou OBM.. Eu ainda nao vi quase nenhum = problema de=20 lah (pq eu estou de ferias :) )e portanto nao tenho mais=20 informacoes..
    O=20 endereco eh http://www.kalva.demon.co.uk/ =
    Quanto=20 aos problemas do Benedito, alguem resolveu o problema 3? Ele parece ser = bem=20 dificil..
   =20 Abracos,
   =20 Marcio
------=_NextPart_000_0006_01BE3A53.F8432880-- From owner-obm-rj@saci.mat.puc-rio.br Fri Jan 8 09:28:20 1999 Received: by saci.mat.puc-rio.br (AIX 3.2/UCB 5.64/4.03) id AA21020; Fri, 8 Jan 1999 09:27:53 -0200 Received: from relay.openlink.com.br by saci.mat.puc-rio.br (AIX 3.2/UCB 5.64/4.03) id AA17688; Fri, 8 Jan 1999 09:27:38 -0200 Received: from default (unverified [200.224.102.158]) by correio2.openlink.com.br (Rockliffe SMTPRA 2.1.6) with SMTP id for ; Fri, 08 Jan 1999 09:26:47 -0300 From: "=?iso-8859-1?Q?Jos=E9-Paulo_Q._Carneiro?=" To: Subject: Re: Site com problemas na Internet Date: Fri, 8 Jan 1999 09:23:13 -0200 Message-Id: <01be3af9$47c65300$LocalHost@default> Mime-Version: 1.0 Content-Type: multipart/alternative; boundary="----=_NextPart_000_0052_01BE3AE8.843D8300" X-Priority: 3 X-Msmail-Priority: Normal X-Mailer: Microsoft Outlook Express 4.71.1712.3 X-Mimeole: Produced By Microsoft MimeOLE V4.71.1712.3 Sender: owner-obm-rj@saci.mat.puc-rio.br Precedence: bulk Reply-To: obm-rj@saci.mat.puc-rio.br Status: RO X-Status: X-Keywords: X-UID: 91 This is a multi-part message in MIME format. ------=_NextPart_000_0052_01BE3AE8.843D8300 Content-Type: text/plain; charset="iso-8859-1" Content-Transfer-Encoding: quoted-printable Marcio. Para sua informacao, a Putnam Competition eh uma especie de Olimpiada de Matematica americana para alunos de curso superior. Se voce quiser detalhes, veja em: http://www-personal.ksu.edu/~math/compete/putnam Jose Paulo Carneiro -----Mensagem original----- De: Marcio Para: obm-rj@saci.mat.puc-rio.br Data: Quinta-feira, 7 de Janeiro de 1999 16:39 Assunto: Site com problemas na Internet =20 =20 Tem um site interessante na Internet com diversos problemas de = olimpiadas Russas, das IMO (olimpiada internacional), Putnam Exams (que = sao bem mais pesados, acho que tem algo a ver com a field medal, mas nao = tenho certeza e gostaria ateh que alguem me esclarecesse o que eh esse = Putnam Exams) e um outro lah que parecem ser do tipo do Putnam.. =20 Grande parte desses problemas tem a resolucao no site.. Se a = pessoa quiser, parece ser um preparo muito bom para IMO ou OBM.. Eu = ainda nao vi quase nenhum problema de lah (pq eu estou de ferias :) )e = portanto nao tenho mais informacoes.. O endereco eh http://www.kalva.demon.co.uk/=20 Quanto aos problemas do Benedito, alguem resolveu o problema 3? = Ele parece ser bem dificil..=20 Abracos, Marcio ------=_NextPart_000_0052_01BE3AE8.843D8300 Content-Type: text/html; charset="iso-8859-1" Content-Transfer-Encoding: quoted-printable
Marcio.
Para sua = informacao, a=20 Putnam Competition eh uma especie de
Olimpiada de Matematica americana para alunos de = curso=20 superior. 
Se voce quiser detalhes, veja em: 
http://www-pers= onal.ksu.edu/~math/compete/putnam
 
Jose Paulo Carneiro
-----Mensagem = original-----
De:=20 Marcio <mcohen@iis.com.br>
Para: = obm-rj@saci.mat.puc-rio.br= =20 <obm-rj@saci.mat.puc-rio.br= >
Data:=20 Quinta-feira, 7 de Janeiro de 1999 16:39
Assunto: Site = com=20 problemas na Internet

    Tem=20 um site interessante na Internet com diversos problemas de = olimpiadas=20 Russas, das IMO (olimpiada internacional), Putnam Exams (que sao bem = mais=20 pesados, acho que tem algo a ver com a field medal, mas nao tenho = certeza e=20 gostaria ateh que alguem me esclarecesse o que eh esse Putnam Exams) = e um=20 outro lah que parecem ser do tipo do Putnam..
   
   =20 Grande parte desses problemas tem a resolucao no site.. Se a pessoa = quiser,=20 parece ser um preparo muito bom para IMO ou OBM.. Eu ainda nao vi = quase=20 nenhum problema de lah (pq eu estou de ferias :) )e portanto nao = tenho mais=20 informacoes..
    O=20 endereco eh http://www.kalva.demon.co.uk/ =
   =20 Quanto aos problemas do Benedito, alguem resolveu o problema 3? Ele = parece=20 ser bem dificil..
   =20 Abracos,
   =20 Marcio
------=_NextPart_000_0052_01BE3AE8.843D8300-- From owner-obm-rj@saci.mat.puc-rio.br Fri Jan 8 12:02:39 1999 Received: by saci.mat.puc-rio.br (AIX 3.2/UCB 5.64/4.03) id AA24426; Fri, 8 Jan 1999 12:02:20 -0200 Received: from mail.iis.com.br by saci.mat.puc-rio.br (AIX 3.2/UCB 5.64/4.03) id AA20836; Fri, 8 Jan 1999 12:01:33 -0200 Received: from default (rio-as5-tty02.iis.com.br [200.202.97.82]) by mail.iis.com.br (8.8.7/8.8.14) with SMTP id MAA10627 for ; Fri, 8 Jan 1999 12:01:11 -0200 From: "Marcio" To: Subject: Re: Site com problemas na Internet Date: Fri, 8 Jan 1999 11:58:58 -0300 Message-Id: <01be3b17$6b61a300$LocalHost@default> Mime-Version: 1.0 Content-Type: multipart/alternative; boundary="----=_NextPart_000_001A_01BE3AFE.46146B00" X-Priority: 3 X-Msmail-Priority: Normal X-Mailer: Microsoft Outlook Express 4.71.1712.3 X-Mimeole: Produced By Microsoft MimeOLE V4.71.1712.3 Sender: owner-obm-rj@saci.mat.puc-rio.br Precedence: bulk Reply-To: obm-rj@saci.mat.puc-rio.br Status: RO X-Status: X-Keywords: X-UID: 92 This is a multi-part message in MIME format. ------=_NextPart_000_001A_01BE3AFE.46146B00 Content-Type: text/plain; charset="iso-8859-1" Content-Transfer-Encoding: quoted-printable Muito obrigado. Eu entrei no site e obtive mais informacoes. Pelo = que eu li, essa olimpiada eh destinada apenas a estudantes dos EUA (e = Canada). Mas o site parecia meio antigo, nao soh pelo formato mas tambem = pela grande quantidade de links que nao funcionavam. Voce sabe se isso = continua assim?=20 Outra coisa, o que eh a field medal? Eu ouvi falar disso num filme e = um professor meu disse que era como se fosse um premio nobel de = matematica.. mas ele nao tinha muita certeza. eh isso mesmo? -----Original Message----- From: Jos=E9-Paulo Q. Carneiro To: obm-rj@saci.mat.puc-rio.br Date: Sexta-feira, 8 de Janeiro de 1999 08:28 Subject: Re: Site com problemas na Internet =20 =20 Marcio. Para sua informacao, a Putnam Competition eh uma especie de Olimpiada de Matematica americana para alunos de curso superior.=20 Se voce quiser detalhes, veja em:=20 http://www-personal.ksu.edu/~math/compete/putnam =20 Jose Paulo Carneiro -----Mensagem original----- De: Marcio Para: obm-rj@saci.mat.puc-rio.br Data: Quinta-feira, 7 de Janeiro de 1999 16:39 Assunto: Site com problemas na Internet =20 =20 Tem um site interessante na Internet com diversos problemas = de olimpiadas Russas, das IMO (olimpiada internacional), Putnam Exams = (que sao bem mais pesados, acho que tem algo a ver com a field medal, = mas nao tenho certeza e gostaria ateh que alguem me esclarecesse o que = eh esse Putnam Exams) e um outro lah que parecem ser do tipo do Putnam.. =20 Grande parte desses problemas tem a resolucao no site.. Se a = pessoa quiser, parece ser um preparo muito bom para IMO ou OBM.. Eu = ainda nao vi quase nenhum problema de lah (pq eu estou de ferias :) )e = portanto nao tenho mais informacoes.. O endereco eh http://www.kalva.demon.co.uk/=20 Quanto aos problemas do Benedito, alguem resolveu o problema = 3? Ele parece ser bem dificil..=20 Abracos, Marcio ------=_NextPart_000_001A_01BE3AFE.46146B00 Content-Type: text/html; charset="iso-8859-1" Content-Transfer-Encoding: quoted-printable
    Muito=20 obrigado. Eu entrei no site e obtive mais informacoes. Pelo que eu li, = essa=20 olimpiada eh destinada apenas a estudantes dos EUA (e Canada). Mas o = site=20 parecia meio antigo, nao soh pelo formato mas tambem pela grande = quantidade de=20 links que nao funcionavam. Voce sabe se isso continua assim?=20
   =20 Outra coisa, o que eh a field medal? Eu ouvi falar disso num filme e um=20 professor meu disse que era como se fosse um premio nobel de = matematica.. mas=20 ele nao tinha muita certeza. eh isso mesmo? 
-----Original = Message-----
From:=20 José-Paulo Q. Carneiro <jpcarneiro@openlink.com.br= >
To:=20 obm-rj@saci.mat.puc-rio.br= =20 <obm-rj@saci.mat.puc-rio.br= >
Date:=20 Sexta-feira, 8 de Janeiro de 1999 08:28
Subject: Re: = Site com=20 problemas na Internet

Marcio.
Para sua = informacao, a=20 Putnam Competition eh uma especie de
Olimpiada de Matematica americana para alunos de = curso=20 superior. 
Se voce quiser detalhes, veja = em: 
http://www-pers= onal.ksu.edu/~math/compete/putnam
 
Jose Paulo Carneiro
-----Mensagem=20 original-----
De: Marcio <mcohen@iis.com.br>
Para:=20 obm-rj@saci.mat.puc-rio.br= =20 <obm-rj@saci.mat.puc-rio.br= >
Data:=20 Quinta-feira, 7 de Janeiro de 1999 16:39
Assunto: = Site com=20 problemas na Internet

    Tem um site interessante na Internet com = diversos=20 problemas de olimpiadas Russas, das IMO (olimpiada = internacional),=20 Putnam Exams (que sao bem mais pesados, acho que tem algo a ver = com a=20 field medal, mas nao tenho certeza e gostaria ateh que alguem me = esclarecesse o que eh esse Putnam Exams) e um outro lah que = parecem ser=20 do tipo do Putnam..
   
   =20 Grande parte desses problemas tem a resolucao no site.. Se a = pessoa=20 quiser, parece ser um preparo muito bom para IMO ou OBM.. Eu = ainda nao=20 vi quase nenhum problema de lah (pq eu estou de ferias :) )e = portanto=20 nao tenho mais informacoes..
    O=20 endereco eh http://www.kalva.demon.co.uk/ =
   =20 Quanto aos problemas do Benedito, alguem resolveu o problema 3? = Ele=20 parece ser bem dificil..
   =20 Abracos,
   =20 = Marcio
------=_NextPart_000_001A_01BE3AFE.46146B00-- From owner-obm-rj@saci.mat.puc-rio.br Sat Jan 9 09:47:50 1999 Received: by saci.mat.puc-rio.br (AIX 3.2/UCB 5.64/4.03) id AA23099; Sat, 9 Jan 1999 09:47:40 -0200 Received: from relay.openlink.com.br by saci.mat.puc-rio.br (AIX 3.2/UCB 5.64/4.03) id AA17719; Sat, 9 Jan 1999 09:47:28 -0200 Received: from default (unverified [200.224.102.175]) by correio2.openlink.com.br (Rockliffe SMTPRA 2.1.6) with SMTP id for ; Sat, 09 Jan 1999 09:46:36 -0300 From: "=?iso-8859-1?Q?Jos=E9-Paulo_Q._Carneiro?=" To: Subject: Re: Site com problemas na Internet Date: Sat, 9 Jan 1999 09:37:43 -0200 Message-Id: <01be3bc4$790d5be0$af66e0c8@default> Mime-Version: 1.0 Content-Type: multipart/alternative; boundary="----=_NextPart_000_001B_01BE3BB3.B5848BE0" X-Priority: 3 X-Msmail-Priority: Normal X-Mailer: Microsoft Outlook Express 4.71.1712.3 X-Mimeole: Produced By Microsoft MimeOLE V4.71.1712.3 Sender: owner-obm-rj@saci.mat.puc-rio.br Precedence: bulk Reply-To: obm-rj@saci.mat.puc-rio.br Status: RO X-Status: X-Keywords: X-UID: 93 This is a multi-part message in MIME format. ------=_NextPart_000_001B_01BE3BB3.B5848BE0 Content-Type: text/plain; charset="iso-8859-1" Content-Transfer-Encoding: quoted-printable Eh exatamente isto. A Medalha Fields eh o premio internacional=20 maximo para um Matematico. Jose Paulo -----Mensagem original----- De: Marcio Para: obm-rj@saci.mat.puc-rio.br Data: Sexta-feira, 8 de Janeiro de 1999 13:04 Assunto: Re: Site com problemas na Internet =20 =20 Muito obrigado. Eu entrei no site e obtive mais informacoes. = Pelo que eu li, essa olimpiada eh destinada apenas a estudantes dos EUA = (e Canada). Mas o site parecia meio antigo, nao soh pelo formato mas = tambem pela grande quantidade de links que nao funcionavam. Voce sabe se = isso continua assim?=20 Outra coisa, o que eh a field medal? Eu ouvi falar disso num = filme e um professor meu disse que era como se fosse um premio nobel de = matematica.. mas ele nao tinha muita certeza. eh isso mesmo?=20 -----Original Message----- From: Jos=E9-Paulo Q. Carneiro To: obm-rj@saci.mat.puc-rio.br Date: Sexta-feira, 8 de Janeiro de 1999 08:28 Subject: Re: Site com problemas na Internet =20 =20 Marcio. Para sua informacao, a Putnam Competition eh uma especie de Olimpiada de Matematica americana para alunos de curso superior. = Se voce quiser detalhes, veja em:=20 http://www-personal.ksu.edu/~math/compete/putnam =20 Jose Paulo Carneiro -----Mensagem original----- De: Marcio Para: obm-rj@saci.mat.puc-rio.br = Data: Quinta-feira, 7 de Janeiro de 1999 16:39 Assunto: Site com problemas na Internet =20 =20 Tem um site interessante na Internet com diversos = problemas de olimpiadas Russas, das IMO (olimpiada internacional), = Putnam Exams (que sao bem mais pesados, acho que tem algo a ver com a = field medal, mas nao tenho certeza e gostaria ateh que alguem me = esclarecesse o que eh esse Putnam Exams) e um outro lah que parecem ser = do tipo do Putnam.. =20 Grande parte desses problemas tem a resolucao no site.. = Se a pessoa quiser, parece ser um preparo muito bom para IMO ou OBM.. Eu = ainda nao vi quase nenhum problema de lah (pq eu estou de ferias :) )e = portanto nao tenho mais informacoes.. O endereco eh http://www.kalva.demon.co.uk/=20 Quanto aos problemas do Benedito, alguem resolveu o = problema 3? Ele parece ser bem dificil..=20 Abracos, Marcio ------=_NextPart_000_001B_01BE3BB3.B5848BE0 Content-Type: text/html; charset="iso-8859-1" Content-Transfer-Encoding: quoted-printable
Eh exatamente isto. A Medalha = Fields  eh o=20 premio internacional
maximo para um = Matematico.
Jose = Paulo
-----Mensagem = original-----
De:=20 Marcio <mcohen@iis.com.br>
Para: = obm-rj@saci.mat.puc-rio.br= =20 <obm-rj@saci.mat.puc-rio.br= >
Data:=20 Sexta-feira, 8 de Janeiro de 1999 13:04
Assunto: Re: = Site com=20 problemas na Internet

    Muito=20 obrigado. Eu entrei no site e obtive mais informacoes. Pelo que eu = li, essa=20 olimpiada eh destinada apenas a estudantes dos EUA (e Canada). Mas o = site=20 parecia meio antigo, nao soh pelo formato mas tambem pela grande = quantidade=20 de links que nao funcionavam. Voce sabe se isso continua assim?=20
    Outra coisa, o que eh a = field medal?=20 Eu ouvi falar disso num filme e um professor meu disse que era como = se fosse=20 um premio nobel de matematica.. mas ele nao tinha muita certeza. eh = isso=20 mesmo? 
-----Original=20 Message-----
From: José-Paulo Q. Carneiro = <jpcarneiro@openlink.com.br= >
To:=20 obm-rj@saci.mat.puc-rio.br= =20 <obm-rj@saci.mat.puc-rio.br= >
Date:=20 Sexta-feira, 8 de Janeiro de 1999 08:28
Subject: = Re: Site=20 com problemas na Internet

Marcio.
Para = sua informacao,=20 a Putnam Competition eh uma especie de
Olimpiada de Matematica americana para = alunos de curso=20 superior. 
Se voce quiser detalhes, veja = em: 
http://www-pers= onal.ksu.edu/~math/compete/putnam
 
Jose Paulo = Carneiro
-----Mensagem=20 original-----
De: Marcio <mcohen@iis.com.br>
Para:=20 obm-rj@saci.mat.puc-rio.br= =20 <obm-rj@saci.mat.puc-rio.br= >
Data:=20 Quinta-feira, 7 de Janeiro de 1999 16:39
Assunto: = Site=20 com problemas na Internet

    Tem um site interessante na Internet com = diversos=20 problemas de olimpiadas Russas, das IMO (olimpiada = internacional),=20 Putnam Exams (que sao bem mais pesados, acho que tem algo a = ver com=20 a field medal, mas nao tenho certeza e gostaria ateh que = alguem me=20 esclarecesse o que eh esse Putnam Exams) e um outro lah que = parecem=20 ser do tipo do Putnam..
   
    Grande parte desses = problemas tem a=20 resolucao no site.. Se a pessoa quiser, parece ser um = preparo muito=20 bom para IMO ou OBM.. Eu ainda nao vi quase nenhum problema = de lah=20 (pq eu estou de ferias :) )e portanto nao tenho mais=20 informacoes..
    O endereco eh http://www.kalva.demon.co.uk/ =
    Quanto aos problemas do = Benedito,=20 alguem resolveu o problema 3? Ele parece ser bem dificil..=20
    = Abracos,
   =20 = Marcio
<= /HTML> ------=_NextPart_000_001B_01BE3BB3.B5848BE0-- From cordeiro@travelnet.com.br Sun Jan 10 04:17:16 1999 Received: from linux1.travelnet.com.br by saci.mat.puc-rio.br (AIX 3.2/UCB 5.64/4.03) id AA16850; Sun, 10 Jan 1999 04:17:14 -0200 Received: from cordeiro.travelnet.com.br (ab-05.travelnet.com.br [200.230.77.100]) by linux1.travelnet.com.br (8.9.0/8.9.0) with SMTP id EAA05696 for ; Sun, 10 Jan 1999 04:13:22 -0200 Reply-To: "Luis Carlos Cordeiro" From: "Luis Carlos Cordeiro" To: Subject: Como funciona a lista... Date: Tue, 5 Jan 1999 19:19:12 -0200 Message-Id: <01be38f1$0a8ae0a0$644de6c8@cordeiro.travelnet.com.br> Mime-Version: 1.0 Content-Type: multipart/alternative; boundary="----=_NextPart_000_0028_01BE38E0.470210A0" X-Priority: 3 X-Msmail-Priority: Normal X-Mailer: Microsoft Outlook Express 4.71.1712.3 X-Mimeole: Produced By Microsoft MimeOLE V4.71.1712.3 Status: RO X-Status: X-Keywords: X-UID: 94 This is a multi-part message in MIME format. ------=_NextPart_000_0028_01BE38E0.470210A0 Content-Type: text/plain; charset="iso-8859-1" Content-Transfer-Encoding: quoted-printable Caro Professor Nicolau, creio que acabei de me cadastrar na = lista para discuss=E3o de problemas de matem=E1tica, e gostaria de = esclarecer algumas d=FAvidas a respeito do funcionamento dela: Vou = receber todos os e-mails enviados pelos outros participantes da lista ?? = Se eu quiser mandar algum coment=E1rio ou d=FAvida a respeito de um = e-mail recebido, pra qual endere=E7o devo enviar ?? Ficarei agradecido com esses esclarecimentos a respeito da = lista, pois apesar de estar interessad=EDssimo, ainda n=E3o entendi = direito como ela funciona. = OBRIGADO. = Luiz Alberto Cordeiro ------=_NextPart_000_0028_01BE38E0.470210A0 Content-Type: text/html; charset="iso-8859-1" Content-Transfer-Encoding: quoted-printable
        Caro Professor = Nicolau, creio=20 que acabei de me cadastrar na lista para discussão de problemas = de=20 matemática, e gostaria de esclarecer algumas dúvidas a = respeito do=20 funcionamento dela: Vou receber todos os e-mails enviados pelos outros=20 participantes da lista ??  Se eu quiser mandar algum = comentário ou=20 dúvida a respeito de um e-mail recebido, pra qual endereço = devo=20 enviar ??
        Ficarei agradecido = com esses=20 esclarecimentos a respeito da lista, pois apesar de estar=20 interessadíssimo, ainda não entendi direito como ela=20 funciona.
 
          &nbs= p;            = ;            =             &= nbsp;           &n= bsp;           &nb= sp;           &nbs= p;            = ;    =20 OBRIGADO.
 
          &nbs= p;            = ;            =             &= nbsp;           &n= bsp;           &nb= sp;           &nbs= p;           =20 Luiz Alberto Cordeiro
------=_NextPart_000_0028_01BE38E0.470210A0-- From nicolau@saci.mat.puc-rio.br Sun Jan 10 10:05:06 1999 Received: from dial03.impa.br by saci.mat.puc-rio.br (AIX 3.2/UCB 5.64/4.03) id AA14102; Sun, 10 Jan 1999 10:05:04 -0200 Received: from localhost (nicolau@localhost) by washoe.domus (8.8.7/8.8.7) with ESMTP id KAA11278; Sun, 10 Jan 1999 10:04:19 -0200 X-Authentication-Warning: washoe.domus: nicolau owned process doing -bs Date: Sun, 10 Jan 1999 10:04:18 -0200 (EDT) From: "Nicolau C. Saldanha" X-Sender: nicolau@washoe To: Luis Carlos Cordeiro Cc: nicolau@saci.mat.puc-rio.br Subject: Re: Como funciona a lista... In-Reply-To: <01be38f1$0a8ae0a0$644de6c8@cordeiro.travelnet.com.br> Message-Id: Mime-Version: 1.0 Content-Type: TEXT/PLAIN; charset=iso-8859-1 Content-Transfer-Encoding: QUOTED-PRINTABLE Status: RO X-Status: X-Keywords: X-UID: 95 On Tue, 5 Jan 1999, Luis Carlos Cordeiro wrote: > Caro Professor Nicolau, creio que acabei de me cadastrar na > lista para discuss=E3o de problemas de matem=E1tica, e gostaria de > esclarecer algumas d=FAvidas a respeito do funcionamento dela: Vou > receber todos os e-mails enviados pelos outros participantes da lista > ?? Se eu quiser mandar algum coment=E1rio ou d=FAvida a respeito de um > e-mail recebido, pra qual endere=E7o devo enviar ?? > Ficarei agradecido com esses esclarecimentos a respeito da > lista, pois apesar de estar interessad=EDssimo, ainda n=E3o entendi direi= to > como ela funciona. Voc=EA de fato j=E1 est=E1 inscrito na lista e deve receber todas as mensag= ens. Para enviar uma mensagem para a lista escreva para o endere=E7o obm-rj@mat.puc-rio.br Voc=EA, sendo um dos membros da lista, deve receber c=F3pias das mensagens que enviar. []s, N. From owner-obm-rj@saci.mat.puc-rio.br Mon Jan 11 19:54:21 1999 Received: by saci.mat.puc-rio.br (AIX 3.2/UCB 5.64/4.03) id AA15769; Mon, 11 Jan 1999 19:40:49 -0200 Received: from boto.mat.puc-rio.br by saci.mat.puc-rio.br (AIX 3.2/UCB 5.64/4.03) id AA17045; Mon, 11 Jan 1999 19:40:36 -0200 Received: from localhost (nicolau@localhost) by boto.mat.puc-rio.br (8.8.7/8.8.7) with ESMTP id TAA23048 for ; Mon, 11 Jan 1999 19:40:35 -0200 Date: Mon, 11 Jan 1999 19:40:35 -0200 (EDT) From: "Nicolau C. Saldanha" To: obm-rj@saci.mat.puc-rio.br Subject: Problema Message-Id: Mime-Version: 1.0 Content-Type: TEXT/PLAIN; charset=US-ASCII Sender: owner-obm-rj@saci.mat.puc-rio.br Precedence: bulk Reply-To: obm-rj@saci.mat.puc-rio.br Status: RO X-Status: X-Keywords: X-UID: 96 Um problema legal, nao sei bem de que grau de dificuldade (mas a resposta eh simples): De quantas formas eh possivel sentar A mulheres e B homens em mesas redondas sem criar nenhuma mesa onde todos sejam homens ou todas sejam mulheres? Por exemplo, para A = B = 2, dois homens h1 e h2 e duas mulheres m1 e m2, ha oito solucoes: Com uma mesa de 4 pessoas: m1 m2 h1 h2 m1 m2 h2 h1 m1 h1 m2 h2 m1 h1 h2 m2 m1 h2 m2 h1 m1 h2 h1 m2 Com duas mesas: m1 h1 m2 h2 m1 h2 m2 h1 Abracos, Nicolau